Mathematical Physics and Special Theory of Relativity

Download as pdf or txt
Download as pdf or txt
You are on page 1of 283

MSCPH501

Board of Studies
Prof. P. D. Pant Prof. S.R. Jha,
Director School of Sciences School of Sciences, I.G.N.O.U., Maidan
Uttarakhand Open University, Haldwani Garhi, New Delhi
Prof. P. S. Bisht, Prof. R. C. Shrivastva,
SSJ Campus, Kumaun University, Almora. Professor and Head, Department of Physics,
Dr. Kamal Devlal CBSH, G.B.P.U.A.&T. Pantnagar, India
Programme Coordinator
Department of Physics
School of Sciences, Uttarakhand Open
University

Department of Physics (School of Sciences)

Dr. Kamal Devlal (Assistant Professor) Dr. Meenakshi Rana (Assistant Professor (AC))
Dr. Vishal Sharma (Assistant Professor) Dr. Rajesh Mathpal (Assistant Professor (AC))
Dr. Gauri Negi (Assistant Professor)

Unit writing and Editing


Editing Writing

Prof. L P Verma 1. Dr. Kamal Devlal


Department of Physics Department of Physics
Govt PG College, Berinag, School of Sciences, Uttarakhand Open University
Pithoragarh, Uttarakhand 2. Prof. Y.K.SHARMA
Department of Physics
G.P.G.C. Karnpryag, Chamoli, Uttarakhand
3. Dr. Rajesh Mathpal
Department of Physics
School of Sciences, Uttarakhand Open University
4. Dr. Kailash Pandey
Department Physics
UPES Dehradun, Uttarakhand
5. Dr. Pranav Kumar
Kirori Mal College, University of Delhi
6.Dr. Arvind Nautiyal
S.G.R.R.P.G. College, Dehradun

Course Title and Code : Mathmatical Physics (MSCPH501)


ISBN :
Copyright : Uttarakhand Open University
Edition : 2021
Published By : Uttarakhand Open University,
2 Haldwani, Nainital- 263139
Printed By :
MSCPH501

MSCPH501

MATHMATICAL PHYSICS

DEPARTMENT OF PHYSICS
SCHOOL OF SCIENCES
UTTARAKHAND OPEN UNIVERSITY
Phone No. 05946-261122, 261123
Toll free No. 18001804025
Fax No. 05946-264232, E. mail [email protected]
htpp://uou.ac.in

3
MSCPH501

Contents
Course: MATHMATICAL PHYSICS Course code: MSCPH501
Credit: 3

Unit Block and Unit title Page


number number
1 Vector 1-60
2 Matrix 61-80
3 Complex Analysis 81-110
4 Tensor 111-140
5 Linear Differential Equations of First and Second Order 141-156
6 Partial Differential equations 157-172
7 Legendre’s And Bessel’s Differential Equations 173-207
8 Hermite And Laguerre Differential Equations 208-227
9 Fourier Integral And Transforms 228-251
10 Laplace Transform 253-279

4
MSCPH501

UNIT 1: VECTOR

STRUCTURE:
1.0 Objective
1.1 Introduction
1.2 Vector representation
1.2.1 Unit Vector
1.2.2 Zero Vector
1.2.3 Addition and subtraction of vectors
1.2.4 Resolution of a vector
1.2.5 Direction cosines
1.2.6 Position vector
1.3 Multiplication of vector
1.3.1 Multiplication and division of a vector by scalar
1.3.2 Product of two vectors
1.3.2.1 Scalar Product or dot product
1.3.2.2 Vector product or cross product
1.3.3 Product of three vectors
1.3.3.1 Scalar triple product
1.3.3.2 Vector triple product
1.4 Differentiation of vector
1.4.1 Partial derivatives
1.4.2 Del Operator
1.4.3 Scalar and Vector functions and fields
1.4.4 Gradient
1.4.5 Physical significance
1.4.6 Divergence of a vector
1.4.6.1 Physical interpretation of Divergence
1.4.7 Curl of a vector function
1.4.7.1 Physical significance of curl
1.5 Vector integral
1.5.1 Line integration
1.5.2 Surface integration
1.5.3 Volume integration
1.6 Vector identities
1.7 Gauss Divergence theorem
1.7.1 Gauss’s law
1.7.2 Gauss’s law in differential form

1
MSCPH501

1.7.3 Poisson’s equation and Laplace’s equation


1.8 Green’s theorem for a plane
1.9 Stoke’s theorem
1.10 Curvilinear coordinate system
1.10.1 Spherical coordinate system
1.10.2 Cylindrical coordinate system
1.11 Summary
1.12 Glossary
1.13 References
1.14 suggested reading
1.15 Terminal questions
1.15.1 Short answer type questions
1.15.2 Essay type question
1.15.3 Numerical question

2
MSCPH501

1.0 Objective:

After reading this unit you will be able to understand:


* Basic idea about vector and vector types
* Vector representation, addition, subtraction
* Multiplication of vectors
* Scalar product, vector product and triple product
* Differentiation of vector
* Gradient, Divergence and curl
* Vector integration
* Gauss Divergence theorem
* Poisson’s equation and Laplace’s equation
* Green’s theorem and Stoke’s theorem
* Curvilinear coordinate systems

1.1 Introduction:
On the basis of direction, the physical quantities may be divided into two main classes.
1.1.1 Scalar quantities: The physical quantities which do not require direction for their
representation. These quantities require only magnitude and unit and are added according to the
usual rules of algebra. Examples of these quantities are: mass, length, area, volume, distance,
time speed, density, electric current, temperature, work etc.
1.1.2 Vector quantities: The physical quantities which require both magnitude and direction
and which can be added according to the vector laws of addition are called vector quantities or
vector. These quantities require magnitude, unit and direction. Examples are weight,
displacement, velocity, acceleration, magnetic field, current density, electric field, momentum
angular velocity, force etc.

1.2 Vector representation:


Any vector quantity say A, is represented by putting a small arrow above the physical quantity
like . In case of print text a vector quantity is represented by bold type letter like A. The vector
can be represented by both capital and small letters. The magnitude of a vector quantity A is
denoted by or mod A or some time light forced italic letter A. We should understand
following types of vectors and their representations.

1.2.1 Unit vector


A unit vector of any vector quantity is that vector which has unit magnitude. Suppose
then unit vector is defined as

3
MSCPH501

=| |.
The unit vector is denoted by and read as ‘A unit vector or A hat’. It is clear that the magnitude

system, the unit vector along x, y and z axis are represented by ̂ , ̂ and respectively as shown
of unit vector is always 1. A unit vector merely indicates direction only. In Cartesian coordinate

in figure 1.1.

̂ x

Figure 1.1: Vector representation

= ̂A + ̂ A + A .
Any vector in Cartesian coordinate system can be represented as

Where ,̂ ̂ and are unit vector along x, y, z axis and, A , A , A are the magnitudes
projections or components of along x, y, z axis respectively.
The unit vector in Cartesian coordinate system can be given as:

=
̂ !̂ " # $
.
% &
' &
( &

1.2.2 Zero vector or Null vector:


A vector with zero magnitude is called zero vector or null vector. The condition for null vector is
= 0.
1.2.3 Equal vectors:
If two vectors have same magnitude and same direction, the vectors are called equal vector.
1.2.4 Like vectors:
If two or more vectors have same direction, but may have different magnitude, then the vectors
are called like vectors.
1.2.5 Negative vector:

4
MSCPH501

A vector is called negative vector with reference to another one, if both have same magnitude but
opposite directions.
1.2.6 Collinear vectors:
All the vectors parallel to each other are called collinear vectors. Basically collinear means the
line of action is along the same line.
1.2.7 Coplanar vector:
All the vectors whose line of action lies on a same plane are called coplanar vectors. Basically
coplanar means lies on the same plane.

1.2.8 Addition and subtraction of vectors:

The addition of two vectors can be performed by triangle law or parallelogram law. According to
triangle law if a vector is placed at the head of another vector, and these two vectors represent

* are two vectors as


two sides of a triangle then the third side or a vector drawn for the tail end of first to the head end
of second represents the resultant of these two vectors. If vectors and )
shown in figure 1.4, then resultant +* can be obtained by applying triangle law.

***
) ***+ ***
)

*** ***

Figure 1.2: Addition of two vectors

* is
and ) ,, and resultant +* makes angle - with vector
magnitude of +* is
If the angle between then

|+| = . / + ) / + 2 ) cos , .

The angle - is given as

) sin ,
-= 456 .
+) ,

You should notice that all three vectors * and +* are concurrent i.e. vectors acting on the
,)
same point O. The same addition can be shown by Figure 1.3

5
MSCPH501

***
) ***+

O α ,

***

Figure 1.3: Addition of two vectors

* from another vector,


Similarly the subtraction of a vector vectors )
* ).
is the addition of vectors
and (−)

* =
−) * ).
+ (−)

1.2.9 Resolution of vector:


A vector can be resolved into two or more vectors and these vectors can be added in accordance
with the polygon law of vector addition, and finally original vector can be obtained. If a vector is
resolved into three components which are mutually perpendicular to each other, called
rectangular components or mutually perpendicular components of a vector. These components
are along the three coordinate axes x, y and z respectively as show in Figure 1.4.

<

= X

>

Z Figure 1.4: Resolution of a vector

6
MSCPH501

If the unit vectors along x, y and x axis are represented by ,̂ ̂ and respectively then any vector
can be give as

= ̂A + ̂ A + A .

constitutes the diagonal of a parallelepiped and, A , A and A are the edges along x, y and z
axes respectively. is the polynomial addition of A , A and A . The rectangular
components A , A and A can be considered as orthogonal projections of vector on x, y and z
axis respectively. Mathematically, the magnitude of vector can be given as:

@= =%A /
+ A /
+ A /
.

1.2.10 Position vector:


In Cartesian co-ordinate system the position of any point P(x, y, z) can be represented by a
vector r, with respect to origin O, the vector r is called position vector of point P. Position vector

coordinate system. If we have two vectors A* and B * with position vectors CD and CE respectively
is often denoted by . Figure1.5 shows the position vector of a point P(x, y, z) in Cartesian

CD = ̂ x6 + ̂ y6 + z6
such as

CE = ̂ x/ + ̂ y/ + z/ .
Where (I6 , J6 , K6 ) and (I/ , J/ , K/ ) are the coordinates of point P and Q respectively.

PQ = OQ – OP ( ∴ OP + PQ = OQ)
Now the vector PQ can be given as

r r r
r = r2 − r1 .
Therefore, vector PQ = position vector of Q − position vector of P

B (I/ , J/ , K/ )
Y

/ A (I6 , J6 , K6 )

O X

Figure1.5: Position vectors

7
MSCPH501

1.3 Multiplication of vectors


1.3.1 Multiplication and division of a vector by scalar:
If a vector P is multiplied by a scalar quantity m then its magnitude becomes m times. For
example if m is a scalar and is a vector then its magnitude becomes m times.
Similarly, in case of division of a vector A by a non zero scalar quantity n, its magnitude
becomes 1/n times.
1.3.2 Product of two vectors:
There are two distinct ways in which we can define the product of two vectors.
1.3.2.1 Scalar product or dot product:
Scalar product of two vectors P and Q is defined as the product of magnitude of two vectors P
and Q and cosine of the angle between the directions of these vectors.

* , then dot product (read as A* N


If , is the angle between two vectors A* and B * ) of two
B
vectors is given by-

****
A. B* = AB cos , = A (B cos ,)

= A (O P 4 Q B 4 A) = A. RS.

The Figure 1.6 shows the dot product. The resultant of dot product or scalar product of two
vectors is always a scalar quantity. In physics the dot product is frequently used, the simplest
example is work which is dot product of force and displacement vectors.

*
B M

O N A*

Figure1.6: Dot product of two vectors

Important properties of dot product

(i) Condition for two collinear vectors:


If two vectors are parallel or angle between two vectors is 0 or π, then vectors are called

**** * = AB
collinear. In this case
A. B 0U = AB.

Then the product of two vectors is same as the product of their magnitudes.

8
MSCPH501

(ii) Condition for two vector to be perpendicular to each other:


If two vectors are perpendicular to each other then the angle between these two vectors is 900,

**** * = AB
then
A. B 90U = 0.

In case of unit vectors ̂ , ̂ and we know that these vectors are perpendicular to each other then
Hence two vectors are perpendicular to each other if and only if their dot product is zero.

̂ . ̂ = ̂. = . ̂ = 0

̂ . ̂ = ̂. ̂ = W. = 1.
similarly

(iii) Commutative law holds:

**** * =B * . A* .
In case of vector dot product the commutative law holds. Then
A. B
(iv) Distributive property of scalar product:
If P, Q and R are three vectors then according to distributive law

* + +* Z = A* . B
A*. YB * + A* . +* .

Example 1.1: Show that vector = 3 + 6P − 2 * = 4 − ̂ + 3 are mutually


and )
perpendicular.

Solution: If the angle between * is , then


and )

* =
.) ) cos ,

cos , =
*
.^ (_` ab5/#) (c`5b _#)
^
= =0
%( &d &
e
&)
f %(^d& ^e& ^f&

cos , = 0, , = 90g . Therefore the vectors are mutually perpendicular.

under the influence of a force h = (−3 ̂ + 4i + 4 ) N. Calculate the work done by the force.
Example 1.2: A particle moves from a point (3,-4,-2) meter to another point (5,-6, 2) meter

Solution: Suppose the particle moves from point A to B. Then displacement of particle is given
by

=O 4 Q)−O 4
** = j(5 − 3) + (−6 + 4)P + (2 + 2) l meter
= (2 − 2P + 4 ) meter.
Work done = h . = [(−3 ̂ + 4i + 4 ).( 2 − 2P + 4 )lN meter =2 joule.

1.3.2.2 Vector product or Cross Product

9
MSCPH501

The vector product or cross product of two vectors is a vector quantity and defined as a vector
whose magnitude is equal to the product of magnitudes of two vectors and sine of angle between

If and )* are two vectors then cross product of these two vectors is denoted by ×)* (read as
them.

* ) and given as
)
* = ) 4∅ 4o = p .
×)
* , and 4o is the unit vector perpendicular to both
Where ∅ is the angle between vectors and )
and***) Y . . 4 q r ℎ Or 4 4 4t 4N )* Z.
* is along y axis then vector product can be considered as an area
is along x axis and )
* and
Suppose
and )
* is positive
of parallelogram OPQR as shown is figure 1.7 in XY plane whose sides are
direction is perpendicular to plane OPQR i.e. along z axis. The cross product and )
if direction of ∅ ( * ) is positive or rotation is anticlockwise as show in figure 1.8, and
)
negative if the rotation of ∅ ( ** ) is clockwise (Figure 1.12).
u

p *
) R Q

O P Y

Figure 1.7 Vector product as area of parallelogram OPQR

Y Y

p= ×) ***
) ***
)

∅ ∅

O X O X

* ×
p=)

Figure1.8: Direction of vector product

10
MSCPH501

Important properties of vector product

* the vector products are defined as


(i) Commutative law does not hold: From the definition of vector product of two vectors
and )
* = AB sin∅ v4
×)
* ×
) = AB sin∅ (−4o) = −AB sin∅ v4 = − × ) *.
Since in case of )* × the angle of rotation becomes opposite to case * , hence product
×)

* ≠) * × .
becomes negative.
Therefore, × )
(ii) Distributive law holds:

*** × Y)
* + pZ = × ) * + × p. ***
In case of vector product the distribution law holds.

(iii) Product of equal vectors:

× = | || | 4∅ v4 = 0.
If two vectors are equal then the angle between them is zero, and vector product becomes

In case of Cartesian coordinate system if ̂, ̂, are unit vectors along x, y and z axes then
Hence the vector product of two equal vectors in always zero.

̂ × ̂ = ̂ × ̂ = × = 0.
(iv) Collinear vectors: Collinear vectors are vectors parallel to each other. The angles

* = | ||)| 4∅ v4 = 0.
between collinear vectors are always zero therefore
×)
Thus, two vectors are parallel or anti-parallel or collinear if its vector product is 0.

* are orthogonal to each


and )
other then angle between such vectors is ∅ = 90°, therefore
(v) Vector product of orthogonal vector : If two vectors

×)* = ) 4∅ v4
×)* = | ||)| v4
In Cartesian coordinate system if ̂, ̂, are unit vector along x, y and z axes then

W
ŷ × ẑ = { W = ŷ |}~ {
ẑ × { W × ŷ = ẑ
̂× ̂=− × ̂ = − ̂ 4N ̂ × = ̂.
(vi) * are two vectors given as
Determinant form of vector product: If and )

= = ̂+ < ̂+ >
* = )= ̂ + )< ̂ + )> .
)

* =Y
Then,
×) = ̂+ < ̂+ > Z × ()= ̂ + )< ̂ + )> )

11
MSCPH501

= = )= ̂× ̂+ = )< ̂× ̂+ = )> ̂× + < )= ̂× ̂+ < )< ̂× ̂+ < )> ̂× +


> )= × ̂+ > )< × ̂+ > )> ×
= = )< − = )> ̂− < )= + < )= ̂+ > )= ̂− > )< ̂
(Since ̂ × ̂ = ̂ × ̂= × = 0 4N ̂ × = − ,̂ ̂ × ̂ = − , × ̂ = − ̂)
= ̂( < )> − > )< ) − ̂ ( = )> − > )= ) + ( = )< − < )= )
̂ ̂
* =•
×) = < > •.
)= )< )>
Physical significance of vector product:
In physics, numbers of physical quantities are defined in terms of vector products. Some basic
examples are illustrated below.

***
(i) Torque: Torque or moment of force is define as
€ = × Q.

Where € is torque, is position vector of a point P where the force Q is applied. (Figure 1.9)

Figure 1.9

* with a velocity •
* at an angle with the direction of magnetic field then force
(ii) Lorentz force on a moving charge in magnetic field: if a charge q is moving in a
magnetic field )
h experienced by the charged particle is give as;

* ×)
h = ‚(• *)

This force is called Lorentz force and its direction is perpendicular to the direction of both
velocity and magnetic field B.

(iii) Angular Momentum: Angular momentum is define as the moment of the momentum
and given as:

* = × O.
ƒ

12
MSCPH501

is the radial vector of circular motion and O is the linear moment of the body under
* is angular momentum along the direction perpendicular to both and O.
Where
circular motion, and ƒ
The law of conservation of angular momentum is a significant property in all circular motions.

1.3.3. Product of three vectors:

If we consider three vectors , ) * 4N p , we can define two types of triple products known as
scalar triple product and vector triple product.

1.3.3.1 Scalar Triple product:

Let us consider three vectors , ) * and p then the scalar triple product of these three vectors is
* × p ) and denoted as „ ) * p …. This is a scalar quantity.
If we consider*** , )* and p the three sides of a parallelepiped as shown in Figure 1.14 then )* ×p
defined as .()

* × p is naturally along Z axis (perpendicular to both )


parallelogram. The direction of ) * and p ).
is a vector which represents the area of parallelogram OBDC which is the base of the

* × p ) and vector
If ∅ is the angle between the direction of vectors ()
* × p ) and vector is given as (Figure 1.10)
, then the dot product of
vectors ()

* × p) = | | )
. () * ×p * × p Z = ℎ. ()
∅ = p ∅Y) * × p)

= Vertical height of parallelogram × area of base of parallelogram

= Volume of parallelogram = j ) pl.

C D

O *
) B X

Figure 1.10: Volume of parallelogram represented by 3 vectors

13
MSCPH501

Therefore, it is clear that . () * × p ) represents the volume of parallelepiped constructed by


vectors*** , )
* and p as its sides. Further, it is a scalar quantity as volume is scalar. It can also be

* 4N p or dot and cross are interchanged.


noted that in case of scalar triple product the final product (volume of parallelepiped) remains
same if the position of , )

*** p … = . Y)
„ ) * × pZ = )
* . Yp × Z = p . Y × )
* Z = Y)
* × p Z. * =Y ×)
= Yp × Z. ) ***
* Z. p.

Scalar triple product can also be explained by determinant as

= < >
*** p … = . Y)
„ ) * × p Z = • )= )< )> •.
p= p< p>

In case of three vectors to be coplanar, it is not possible to construct a parallelepiped by using


such three vectors as its sides; therefore the scalar triple product must be zero.

* p … = . Y)
„ ) * × p Z = 0.

1.3.3.2 Vector triple product:


The vector triple product of three vectors is define as

× Y) * − Y .)
* × p Z = Y . p Z) * Zp .

The vector triple product is product of a vector with the product of two another vectors. The
vector triple product can be evaluated by determinant method as given below.

P
* × p ) = •)=
() )< )> •
p= p< p>

= Y)< p> − )> p< Z − P()= p> − )> p= ) + ()= p< − )< p= )

P
* × pZ = •
× Y) = < > •
)< p> − )> p< )> p= − )= p> )= p< − )< p=

* − ( .)
= Y . p Z) * )p .

As in cross product the vector × Y) * × p Z will be perpendicular to plane containing vectors


* × p Z. Since Y)
Y) * × p Z is itself in the direction perpendicular to plane containing )
* and
* × p Z will be along the plan containing ) * and p ,
and
p , therefore the direction of × Y)
ℎ 4 r4 combination of *** ***
) 4N p.

14
MSCPH501

1.4 Differentiation of vector:


Suppose is the position vector of a particle situated at point P with respect to origin O. If
particle moves with time, then vector varies corresponding to time t, and is said to be
vector function of scalar variable t and represented as =F(t) as shown in Figure 1.11

If P is the position of particle at time t then OP = .

If Q is the position of particle at time t+† and position vector of Q is ( + † )

as ***** ****** -‡A


AB = ‡B *****

= +† − .

In limiting case if † → 0 then † → 0 and P tends to Q and the chord become the tangent at
P. Differentiation is define as

N † ( +† )− ( )
= lim = lim .
N ‹Œ→g † ‹Œ→g †
When the limit exists only then the function is differentiable. If we further differentiate
function with respect to t and hence it is called second order differentiation. If should be
cleared that the derivatives of a vector (say ) are also vector quantities.

+† †

O ***

Figure 1.11

Properties of vector differentiation:

If * are two vectors, ∅ is a scalar field and p is a constant vector then


and )

(1) * Z = • + •^.
Y +)
• *
•Œ •Œ •Œ

(2) ( × ∅) = ∅+ .
• • •∅
•Œ •Œ •Œ

(3) *Z= . +
Y .) *.
.)
• •^* •
•Œ •Œ •Œ

15
MSCPH501

(4) *Z=
Y ×) × *.
+ •Œ × )
• *
•^ •
•Œ •Œ

(5) = 0.
•Ž
•Œ

(6) =
•• •• ••
•Œ •• •Œ
where s is the scalar function of t.
(7) ( /) = ( . )= + =2 , where
• • •• •• ••
•Œ •Œ •Œ •Œ •Œ
is the position vector.

Example 1.3: A particle is moving along the curve x = / + 2 , y= / + 1 and z= 3 + 5.


Find the velocity and acceleration of particle along the direction 3i+2j+6k at time t=2.

Curve is define as x = / + 2 , y= / + 1 and z= 3 + 5.


Solution:

̅ = I + JP + K
The position vector of particle at any time t is given as

̅ = ( + 2 ) + ( / + 1)P + (3 + 5)
/

O N B
Figure 1.12

N ̅
Velocity is given as

=3 /
+2 P+3
N

N ̅
at t=2 velocity becomes

= 12 + 4P + 3 .
N
* (
Component of the velocity along the direction 3 + 2P + 6 = ) J)

‡S = | ̅ | cos , . “ = | ̅ |
”•.^• ^ (”•.^•)^
|”•||^•| |^•| |^|&
. =

(3 + 2P + 6 ) = c— (3 + 2P + 6 )
(6a ` cb _#).(_` /b a#) –c
_& /& a&
=

acceleration • can be given as • = =6 + 2P


••̅
•Œ
acceleration • at t=2 can be given as • = 12 + 2P.

Component of acceleration along direction )• is given as

16
MSCPH501

= | •| cos , . “ = | •| |˜•| |^| |^| =


™ ^•
˜. ^• ™ ^•) ^•
(˜.
|^|&

(3 + 2P + 6 )
(6/` /b).(_` /b a#)
_/& /& a&
=
= c— (3 + 2P + 6 ).
š/

1.4.1 Partial derivative:

If f is a vector function which depends on variable (x, y, z), then the partial derivatives are
defined as

›Q Q(I + †I, J, K) − Q(I, J, K)


= lim
›I ‹=→g †I
›Q Q(I, J + †J, K) − Q(I, J, K)
= lim
›J ‹<→g †J

›Q Q(I, J, K + †K) − Q(I, J, K)


= lim .
›K ‹>→g †K
In case of partial derivatives with respect to a variable, all the other remaining variables are taken
as constant.

Partial derivatives of second order are defined as:

= œ= (œ= )
œ& • œ œ•
œ= &

= œ< (œ<)
œ& • œ œ•
œ< &

=œ> žœ> Ÿ.
œ& • œ œ•
œ> &

1.4.2 Del operator:

The vector differential operator del is denoted by and defined as

› › ›
=i +P + .
›I ›J ›K

1.4.3 Scalar and vector point functions:

(1) Field: Field is a region of the space defined by a function.

(ii) Scalar point function: A scalar function ∅(I, J, K) defines all scalar point in the space. For
example, gravitational potential is a scalar function defined at all gravitational fields in the space.

17
MSCPH501

(iii) Vector potential function: If a vector function h (x, y, z) defines a vector at every point in
space then it is called vector point function. For example gravitational force is a vector function
defined at a gravitational field in the space.

1.4.4 Gradient:

The gradient of a scalar function ∅ is defined as

grad ∅ = ∇∅ = (i +P + )∅
œ œ œ
œ= œ< œ>

= i +P +
œ∅ œ∅ œ∅
œ= œ< œ>

grad ∅ ‚¢4 J.

Total differential d∅ of a scalar function ∅(I, J, K) can also be expressed as,

›∅ ›∅ ›∅
N∅ = NI + NJ + NK.
›I ›J ›K

Total differential d∅ of a scalar function ∅ can be expressed as

d∅ = NI + œ< NJ + œ> NK
œ∅ œ∅ œ∅
œ=

+ P œ< + ) ( NI + PNJ + NK)


œ∅ œ∅ œ∅
œ= œ>
=(

* ∅Z. N = |∇∅||dr|cosθ = (∇
N∅ = Y∇ * ∅). dr ̂ , (where ̂ is a unit vector along d )

also , is angle between *∇∅ and d (The direction of displacement).


•∅
* ∅). ̂ .
So, •• = (∇

This, •• is the directional derevative of ∅. The rate of change is maximum if ̂


•∅

is along *∇∅ and ̂ is zero.

Hence gradient of the sector field ∅ defines a vector field the magnitude of which is equal to the
maximum rate of change of ∅ and the direction of which is the same, as the direction of
displacement along with the rate of change is maximum.

Example 1.4: In the heat transfer, the temperature of any point in space is given by
T=xy+yx+zx. Find the gradient of T in the direction of vector 4i-3k at a point (2, 2, 2).

Solution:

18
MSCPH501

Temperature is define as

T=xy+yx+zx

gradient of temperature T is given as

› › ›
t N¥ = ∇¥ = ¦ + P + ›§ (IJ + JK + KI).
›I ›J ›K

∇ ¥ = (J + K) + P(I + K) + (I + J)

at point (2, 2, 2) the ∇ ¥ is (4 + 4P + 4 ).

The gradient T in the direction of vector 4i-3k is

= (4 + 4P + 4 ). Unit vector along (4 − 3 )

=(4 + 4P + 4 ). √c&
(c`5_#)
_&

=4/5.

1.4.5 Physical significance of grad ∅ :

The physical significance of grad ∅ can be explained on the basis of surface defined by scalar
field ∅. The value of ∅ remains constant on the surface S, as shown in Figure 1.13 and it is called

function ∅ and ∅+d∅ respectively. Suppose 4* is normal to the surfaces S and S'. If the
a level surface or equi-scalar surface. Let us consider two surfaces S and S' defined by scalar

coordinates of point P and Q are (x, y, z) and (x+dx, y+dy, z+dz) then the distance between P
and Q are

N = NI + PNJ + NK

as the definition of differentiation

›∅ ›∅ ›∅
N∅ = NI + NJ + NK
›I ›J ›K

›∅ ›∅ ›∅
=¦ + J+ § . (NI + NJ P + NK )
›I ›J ›K

N∅ = *∇∅. N .

If we consider the point Q approaches to P and finally lies on P then

N∅ = 0

19
MSCPH501

*∇∅. N = 0.

Where ∇∅ 4N N O O 4N ¢r ℎ ℎ .

Therefore, ∇∅ is a vector which is perpendicular to the surface S.

If 4* is normal on the surface S and d4* represents the distance between surfaces S to S' then
N4 = N , = 4o. N

and N∅ = œ© N4 = œ© 4o. N .
œ∅ œ∅

* ∅. N = œ∅ 4.
By using equation (1), ∇ vN
ϩ
›∅
*∅=
∇ 4o .
›4
Thus, ∇∅ is defined as a vector whose magnitude is rate of change of ∅ along normal to the
surface and direction is along the normal to the surface.

Example1.5: Find the directional derivative of a scalar function ∅(I, J, K) = I / + xy + z / at


the point A (2,-1,-1) in the direction of the line AB where coordinate of B are (3, 2, 1).
Solution:
The component of ∇∅ along the direction of a vector is called directional derivative of ∅ and
given as ∇∅ .
Now ∇∅ = ž + P œ< + Ÿ (I / + IJ + K / )
œ œ œ
œ= œ>
=(2I + J) + I P + 2K
gradient at point A (2,-1,-1)

20
MSCPH501

∇∅ = 3 + 2 P − 2 .
The vector *****) = O 4 Q ) –O 4 Q
= (3 + 2P + ) − (2 − P − ) = + 3P.
Directional derivative of ∅ in the direction of AB is
( + 3P) 9
*∇∅ . ª) = (3 + 2P − 2 ). = .
√1 + 9 √10

The divergence is defined as dot product of del operator with any vector point function ***
Q or any
1.4.6 Divergence of Vector:


vector h and given as,
div. Q = ∇. Q = ž + P œ< + Ÿ. ( Q= + PQ< Q> ) where Q = Q= + PQ< Q>
œ œ œ
œ= œ>

= + + .
ϥd ϥe ϥf
œ= œ< œ>

Since divergence of a vector Q is dot product of del operator *∇ and that vector Q , therefore it is a
scalar quantity.
1.4.6.1 Physical Significance of Divergence:
On the basis of fluid dynamics or a fluid flow, the divergence of a vector quantity can be
explained. Let us consider a parallelepiped of edges dx, dy and dz along the x, y, z directions as
shown in figure 1.14.

R R'

Q Q'

A dy B

O dx O' X

dz

P P'

Figure 1.14: Physical Significance of Divergence

Let is the velocity of fluid at A(x, y, z) and given as

= = + <P + > .

21
MSCPH501

Where the =, < , > ℎ qO 4 4 Q r J r 4t I, J, K N 4 .

Amount of fluid entering through the surface O'P'Q'R' per unit time is given as:

r J× = = NJNK.

Amount of fluid flowing out through the surface O'P'Q'R' per unit times is given as

= = •= NJNK

+ NI)NJNK.
œ”d
=( = œ=

Decrease in the amount of fluid in the parallelepiped along x axis per unit time.

= = NJNK − ž + NIŸ NJNK


œ”d
= œ=

=− NINJNK.
œ”d
œ=

Negative sign shows, decrease in the amount of fluid inside the parallelepiped.

Similarly decrease of amount of fluid along y axis

NINJNK.
œ”e
œ<
=−

Decrease of amount of fluid along z axis

=− NINJNK.
œ”f
œ>

Total amount of fluid decrease inside the parallelepiped per unit time= − ž œ=d + +
œ” œ”e
œ<

Ÿ NINJNK).
œ”f
œ>

Thus, the rate of loss of fluid per unit volume = + +


œ”d œ”e œ”f
œ= œ< œ>

(We can ignore negative sign when we specify that the negative sign indicates decrease in the
amount of fluid).

Further the rate of loss of fluid per unit volume

+ P œ< + Ÿ( + <P + ) = *∇. =N


œ œ œ
=ž œ= œ> = >

Thus, the divergence of velocity vector shows the rate of loss of fluid per unit timer per unit
volume.

22
MSCPH501

therefore N = 0.
If we consider fluid is incompressible, there is not any loss or gain in the amount of fluid,

If the divergence of a vector is 0, then the vector function is called solenoidal.

Example 1.6: If u=x2+y2+z2 and ̅ = 2xi + 3yj + 2zk, then find the div (u ).

Solution : Div (u ) = ∇. (u )

› › ›
¦ +P + § . j(I / + J / + K / )(2I + 3JP + 2K )l
›I ›J ›K

› / › › /
= (I 2I) + P (J / 3J)P + (K . 2K)
›I ›J ›K

= 6I / + 9J / + 6K / .

1.4.7 Curl

The curl of a vector h is defined as

Curl h = ∇ × h (where h• =h= + h< P + hb )

+ P œ< + ) × Yh= + h< P + hb Z.


œ œ œ
œ= œ>
=(

In terms of determinant of vector product

P
Curl h• =®œ= ®.
œ œ œ
œ< œ>
h= h< h>

Since curl is vector product of two vectors, therefore it is a vector quantity.

1.4.7.1 Physical significance of curl:

On the basis of angular velocity and linear velocity the curl can be explained.

around origin O. Let ¯


* is the angular velocity of particle then
Let us consider a particle moving with velocity and is the position vector of particle rotating

¢ r =∇×

= ∇ × (¯
™× ) (∵ =¯
™ × )

= ∇Y¯= + ¯< P + ¯> Z × (I + JP + K )

23
MSCPH501

P
= ∇ × ±¯= ¯< ¯> ±
I J K

= ∇ × „Y¯< K − ¯> JZ − (¯= K − ¯> I)P + (¯= J − ¯< I) …

P
= ² ³
œ œ œ
œ= œ< œ>
¯< K − ¯> J ¯> I − ¯= K ¯= J − ¯< I

¢ r = 2Y¯= + ¯< P + ¯> Z = 2¯


™.

Thus, the curl of linear velocity shows angular velocity which means rotation of particle. i.e.
Curl of a vector quantity is connected with rotational properties of vector field. If curl of a vector
is zero, ∇ × Q = 0, there is no rotational property and Q is called irrotational.

Example 1.7: Calculate the curl of a vector given by h = IJK + 2I / JP + (I / K / − 2J / )k.

Solution:

¢ rh = ∇×h

=ž + P œ< + Ÿ × (IJK + 2I / JP + (I / K / − J / ) )
œ œ œ
œ= œ>

P
=² ³
œ œ œ
œ= œ< œ>
IJK 2I J
/
I / K / − 2J /

= −4J − (2IK / − IJ)P + (4IJ − IK) .

Example 1.8:

Show that h = (J / + 2IK / ) + (2IJ − K)P + (2I / K − J + 2K) * r 4 r.

Solution:

¢ rh = ∇×h

=ž + P œ< + Ÿ × j(J / + 2IK / ) + (2IJ − K)P + (2I / K − J + 2K) * ]


œ œ œ
œ= œ>

= 0.

Therefore, h is irrotational.

24
MSCPH501

1.5 Vector integral:


1.5.1 Line Integral: The integral of a vector function F along a line or curve is called line
integral.

Suppose h (x, y, z) be a vector function and PQ is a curve and ***Nr is a small length of curve as
shown in figure 1.15 then line integral of vector h along a length ***
Nr is given as

*
´µ h . Nr.

Q Nr

O x

Figure 1.15: Line Integral

The integral may be closed or open depending on the nature of the curve whether closed or open.
To compute the line integral of a function F, any method of integral calculus may be employed.
In case of fore h acting on a particle along a curve PQ, the total work done can be calculated as
line integral of force.

*** .
Work done= ´· h . Nr

1.5.2 Surface integral:

Similarly as line integral of F is a vector function and s is a surface, then surface integral of a
vector function F over the surface s is given as

*** .
Surface integral=∬• h . Nr

The direction of surface integral is taken as perpendicular to the surface s.

If ds is written as ds = dxdy.

25
MSCPH501

**** = ´ ´ h. NINJ .
Surface integral=∬• h . N = <

Surface integral represents flux through the surface S.

1.5.3 Volume integral:

If dV denotes the volume defined by dxdydz then the volume integration of a vector F is define
as

Volume integral=´¹ hN• = ´= ´< ´> h. NINJNK.

The volume integral can be explained in terms of total charge inside a volume. Suppose ρ is
charge density of a volume dV then total charge inside the volume is given as q=´” ρ N•.

1.6 Vector identities:

If ∅1 and ∅2 are two scalar point functions and * are two vectors, then
and )

∇(∅6 + ∅/ ) = ∇∅6 + ∇∅/

∇(∅6 ∅/ ) = ∅6 ∇∅/ + ∅/ ∇∅6

N *Z=N
Y +) +N *
)

N * Z = *** . N
Y .) *** N
) + ).

*Z= ¢ r
¢ rY +) *
+ ¢ r)

N Y∅ Z = ∅ N + .t N∅

¢ r Y∅ Z = ∅ ¢ r +t N∅ ×

N ¢ r =0

¢ rt N ∅=0

N *Z=)
Y ×) *. ¢ r *
+ . ¢ r)

¢ r ¢ r =t NN − ∇/ .

Example 1.9 : Prove that

(1) N ¢ r =0
(2) ¢ r t N ∅ = 0.

26
MSCPH501

Solution:
(1) (1) N ¢ r = ∇.∇ ×
P
= ∇. ® œ= ®
œ œ œ
œ< œ>
= < >

= ∇. º ž − Ÿ+ P( − ) + k( − )»
œ f œ e œ d œ f œ e œ d
œ< œ> œ> œ= œ= œ<

ž − Ÿ + − )+ ( − )
œ œ f œ e œ œ d œ f œ œ e œ d
œ= œ< œ> œ< œ> œ= œ> œ= œ<
= (

= 0.

(2) ¢ r t N ∅ = ∇ × ∇∅
P
› › ›
® ®
›I ›J ›K
®›∅ ›∅ ›∅®
›I ›J ›K
› ∅
/
› ∅
/
›/ ∅ › /∅ › /∅ › /∅
= ¼ − ½+P¼ − ½+ ¼ − ½ = 0.
›J›K ›K›J ›K›I ›I›K ›I›J ›J›I

Example 1.10:

Show that

(i) N *Z=)
Y ×) *. ¢ r − *
. ¢ r)
(ii) ¢ r ¢ r =t NN − ∇/ .

Solution (i) N * Z = ∇. ( × )
Y ×) *)

= (o + ̂ œ< + ). [(AyBz – AzBy) ̂ + (AzBx - AxBz) ̂ + (Ax By – AyBx) ]


œ œ œ
œ= œ>

œ œ œ
œ=
= (AyBz – AzBy) + œ< (AzBx – AxBz) + œ> (AxBy – AyBx)

œ > œ < œ = œ > œ < œ = œ^> œ^< œ^= œ^>


= Bx( œ< -
œ>
) + By( œ> -
œ=
) + Bz( œ= -
œ<
) – Ax( œ< -
œ>
) – Ay( œ> -
œ=
)

œ^< œ^=
– Az( œ= -
œ<
)

= (Bx ̂ + By ̂ + Bz ). [( œ< - ) ̂ + ( œ> - ) ̂ + ( œ= -


œ > œ < œ = œ > œ < œ =
œ> œ= œ<
) ]–

27
MSCPH501

(Ax ̂ + Ay ̂ + Az ). [( œ< - ) ̂ + ( œ> - ) ̂ + ( œ= -


œ^> œ^< œ^= œ^> œ^< œ^=
œ> œ= œ<
) ]

*** curl
= ). - *** . curl )
*

= curl * - curl )
.) * . .

Solution (ii)

¢ r ¢ r ̅ = ∇ × (∇ × ̅ )

P
=( +P + ) × ® œ= ®
œ œ œ œ œ œ
œ= œ< œ> œ< œ>
= < >

=( +P + ) × º ž œ<f − Ÿ − P( œ>d − ) + k( œ= − )»
œ œ œ œ œ e œ œ f œ e œ d
œ= œ< œ> œ> œ= œ<

P
œ œ œ
= ®® œ= œ< œ> ®®
ž œ< − Ÿ ( œ>d − ) ( œ= − )
œ e œ e œ œ f œ e œ d
œ> œ= œ<

= º ž − Ÿ − ( − )» +Pº ž − Ÿ− ( − d )»
œ œ e œ d œ œ d œ f œ œ f œ e œ œ e œ
œ< œ= œ< œ> œ> œ= œ> œ< œ> œ= œ= œ<

+ ºœ= ž − Ÿ− ( − )»
œ œ d œ f œ œ f œ e
œ> œ= œ< œ< œ>

= ºœ<œ= − − + œ>œ=f » + P ºœ>œ<f − − + œ=œ<d »


œ& e œ& d œ& d œ& œ& œ& e œ& e œ&
œ< & œ> & œ> & œ= &

+ ºœ=œ>d − − + œ<œ>»
œ& œ& f œ& f œ& e
œ= & œ< &

= ∑ ºž œ= &d + œ<œ= + œ>œ=f Ÿ − ( + + )»


œ& œ& e œ& œ& d œ& d œ& d
œ= & œ< & œ> &

=∑ ž œ=d + + œ>f Ÿ − ∑ ºž &d + + Ÿ»


œ œ œ e œ œ& œ& d œ& d
œ= œ< œ= œ< & œ> &
=t NN ̅−∇ / ̅
.

1.7 Gauss divergence theorem:


Gauss divergence theorem is a relation between surface integration and volume integration. The
theorem states:
The surface integral of a vector filed h over a closed surface s is equal to the volume integral of
divergence of h taken over the volume enclosed by surface s.

28
MSCPH501

Mathematically ∬• h . N = ∭” N hN .

Let us consider a vector h = h6 ̂ + h/ ̂ + h_ W.


Mathematical proof:

According to Gauss divergence theorem ∬• (h6 ̂ + h/ ̂ + h_ ) . N = ∭” ž ̂ œ= + ̂ œ< +


œ œ

Ÿ . (h6 ̂ + h/ ̂ + h_ )NINJNK
œ
œ>

Or ∬• (h6 ̂ + h/ ̂ + h_ ) N = ∭” ž + + Ÿ NINJNK
œÀÁ œÀ& œÀÂ
œ= œ< œ>
(1)
Y

Ã6 Ã/

Q6 (J, K) Q/ (J, K)

Figure 1.16: vector filed *Ä over a closed surface and corresponding enclosed volume

Now we can prove equation (1)

Let us first evaluate ∭” NINJNK = ∬• º´=Å• &(<,>) NI» NJNK


œÀÁ =Å• (<,>) œÀÁ
œ= Á œ=

= ∬•jh6 (I, J, K)l=Å•& (<,>) dydz


dÆÇÁ (e,f)

= ∬Èjh6 (Q/ , J, K) − h6 (Q6 , J, K)lNINJ (2)

Now, the right portion of surface i.e. Ã/ can be given as

NJNK = 4o/ . N / where 4o/ is the direction of unit vector perpendicular to the surface.

Similarly the left portion of surface Ã6 can be given as

NJNK = 4o6 . N 6 .

Putting the value of area in the factors of RHS of equation (2) we have

É Êh6 (Q/ , J, K)NJNK = + É h6 4o/ . N / Ë


• •&

29
MSCPH501

É Êh6 (Q6 , J, K)NJNK = − É h6 4o6 . N 6 Ë.


• ÌÁ

Since the outward flux at surface Ã/ is in the direction along the x axis and flux at surface Ã6 is
along the negative direction of x axis. Therefore, Ã6 component is negative.

Putting the above values in equation (2) we have

›h6
Í NINJNK = É h6 4o / . N + É h6 4o 6 . N
” ›I •&
/
•Á
/

›h6
Í N = É h6 4o. N .
” ›I Ì

Since 4o 6 and 4o / are the direction perpendicular to yz plane that is along x axis shown by 4o.

Similarly it can be shown that

›h/
Í N = É h/ 4o. P N
” ›J •

›h_
and

Í N = É h_ 4o. N .
” ›K •

Adding all above terms

›h6 ›h/ ›h_


ͦ + + § N = É (h6 4o. + h/ 4o. P + h_ 4o. ) N
” ›I ›J ›K Ì

Or **** .
∭” (∇. h)N = ∬• h . N

**** ) to the behavior of vector field (∇


* . h ) inside the volume.
This is Gauss divergence theorem. The theorem relates the flux of a vector filed through a
surface (h .N

1.7.1 Deduction of Gauss law with Gauss Divergence theorem:

In electrostatics the Gauss law is one of the fundamental law and frequently used. This law is a
result of Gauss theorem in electric field.
6
∈Ï
Statement: The total electric flux through a closed surface is equal to times total charge
enclosed inside the surface.

30
MSCPH501

**** = 6 ( total charge inside the surface)


Mathematically: ∬• Ð* . N ∈ Ï

**** = 6 ∑Ø q Ø.
∬• Ð* . N
Ï∈

Proof: Let us consider a charge q is situated at O, the origin of Cartesian coordinate system.
Consider an imaginary surface called Gaussian surface around the charge q. The Gaussian
surface may be of any shape but closed.

(radial) of this surface is r from the origin and it subtends a solid angle N¯ at the centre.
Consider a small surface ds on the Gaussian surface as shown is Figure 1.17. The distance

The electric flux through this small surface ds is

N∅ = Ð* . N .

The total electric flux through the whole surface

*** . N .
∅=É Ð

Now the electric field on the surface ds is given by

1 ‚
Ð= ̂. N
4Ù ∈g /

31
MSCPH501

where ̂ is unit vector along the direction of . The flux ∅

1 ‚
∅=É ̂. N
• 4Ù ∈g /

1 v. 4o N
= É ‚.
4Ù ∈g • /

Úℎ 4o ¢4 O O 4N ¢ r ¢ Q N .

1 ‚N ,
∅= É .
4Ù ∈g • /

Where ̂ . 4o = ,. Now is solid angle subtended by surface ds and denoted by N¯.


•• ÛU•Ü
•&

1 1 ‚
∅= É ‚. N¯ = . ‚. 4Ù = .
4Ù ∈g • 4Ù ∈g ∈g

Since total angle subtended by whole surface S at the centre is 4Ù.

Hence ∬• Ð. N = ∑` ‚` .
6
∈Ý

density Þ then the statement can be given as


In case the charge in the closed surface is distributed in the volume V with volume charge

1
∅ = É Ð. N = Í Þ N• .
• ∈g ”

1.7.2 Gauss law in differential form:

Gauss law in electrostatics is given as

1
É Ð* . N = ( r ℎ t 4 N ¢ Q ).
• ∈g

If Þ is volume charge density inside the volume and is enclosed by surface s then,

1
É Ð* . N = Í ÞN•
• ∈g ”

É Ð* . N = Í N Ð* N .
• ”

Applying Gauss divergence theorem

32
MSCPH501

1
ÍN Ð* N• = Í ÞN•
” ∈g ”

Þ
Í ¦N Ð* − § N• = 0
” ∈g
Þ
N Ð* − =0
∈g
Þ
N Ð* = .
∈g

This is called differential equation of Gauss law.

1.7.3: Poisson’s equation and Laplace equation:

If we consider Ð* as electric field and ∅ as electric potential then the electric field can be given as

Ð* = −∇∅ .

Now using differential form of Gauss law


Þ
N (−∇∅) =
∈g
Þ
∇(−∇∅) =
∈g
Þ
∇/ ∅ = − .
∈g

equation and operator ∇/ is an operator defined as


This is called Poisson’s equation. Poisson’s equation is basically second order differential

›/ ›/ ›/
∇ = ∇. ∇= / + / + / .
/
›I ›J ›K

This is called Laplacian operator.

If there is no charge inside the volume i.e. Þ=0, then above equation becomes

∇/ ∅ = 0.

This is called Laplace equation.

∬• . d .
Example 1.11: If is position vector of any point on the surface s whose volume is V, find

33
MSCPH501

Solution:

É **** = Í N
.N N•
• ”

= ∭” ž +P + Ÿ . ( I + PJ + K) N•
œ œ œ
œ= œ< œ>

= ∭” ž + + Ÿ N•
œ= œ< œ>
œ= œ< œ>

= ∭” 3 N• = 3• .

Example 1.12:

Using Gauss divergence theorem find out ∬• . N where, = I _ + J _ P + K _ and s is a


surface of a sphere defined by I / + J / + K / = / .

Solution:

∬• . N = ∭” ∇. N•

= ∭” ž + P œ< + Ÿ . (I _ + J _ P + K _ ) N•
œ œ œ
œ= œ>

= ∭”(3I / + 3J / + 3K / ) N•

= 3 ∭”(I / + J / + K / ) N•

= 3 ∭” /
N• = = 3 /
∭” N•

=3 ž_ Ù _ Ÿ
/ c
=ž_ Ù Ÿ.
6/ š

1.8 Green’s Theorem for a Plane:

Statement: If ∅6 (I, J) and ∅/ (I, J) are two scalar functions which are continuous and have
4N
œ∅Á œ∅&
œ< œ=
continuous derivatives over a region R bounded by simple closed curve c in x-y
plane, as

∮Û(∅6 NI + ∅/ NJ) = ∬È ( œ=& − )NINJ


œ∅ œ∅Á
œ<

curve 6(ABC) and / (pà ) as shown in Figure 1.18.


Proof: Let us consider a close path ABCD denoted by curve c, and curve c divided into two parts

34
MSCPH501

The equation of curve 6 is

J = J6 (I).

The equation of curve / is

J = J/ (I).

First we calculate the value of

∬È NINJ = ´=Ř º´<Å< & NJ» NI


œ∅Á =ÅÛ <Å< (=) œ∅Á
œ< Á(d) œ<

= ´˜ j∅6 (I, J)l<&Á(d) NI


Û < (=)

= ´˜ j∅6 (I, J/ ) − ∅6 (I, J6 )l NI


Û

= − ´Û ∅6 (I, J/ ) NI − ´˜ ∅6 (I, J6 )NI


˜ Û

= −„´Û ∅6 (I, J/ )NI + ´˜ ∅6 (I, J6 )NI…


˜ Û

= − º´Û ∅6 (I, J)NI + ´Û ∅6 (I, J)NI»


& Á

= − ∮Û ∅6 (I, J)NI.

Thus ∮Û ∅6 (I, J)NI = − ∬È NINJ.


œ∅Á
œ<

35
MSCPH501

Similarly it can be proved that

∮Û ∅/ (I, J)NJ = + ∬È NINJ.


œ∅&
œ=

Adding above two equations

›∅/ ›∅6
á (∅6 (I, J)NI + ∅/ (I, J)NJ) = É ¦ − § NINJ
Û ›I ›J
È

›∅/ ›∅6
á (∅6 NI + ∅/ NJ) = É ¦ − § NINJ.
Û ›I ›J
È

This is Green’s theorem for a plane.

Example 1.13: A vector field h is given by h = 4J + I(1 + J)P . Evaluate the line
integral ´Û h . N ** , where c is the circular path given by I / + J / = /
.

Solution: The vector field F is given as

h= 4J + I(1 + J)P.

Taking line integral along the curve c

â h. N = â j 4 J + I(1 + J)Pl. ( NI + PNJ)


Û Û

= â ( 4JNI + I(1 + J)NJ)


Û

ℎ = I + PJ N = NI + PNJ.

Now take 4 J = ∅6 and I(1 + J) = ∅/ .

On applying Green’s theorem

›∅/ ›∅6
â (∅6 NI + ∅/ NJ) = É ( − )NINJ
Û È ›I ›J

›(I(1 + J)) › 4J
=ÉÊ − Ë NINJ
È ›I ›J

= É j(1 + J) − JlNINJ
È

36
MSCPH501

= É NINJ = Ù /
.
È

Since R is the region of circular path along xy plane given by I / + J / = /


. Therefore the
radius of circular path is a.

Example 1.14: Applying Green’s theorem evaluate

â j(I / + 3IJ)NI + (I / + J / )NJl .


Û

Where c is a curve which form a square between the line J = ±1 4N I = ±1.

Solution: Given integral is

´Ûj(I / + 3IJ)NI + (I / + J / )NJl.

Applying Green’s theorem

∮Û(∅6 NI + ∅/ NJ) = ∬ ž œ=& − Ÿ NINJ


œ∅ œ∅Á
œ<

= ´56 ´56 º − œ< (I / + 3IJ)» NINJ


6 6 œ(= & < & ) œ
œ=

= ´56 ´56j2I − 3I)lNINJ


6 6

= − ´56 ´56 ININJ = − ´56 INI ´56 NJ


6 6 6 6

6
= −º » jJl656 = − (1/ − 1/ )(1 + 1) = 0.
=& 6
/ 56 /

1.9 Stoke’s Theorem:

Stoke’s theorem transforms the surface integral of the curl of a vector into line integral of that
vector over the boundary C of that surface.

Statement: The surface integral of the curl of a vector taken over the surface s bounded by a
curve c is equal to the line integral of the vector A along the closed curve c.

Mathematically:

É p¢ r . N = á . N *** .
• Û

37
MSCPH501

Since the curl A of a vector or vector function is along the normal to the surface, therefore the
above statement may also be represented as

É ¢ r . 4o N = á . N ** .
• Û

Where 4o is a unit vector perpendicular to the surface ds. Unit vector 4o can be given as

4o = cos - + äP+ å .

Proof: Let us consider a vector function A given as

= = + <P + >

and = I + JP + K

N = NI + PNJ + NK.

**** = ∬ ¢ r . 4o N
Using the Stoke’s theorem ´Û . N •

´ÛY = + <P + > Z . ( NI + PNJ + NK)

› › ›
= É æ¦ +P + §×Y + <P + Zç . ( -+P ä+ å)N
• ›I ›J ›K = >

or
´ÛY = NI + < NJ + > NKZ = ∬• ºž œ<f − Ÿ + ž œ>d − Ÿ P + ž œ= − Ÿ ».( -+
œ œ e œ œ f œ e œ d
œ> œ= œ<
P ä+ å) N

´ÛY = NI + < NJ + > NKZ = ∬• ºž œ<f − Ÿ - + ž œ>d − Ÿ ä + ž œ= −


œ œ e œ œ f œ e
œ> œ=
or

Ÿ å)» N
œ d
œ<

or
´ÛY = NI + < NJ + > NKZ = ∬• ºž œ>d ä− åŸ + ž− -+ åŸ +
œ œ d œ e œ e
œ< œ> œ=

ž œ<f -− 埻 N .
œ œ f
œ=
(1)

Let us first prove the first term

´Û = NI = ∬• ž œ>d ä− åŸ N .
œ œ d
œ<
(2)

Consider the = is function of (I, J, K) as = (I, J, K) and K = t(I, J) describes an equation of


surface s, and ds is a small elementary part of this surface as shown in Figure 1.19.

38
MSCPH501

´Ž = (I, J, K)NI = ´Û = (I, J, t(I, J))NI

= ´Ûj + NJ]
œ d (=,<,è(=,<))
= œ<
dx.

By using Green’s theorem

´Ž = (I, J, K)NI = ∬•( + ) NINJ.


œ d œ d œè
œ< œ> œ<
(3)

The direction cosines of the normal to the surface s are given as

p - p ä p å
= =
›t ›t 1
− −
›I ›J

If the projection of ds on x-y plane is N å

Then NINJ = N å or N =
•=•<
ÛU•é
.

Putting this value on equation (2)

39
MSCPH501

› = › = › = › = p å NINJ
ɦ p ä− å§ N = É ( p ä− )
• ›K ›J • ›K ›J å

› =p ä › =
=ɦ − § NINJ
• ›K p å ›J

› = ›t › =
=Éæ ¦− § − ç NINJ
• ›K ›J ›J

= −∬j + . lNINJ.
œ d œ d œè
œ< œ> œ<

Putting the value of R.H.S. from equation (3)

∬•( œ>d p ä − å) N = ´Ž = (I, J, K)NI


œ œ d
œ<

or ´Ž = (I, J, K)NI = ∬•( œ>d p ä − å) N .


œ œ d
œ<
(4)

Similarly

´Û < NJ = ∬•( œ= p å − p -) N
œ e œ e
œ>
(5)

and ´Û > NK = ∬•( œ<f p å − p ä)N .


œ œ f
œ=
(6)

On adding above equations (4), (5) and (6)

´ÛY = NI + < NJ + > NIZ = ∬• º œ> p ä − p å+ å− p -+ cos −


œ = œ d œ e œ e œ f
œ< œ= œ> œ<

ä» N
œ f
œ=

**** = ∬ p¢ r . N
Or ´Û *** . N **** .

Hence Stoke’s theorem is proved.

Example 1.15: Using Stoke’s theorem evaluate

´Ûj(2I − J)NI − JK / NJ − J / KNKl Úℎ ℎ r I / + J / = 1, corresponding to the


surface of a sphere of radius 1.

Solution:

The given integral

â j(2I − J)NI − JK / NJ − J / KNKl


Û

40
MSCPH501

= â j(2I − J) − JK / P − J / K l. ( NI + PNJ + NK)


Û

**** .
= ´Û . N

Where = (2I − J) − JK / P − J / K and ****


N = NI + PNJ + NK

**** = ∬ ∇ × *** . N
Using stokes theorem ´Û . N ****

= ∬• ∇ × A. no ds (1)

where no = unit vector perpendicualr to surface ds

P
∇× =® ®
œ œ œ
œ= œ< œ>
2I − J −Jí / −J K
/

= i(-2zy+2yz)-j(0-0)+k(0+1)

= k.

Putting this value in equation (1)

´Û . N = ∬• . 4oN .

Since ds is area of a circle described by I / + J / = 1 along xy plane, therefore direction of ds is


along perpendicular to surface which is along z axis.

Thus, ∬• . 4oN = ∬• N = ∬ NINJ = Ù.

Example 1.16: Verify Stoke’s theorem for vector filed given by h = (3I − 2J) + I / KP +
J / (K + 1) for a plane rectangular area with corners at (0,0),(1,0) (1,2) and (0,2) in x-y plane.

Solution: the given function is

h = (3I − 2J) + I / K P + J / (K + 1) .

Since the vector field is applying in an area which is described in x-y plane only, therefore z=0
and function becomes

h = (3I − 2J) + J / . (1)

According to Stoke’s theorem

41
MSCPH501

**** = ∬ ∇ × h . N
´Û h . N **** .

(2)

The line integral along the close path described by rectangle OADC as shown in figure 3.5 and
can be given as

**** = ´ h . N
´Û h . N **** + ´ h . N
**** + ´ h . N
**** + ´ ***
h. ****
N .
ÁÛ Û & Û Â Û î

Where p6 , p/ , p_ , pc are components of curve C.

´Ž h. N = ´g j(3I − 2J) + J / l. NI + ´g j(3I − 2J) + J / l. PNJ


6 /

+ ´6 j(3I − 2J) + J / l. NI + ´/ j(3I − 2J) + J / l. PNJ


g g

= ´g 3INI + ´g 0 NJ + ´6 (3I − 4)NI + ´/ 0. NJ


6 / g g

=/+0+/+0= 4
_ š

C(0,2) D(1,2)

p_

pc p/

p6

O (0,0) A(1,0) X

Figure 1.20

The L.H.S of equation (2) become 4 for given field. Now we calculate the R.H.S of equation (2 )

P
› › ›®
∇ × h = ®® ®
›I ›J ›K
3I − 2J 0 J/

= (2J + 0 + 2 ) = 2J + 2 .

Now ∬• ∇ × h. N = ∬•(2J + 2 ). 4o NINJ

= ∬• 2NINJ = 2 ∬• NINJ

42
MSCPH501

=2. Area of rectangle =2.2 = 4.

On comparing equation (3) and (4) the Stoke’s theorem has been verified.

1.10 CURVILINEAR COORDINATES:


We are well familiar about Cartesian coordinate system in which any point or vector can be defined with
the help of origin and three mutually perpendicular axes. Some time we need to use some another type of
coordinate system which is more convenient for describing a system or solving an equation. In this unit
we will study how the component of a vector can be formulated in another coordinate system called
curvilinear coordinate system.
Let us consider a rectilinear Cartesian coordinate system in which a point is defined by a vector
P(x,y,z). We can say the point P is determined by intersection of three mutually planes x=constant, y=
constant and z= constant. Now we introduce another system of coordinates which is defined by the
superposition of three another plans described by u1 = constant, u2 = constant, u3 = constant. The family
of these new plans is not necessarily parallel and mutually perpendicular. These new lanes are intersecting
to each other at point P. These three new surfaces are called curvilinear surfaces. If these new surfaces are
perpendicular to each other, then the coordinate defined by these plans called orthogonal curvilinear
coordinate system. The three surfaces intersect each other at three curves or lines which are called
coordinate lines. The three axis of this new coordinate system are defined by these coordinate lines.
Consider a point in the space is defined by P(x,y,z) in Cartesian coordinate system, and P(u1, u2,u3), in
curvilinear coordinates system as shown in Figure 1.21.

The transformation equations between the curvilinear coordinates and the Cartesian coordinates
are
x = x(u1, u2, u3)

43
MSCPH501

y = y(u1, u2, u3) (1)


z = z(u1, u2, u3).

The functions (1) are single-valued functions of u1, u2, and u3 and are assumed to be
continuously differentiable.
The set of eqs.(1) may be solved for u1, u2, u3 in terms of x, y, z.
u1 = u1(x, y, z) (2a)
u2 = u2(x, y, z) (2b)
u3 = u3(x, y, z) . (2c)
Here, u1, u2, u3 are single- valued, continuously differentiable functions of x, y, and z.
The set of equations (1) and (2) define a one-to-one correspondence between each point (x, y, z)
and the related set of values (u1, u2, u3). The partial derivative of of equation 1 is given as
dx = ïð du6 + ïð du/ + ïð du_ (3a)
ï ï ï
Á & Â
∂y ∂y ∂y
dy = du6 + du/ + du (3b)
∂u6 ∂u/ ∂u_ _

dz = ïð du6 + ïð du/ + ïð du_ . (3c)


ï ï ï
Á & Â
The distance between two points in the space with respect to curvilinear coordinate system is

ds / = dx / + dy / + dz / .
given by

Putting the value from equation (3) and for simplicity 1, 2, 3 are denoted by i, j = 1,2,3 then
(4)

expression for ds / can be represented as


∂x ∂x ∂y ∂y ∂z ∂z
ds / = ó ¼ + + du du ½ . (5)
∂uØ ∂uô ∂uØ ∂uô ∂uØ ∂uô Ø ô
Ø,ô

∂x ∂x ∂y ∂y ∂z ∂z
Further for more simplification if
+ + = hØô
∂uØ ∂uô ∂uØ ∂uô ∂uØ ∂uô
Then
ds/ = ó hØô duØ duô .
Ø,ô
v Ø normal to each surface uØ = const. where i = 1,2,3.
For convenience we introduce a unit vector w
Now for orthogonal curvilinear coordinate system in which surfaces always intersect to each

as w
v Ø. w
v Ø = 1 , but w
v Ø. w
v ô = 0.
other at right angles then

hØØ = hØ ; hôô = hô and höö = hö and hØô = 0 if i ≠ j .


We have

44
MSCPH501

ds / = ( h6 du6 )/ + ( h/ du/ )/ + ( h_ du_ )/ .


Now above equation (5) becomes

In this orthogonal coordinate system h6 , h/ and h_ are called scale factor. The dimension of ds
(6)

in space is of length. The product of h and u is of dimension of length however, the h and u may
have any unit. The distance between two points in space along the coordinate line can be given

dsØ = hØ duØ .
as
(7)

element on the plane defined by coordinate axis dsØ and dsô can be given as
Equation (7) indicates the distance between two points in the coordinate axis therefore surface

d÷Øô = hØ duØ hô duô where i, j = 1,2,3 .


Similarly the volume element in the orthogonal coordinate system defined by coordinate axis dsØ ,
(8)

dsô and dsö can be given as

dVØôö = hØ duØ hô duô hö duö where i, j, k = 1,2,3 .

Differential operator in terms of orthogonal coordinate system:

constant, u3 = constant. Now let us consider a scalar function ψ(u6 , u/ , u_ ) and a vector function
Suppose we have three mutually perpendicular curvilinear planes defined by u1 = constant, u2 =

V = uo6 V6 + uo/ V/ + uo_ V_ where uo6 , uo/ and uo_ are unit vector along the direction of curvilinear

We know that del operator ∇ is a vector which give the maximum rate of change of space of a scalar
coordinates u1 , u2 and u3.

function ψ(u6 , u/ , u_ ) . In Cartesian coordinate system in we consider only one dimension then
δψ ∂ψ
∇ ψ = lim = .
ú →g δx ∂x
As del operator ∇ is a vector quantity, then we can introduce a unit vector along the direction of x and can

∂ψ
write as
∇ ψ = ı̂ .
∂x
In three dimensional case ∇ ψ = ı̂ ï + ȷ̂ ï + k ï .
ïý ïý ïý

δψ ∂ψ ∂ψ
Similarly in curvilinear coordinate system for one dimensional case
∇ ψ = lim = =
ú Á →g δs6 ∂s6 h6 ∂u6
∂ψ
∇ ψ = uo6 .
h6 ∂u6

∂ψ ∂ψ ∂ψ
For three dimensional case

∇ ψ = uo6 + uo/ + uo_ (9)


h6 ∂u6 h/ ∂u/ h_ ∂u_
∂ ∂ ∂
∇ ≡ uo6 + uo/ + uo_ (10)
h6 ∂u6 h/ ∂u/ h_ ∂u_

∇ ψ is nothing but gradient of a scalar function in curvilinear coordinate system.

45
MSCPH501

Divergence: Similarly the divergence of a vector function in the space defined by curvilinear coordinates
u1 , u2 and u3 can be given as

∇ . V = ∇. (uo6 V6 + uo/ V/ + uo_ V_ )

∇ . V = ∇. uo6 V6 + ∇. uo/ V/ + ∇. uo_ V_ . (11)

div (ϕA) = ϕdivA + A Grad ϕ


Since we know that

div (uo6 VØ ) = VØ ∇. uoØ + uoØ . ∇ VØ . (12)

Now we calculate ∇. uoØ as follow and we will put the value in above equation (12)
curl ž Á Ÿ = ∇ × ž Á Ÿ = ∇ × uo6 + ∇ × uo6
v
ð v
ð 6 6
Since (13)
Á Á Á Á

We know that ∇ × u6 =

ð
.
Á

Since curl grad u6 = 0 therefore ∇ × = 0 and above equation (13) becomes



ð
Á

u6 × ∇ ž Ÿ = ∇ × u6 .
6 6
(14)
Á Á

1 uo6 ∂ 1 uo/ ∂ 1 uo_ ∂ 1


Using equation (10)

∇¦ § = + +
h6 h6 ∂u6 h6 h/ ∂u/ h6 h_ ∂u_ h6
1 uo6 ∂h6 uo/ ∂h6 uo_ ∂h6
∇¦ § = − _ + / − / .
h6 h6 ∂u6 h6 h/ ∂u/ h6 h_ ∂u/

Putting this value in equation (14)

uo6 × uo6 ∂h6 uo6 × uo/ ∂h6 uo6 × uo_ ∂h6


∇ × uo6 = − + − .
h6 / ∂u6 h6 h/ ∂u/ h6 h_ ∂u/

Using the identity uØ × uØ = 0


uo/ ∂h6 uo_ ∂h6
∇ × uo6 = − (15a)
h6 h_ ∂u_ h6 h/ ∂u/

uo_ ∂h/ uo6 ∂h/


similarly

∇ × uo/ = − (15b)
h/ h6 ∂u6 h/ h_ ∂u_

uo6 ∂h_ uo/ ∂h_


∇ × uo_ = − . (15c)
h_ h/ ∂u/ h_ h6 ∂u6

div(A × B) = B. curl A − A. curl B


Now again using identity

46
MSCPH501

∇ × uo6 = ∇. (uo/ × uo_ ) = uo_ . (∇ × uo/ ) − uo/ . (∇ × uo_ )

Putting the value of ∇ × u


o / and ∇ × uo_ from equation 15 we have

1 ∂(h6 h/ )
∇. uo6 = .
h6 h/ h_ ∂u6

∇. (uo6 V6 ) = V6 ∇. uo6 + uo6 ∇ V6


From equation (12)

V6 ∂(h/ h_ ) uo6 ∂V6 uo6 ∂V6 uo_ ∂V6


∇. (uo6 V6 ) = + uo6 . ¦ + + §
h6 h/ h_ ∂u6 h6 ∂u6 h6 ∂u/ h_ ∂u_
V6 ∂(h/ h_ ) 1 ∂V6 1 ∂(V6 h/ h_ )
∇. (uo6 V6 ) = + = . (16a)
h6 h/ h_ ∂u6 h6 ∂u6 h6 h/ h_ ∂u6

1 ∂(V/ h_ h6 )
Similarly

∇. (uo/ V/ ) = (16b)
h6 h/ h_ ∂u/
1 ∂(V_ h6 h/ )
∇. (uo_ V_ ) = . (16c)
h6 h/ h_ ∂u_

Combining all equation and using equation (11)

1 ∂(V1 h2 h3 ) ∂(V2 h3 h1 ) ∂(V3 h1 h2 )


div V = ∇ . V == Ê + + Ë. (17)
h1 h2 h3 ∂u1 ∂u2 ∂u3

Laplacian: The Laplacian operator is define as ∇ . ∇ or denoted as ∇/ . Putting the value of ∇ ψ from
equation (9 ) in place of vector V in equation (11)

1 ∂ ∂ψ ∂ ∂ψ ∂ ∂ψ
∇ . ∇ψ = æ h/ h_ + h_ h6 + h6 h/ ç
h6 h/ h_ ∂u6 h6 ∂u6 ∂u/ h/ ∂u/ ∂u_ h_ ∂u6

1 ∂ ∂ψ ∂ ∂ψ ∂ ∂ψ
∇/ ψ = æ h/ h_ + h_ h6 + h6 h/ ç (18)
h6 h/ h_ ∂u6 h6 ∂u6 ∂u/ h/ ∂u/ ∂u_ h_ ∂u6

Curl : Similarly the curl of a vector F in curvilinear coordinate system is given as

curl F = ∇ × F = ∇ × (uo6 F6 + uo/ F/ + uo_ F_ )

We have vector F as F = uo6 F6 + uo/ F/ + uo_ F_

∇ × F = ∇ × uo6 F6 + ∇ × uo/ F/ + ∇ × uo_ F_ . (19)

We know that curl (ϕA) = ϕ curl A − A × grad ϕ

curl (uo6 F6 ) = F6 (∇ × uo6 ) − uo6 × ∇F6.

47
MSCPH501

Substituting the value of ∇ × uo6 from equation (15) and ∇F6 from equation (9)

uo/ ∂h6 uo_ ∂h6 uo6 ∂V6 uo/ ∂V6 uo_ ∂V6
∇ × (uo6 F6 ) = F6 æ − ç − uo6 × æ + + ç
h6 h_ ∂u/ h6 h/ ∂u/ h6 ∂u6 h/ ∂u/ h_ ∂u_

uo/ F6 ∂h6 uo_ F6 ∂h6 uo_ ∂F6 uo/ ∂V6


=æ − − + ç
h6 h_ ∂u_ h6 h/ ∂u/ h/ ∂u_ h_ ∂u_

F6 ∂h6 1 ∂F6 F6 ∂h6 1 ∂F6


= uo/ æ + ç − uo_ æ + ç
h6 h_ ∂u_ h6 ∂u_ h6 h/ ∂u/ h_ ∂u/

uo/ ∂(F6 h6 ) uo_ ∂(F6 h6 )


= −
h6 h_ ∂u_ h6 h/ ∂u/

uo/ ∂(F6 h6 ) uo_ ∂(F6 h6 )


∇ × (uo6 F6 ) = − . (20a)
h6 h_ ∂u_ h6 h/ ∂u/

uo_ ∂(F/ h/ ) uo6 ∂(F/ h/ )


Similarly

∇ × (uo/ F/ ) = − (20b)
h6 h/ ∂u6 h/ h_ ∂u_

uo6 ∂(F_ h_ ) uo/ ∂(F_ h_ )


∇ × (uo_ F_ ) = − . (20c)
h/ h_ ∂u/ h_ h6 ∂u6

Substituting all values in equation (19)

uo/ ∂(F6 h6 ) uo_ ∂(F6 h6 ) uo_ ∂(F/ h/ ) uo6 ∂(F/ h/ ) uo6 ∂(F_ h_ )
∇×F = − + − +
h6 h_ ∂u_ h6 h/ ∂u/ h6 h/ ∂u6 h/ h_ ∂u_ h/ h_ ∂u/
uo/ ∂(F_ h_ )

h_ h6 ∂u6

uo6 ∂(F_ h_ ) ∂(F/ h/ ) uo/ ∂(F6 h6 ) ∂(F_ h_ ) uo_ ∂(F/ h/ ) ∂(F6 h6 )


∇×F= æ − ç+ æ − ç+ æ − ç. (21)
h/ h_ ∂u/ ∂u_ h_ h6 ∂u_ ∂u6 h6 h/ ∂u6 ∂u/

The determinant form of above equation is given as

h6 uo6 h/ uo/ h_ uo_


1 ∂ ∂ ∂
∇×F = ® ®. (22)
h6 h/ h_ ∂u6 ∂u/ ∂u_
h6 F6 h/ F/ h_ F_

1.10.1 Spherical Coordinate System:


A spherical coordinate system is a coordinate system for three-dimensional space where the
position of a point is specified by three coordinates (r, θ, ) as radial distance r of that point from
a fixed origin, its polar angle θ measured from a fixed zenith direction, and the azimuthal angle

48
MSCPH501

of its orthogonal projection on a reference plane that passes through the origin and is
orthogonal to the zenith, measured from a fixed reference direction on that plane. It can be seen
as the three-dimensional version of the polar coordinate system. The radial distance is also called
the radius or radial coordinate. The polar angle may be called colatitude, zenith angle, normal
angle, or inclination angle. In physics (r, θ, φ) gives the radial distance, polar angle, and
azimuthal angle,

If the coordinate of a point is given by (r, θ, ∅) in spherical coordinate system and (x,y,z) in
Cartesian coordinate system. From the Figure 1.22

x = r sin θ cos ∅ (23a)

y = r sin θ cos ∅ (23b)

z = r cos θ (23c)

and r = .x / + y / + z / . (24)

We know x is function of (r, θ, ∅) thus x ≡ x(r, θ, ∅)

∂x ∂x ∂x
dx = dr + dθ + d∅.
∂r ∂θ ∂∅

49
MSCPH501

Partially differentiating equation (23a) with respect to r, θ, ∅ and putting the values in this
equation we get

NI = 4, ∅N + , ∅ N, − 4, ∅ N∅ (25)

Similarly y≡ J( , ,, ∅) and

›J ›J ›J
NJ = N + N, + N∅
› ›, ›∅

NJ = 4, 4∅N + , 4 ∅ N, + 4, ∅ N∅ (26)

and z ≡ K( , ,, ∅)

›K ›K ›K
NK = N + N, + N∅
› ›, ›∅

NK = ,N − 4 , N,. (27)

We know the line element ds in Cartesian coordinate system is given as

N /
= NI / + NJ / + NK / . (28)

Substituting the value of dx, dy and dz from above equations 25,26 and 27

N /
=N /
+ /
N, / + /
4/ , N∅/ . (29)

Compare this equation (29) with standard curvilinear equation as given below

N /
= (ℎ6 N‚6 )/ + (ℎ/ N‚/ )/ + (ℎ_ N‚_ )/ . (30)

We have

ℎ6 = 1 4N ‚6 = ; ℎ/ = 4N ‚/ = , ; ℎ_ = 4 , 4N ‚/ = ∅. (31)

Gradient:

Putting the coefficients in the equation of gradient in curvilinear coordinate

› › ›
t N = = ¢o6 + ¢o/ + ¢o_
ℎ6 ›¢6 ℎ/ ›¢/ ℎ_ ›¢_

› › ›
t N = = ¢o• + ¢o, + ¢o∅
1› ›, 4, ›∅
› › ›
≡ ¢o• + ¢o, + ¢o∅ . (32)
› ›, 4, ›∅

50
MSCPH501

Divergence:

In orthogonal curvilinear equation divergence of a vector can be given as

1 ›(•1 ℎ2 ℎ3 ) ›(•2 ℎ3 ℎ1 ) ›(•3 ℎ1 ℎ2 )


N •= .• = Ê + + Ë.
ℎ1 ℎ2 ℎ3 ›‚1 ›‚2 ›‚3

Putting the value of ℎ6 ℎ/ ℎ_ and ‚6 ‚/ ‚_ from equation (31)

1 ›Y• 2 4 ,Z ›(•, 4 ,) ›(•∅ )


N •= Ê + + Ë
2 4, › ›, ›∅

1› 2 1 › 4, 1 ›
N •≡ 2 ›
+ + . (33)
4 , ›, 4 , ›∅

Laplacian :

In orthogonal curvilinear equation Laplacian of a funtion can be given as

1 › › › › › ›
/
= æ ℎ/ ℎ_ + ℎ_ ℎ6 + ℎ6 ℎ/ ç.
ℎ6 ℎ/ ℎ_ ›¢6 ℎ6 ›¢6 ›¢/ ℎ/ ›¢/ ›¢_ ℎ_ ›¢6

Putting the value of ℎ6 ℎ/ ℎ_ and ‚6 ‚/ ‚_ from equation (31)

1 › › › › › 1 ›
/
= Ê ¦ 2 4, §+ ¦ 4, § + ¼ ½Ë
2 4, › › ›, ›, ›∅ 4 , ›∅

1 › › 1 › › 1 ›
/
= 2›
¦ 2 §+ ¦ 4, §+ . (34)
› 2 4 , ›, ›, 2 4 , ›∅
2

1.10.2 Cylindrical Coordinate system:

A cylindrical coordinate system is a three-dimensional coordinate system (r, Ɵ, z ) that specifies


point positions by the distance from a chosen reference axis z, the direction from the axis relative
to a chosen reference direction, and the distance from a chosen reference plane perpendicular to
the axis. The cylindrical coordinate system consist of a right circular cylinder having reference
axis z, r is the perpendicular distance of a point from z axis and Ɵ is the angle r with respect to x
axis as shown in figure . If (x,y,z) are Cartesian coordinate of point specified by (r, Ɵ, z ).
Thus
I= , (35 )
J= 4, (35“)

51
MSCPH501

K=K (35 )

I = I ( , ,, K)

›I ›I ›I
NI = N + N, + NK .
› ›, ›K
Partially differentiated equation (35) and putting the value in above equation

NI = ,N − 4 , N,. (36 )

Similarly

NJ = 4, N − , N, (36“)

NK = NK (36 )

In Cartesian coordinate system, the line segment is given as

N /
= NI / + NJ / + NK / . (37)

Substituting the value of dx, dy and dz from above equations (36)

N /
= /
,N /
+ /
4/ , N, /
− 2 4, , N N, + 4/ ,N /
+ / /
, N, /
+2 4, , N N, + NK /

N /
=N /
+ /
N, / + NK / . (38)

52
MSCPH501

Compare this equation (29) with standard curvilinear equation as given below

N /
= (ℎ6 N‚6 )/ + (ℎ/ N‚/ )/ + (ℎ_ N‚_ )/ .

We have

ℎ6 = 1 4N ‚6 = ; ℎ/ = 4N ‚/ = , ; ℎ_ = 1 4N ‚_ = K. (39)

Now we can put the values of h and q and find out the value of gradient, curl and Laplacian.

Gradient:

Putting the coefficients in the equation of gradient in curvilinear coordinate

› › ›
t N = = ¢o6 + ¢o/ + ¢o_
ℎ6 ›¢6 ℎ/ ›¢/ ℎ_ ›¢_

putting the values of hand q

› › ›
t N = = ¢o• + ¢o, + ¢oK
1› ›, ›K
› › ›
≡ ¢o• + ¢o, + ¢oK . (40)
› ›, ›K

Divergence:

In orthogonal curvilinear equation divergence of a vector can be given as

1 ›(•1 ℎ2 ℎ3 ) ›(•2 ℎ3 ℎ1 ) ›(•3 ℎ1 ℎ2 )


N •= .• = Ê + + Ë.
ℎ1 ℎ2 ℎ3 ›‚1 ›‚2 ›‚3

Putting the value of ℎ6 ℎ/ ℎ_ and ‚6 ‚/ ‚_ from equation (39)

1 › • ›(•, ) ›(•K )
N •= Ê + + Ë
1. . 1 › ›, ›K

1 › ( • ) 1 ›(•, ) ›•K
N •= + +
› ›, ›K
1› 1 › ›
N ≡ + + . (41)
› ›, ›K
Laplacian :

In orthogonal curvilinear equation Laplacian of a function can be given as

1 › › › › › ›
/
= æ ℎ/ ℎ_ + ℎ_ ℎ6 + ℎ6 ℎ/ ç.
ℎ6 ℎ/ ℎ_ ›¢6 ℎ6 ›¢6 ›¢/ ℎ/ ›¢/ ›¢_ ℎ_ ›¢6

53
MSCPH501

Putting the value of ℎ6 , ℎ/ , ℎ_ and ‚6 , ‚/ , ‚_ from equation (39)

1 › ∂ψ ∂ 1 ∂ψ ∂ ∂ψ
/
= Ê ¦r §+ ¼ ½ + ¦r §Ë
› ∂r ∂θ r ∂θ ∂z ∂z

1 ∂/ ψ ∂ψ 1 ∂/ ψ ∂/ ψ
∇/ ψ = ʼr / ½ + +¼ ½ + r Ë
r ∂r ∂r r ∂θ2 ∂z2

∂/ ψ 1 ∂ψ 1 ∂/ ψ ∂/ ψ
∇/ ψ = + + +
∂r / r ∂r r2 ∂θ2 ∂z2

∂/ 1 ∂ 1 ∂/ ∂/
∇/ ≡ + + + . (42)
∂r / r ∂r r2 ∂θ2 ∂z2

1.11 Summary:
1. Physical quantities are of two types, scalar and vector. The scalar quantities have
magnitude only but no direction. The vector quantities have magnitude as well as direction.
2. Two vector quantities can be added with parallelogram law and triangle law. In
parallelogram law, the resultant is denoted by the diagonal of parallelogram whose adjacent
sides are represented by two vectors. In triangle law, we place the tail of second vector on the
head of first vector, and resultant is obtained by a vector whose head is at the head of second
vector and tail is at the tail of first vector.
3. For subtraction, we reverse the direction of second vector and add it with first vector.
4. In case of more than two vectors we simply use Polygon law of vector addition.
5. Any vector can be resolved into two or more components. By adding all components we

If a vector makes angles -, ä and å with three mutual perpendicular axes x, y and z
can find the final vector.

-, ä and å are called direction cosines.


6.

Scalar product of two vectors is defined as **** * = AB cos , which is a scalar quantity.
respectively then
A. B
* = ) 4∅ 4o which is a vector
7.
×)
*.
8. Vector product of two vectors is defined as
quantity. The direction of vector is perpendicular to and )

vectors **** * = AB or A* × B * = 0.
9. If two vectors are parallel to each other then they are said to be collinear. For collinear
A. B

**** * = 0.
10. If the angle between two vectors is 900, then vectors are called orthogonal. In this case
A. B
11. Cross product of two vectors can also be calculated by determinant. The determinant
form of cross product is

54
MSCPH501

P
* =•
×) = < > •.
)= )< )>

12. Scalar triple product of three vectors can also be calculated by determinant. The
determinant form of Scalar triple product is
= < >
*** . ()
* × p ) = • )= )< )> •.
p= p< p>
13. Vector triple product is defined as

* × p Z = Y . p Z)
× Y) * − Y .)
* Zp .

14. Differentiation and integration techniques are used to solve and explain many physical
problems. Differentiation of a vector is defined as

= lim‹Œ→g ‹Œ = lim‹Œ→g .
•• ‹• • (Œ ‹Œ)5• (Œ)
•Œ ‹Œ

15. If we further differentiate function with respect t then it is called second order differentiation.
If should be cleared that the derivatives of a vector (say ) are also vector quantities. If r is a
••
position vector of a particle at time t then •Œ denotes its velocity.

›Q Q(I + †I, J, K) − Q(I, J, K)


16. Partial derivative is defined as

= lim
›I ‹=→g †I
In case of partial derivative with respect to a variable, all the other remaining variables are taken
as constant.

17. Vector differential operator del is denoted by ∇ and defined as

› › ›
∇= i +P + .
›I ›J ›K

18. The gradient of a scalar function ∅ is defined as

grad ∅ = ∇∅ = ži œ= + P œ< + Ÿ ∅.
œ œ œ
œ>

19. The divergence is dot product of del operator with any vector point function ***
Q and is given
as

div. Q = ∇. Q = ž + P œ< + Ÿ. ( Q= + PQ< Q> ) where Q = Q= + PQ< Q>


œ œ œ
œ= œ>

55
MSCPH501

= + +
ϥd ϥe ϥf
œ= œ< œ>
.

20. The curl of a vector h = h= + h< P + hb is defined as

Curl h = ∇ × h = ( +P + ) × Yh= + h< P + hb Z.


œ œ œ
œ= œ< œ>

21. The integral of a vector function h along a line or curve is called line integral and given as
*** .
´ h . Nr
µ

22. If h is a vector function and s is a surface, then surface integral of a vector function h over
the surface S is given as ∬• h . N ** .

defined as ´¹ hN• = ´= ´< ´> h. NINJNK.


23. If dV denotes the volume defined by dxdydz then the volume integration of a vector F is

24. Gauss divergence theorem transforms surface integral into volume integral and vice-versa.
The theorem states that the surface integral of a vector filed h over a closed surface s is equal to
the volume integral of divergence of h taken over the volume enclosed by surface s.

**** = ∭ N
∬• h . N hN .

25. Gauss law is a result of Gauss theorem in electric field. According to this law the total
6
∈Ï
electric flux through a closed surface is equal to times total charge enclosed inside the surface.

∬• Ð. N = ∈ ( total charge inside the surface).


6
Ï

26. Gauss law in differential form:


Þ
N Ð= .
∈g

27. Poisson’s equation and Laplace equation:


Þ
∇/ ∅ = −
∈g

equation and operator ∇/ is an operator called Laplacian operator and defined as


This is called Poisson’s equation. Poisson’s equation is basically second order differential

›/ ›/ ›/
∇/ = ∇. ∇= + +
›I / ›J / ›K /

56
MSCPH501

If there is no charge inside the volume i.e. Þ=0, then above equation becomes Laplace equation

∇/ ∅ = 0.

28. Green’s Theorem for a Plane: If ∅6 (I, J) and∅/ (I, J) are two scalar functions which are
4N
œ∅Á œ∅&
œ< œ=
continuous and have continuous derivatives over a region R bounded by simple
closed curve c in x-y plane, then

∮Û(∅6 NI + ∅/ NJ) = ∬È ( œ= − )NINJ.


& œ∅ œ∅Á
œ<

29. Stoke’s Theorem: Stoke’s theorem transforms the surface integral of the curl of a vector into
line integral of that vector over the boundary C of that surface. According to this theorem the
surface integral of the curl of a vector taken over the surface s bounded by a curve c is equal to
the line integral of the vector A along the closed curve c.

**** = ∮
∬• ¢ r . N **** .
.N
Û

1.12 Glossary
Vector- Physical quantity with direction
Scalar quantities- Physical quantity without direction
Collinear – in same line or direction
Orthogonal- perpendicular to each other
Coplanar – on same plane Displacement – net change in location of a moving body.
Differentiation- instantaneous rate of change of a function with respect to one of its variables
Integration- The process of finding a function from its derivative. (Reverse of differentiation)
Partial derivative- derivative of a function with respect to a variable, if all other remaining
variables are considered as constant
Operator – An Operator is a symbol that shows a mathematical operation.
del operator - vector differentiation operator
gradient- derivative of function.(rate of change of a function or slope)
divergence- rate at which density exits at a given region of space. (flux density)
Curl- describes the rotation of vector field.
line integral- Integration along a line.
surface integral- Integration along a surface.
volume integral- Integration along a volume.
Transformation- conversion

57
MSCPH501

Flux – scalar product of a field vector and area


divergence- rate at which density exits in a given region of space. (flux density)
Curl- describes the rotation of vector field.

1.13 Reference Books:


1. Mathematical Physics – Satya prakash, Sultan Chand, Meerut
2. Mathematical Physics- H K Dass, S Chand and Company Ltd. New Delhi

1.14 Suggested readings:


1. Mathematical Methods for Physicists: Arfken.
2 Mathematical Methods for Physics: Wyle.
3. P.K. Chakrabarti and S.N. Kundu, A Text Book of Mathematical Physics, New Central Book
Agency, Kolkata.
4. A.K. Ghatak, I.C. Goyal and S.H. Chua, Mathematical Physics Macmillan India, New Delhi.
5. B S Rajput, Mathematical Physics, Pragati Publication

1.15 Terminal questions


1.15.1 Short answer type questions

1. Define unit vector, like vector and equal vectors.

What angle does the vector 3 + √2P + make with y axis?


2. What are direction cosines? Give its significance.
3.
4. What is the condition for vector to be collinear?
5. Explain the difference between dot and cross products.
6. What is angular momentum? How the direction of angular momentum can be
decided?
7. Give some examples of dot product in physics.
8. Give some examples of cross product in physics.
9. Define scalar triple product.

11. Define gradient of a scalar function ∅.


10. How the angle between two vectors can be obtained?

12. Show that ∇∅ is a vector whose magnitude is equal to maximum rate of change of ∅

13. Show that ∇∅ is perpendicular to surface ∅.


with respect to space variable.

14. Solve ∇ ž Ÿ for ≠0


6

58
MSCPH501

15. If vector h = 6IK − J / j + yzk then calculate ´Ì h . 4o NÃ where S is the surface of a


cube with boundaries I = 0 I = 2, y= 0 J = 2, K = 0 K = 2.
16. Obtain the value jt N ∅( )l ×
17. Find the area of parallelogram determined by the vectors ( + 2j + 3k) and (−3 −
2j + 4k).

19. If F is a scalar function which is solution of Laplace equation ∇/ h = 0 in a volume


18. Explain the physical significance of Gauss’s divergence theorem.

V bounded by the piecewise smooth surface S, then apply the Gauss theorem and

20. ∬• 4o. ∇h NÃ = 0
show that

21. Verify Green’s theorem in a plane for j(3I / − 8J / )NI + (4J − 6IJ)NJl where C is
boundary of a region defined by I = 0, J = 0, I + J = 1
**** = 0
22. Prove that 4o. NÃ = 0 and ∬ (∇ × h ) . NÃ

* taken over the surface bounded by the given closed curve then show
23. If the line integral of a vector around a closed curve is equal to the surface integral
of the vector )
that ) * = ¢ r .

1.15.2 Essay type questions

1. If |@ + u| = |@ − u| , show that A and B are perpendicular to each other.

Show that = 5 + 2P + 4 4N ) = 2 + 3P − 4 are perpendicular to each other.


2. What is the significance of dot product? Give the properties of cross product.
3.
4. What is the vector product? Give the properties of vector product.
5. Find out the condition if two vectors are collinear.
6. Find the components of a vector along and perpendicular to the direction of another vector.
7.
for the divergence of a vector h .
Define divergence of a vector function and its physical significance. Obtain the expression

8. Define curl of a vector function and its physical significance. Obtain the expression for the
curl of a vector h .
9. Prove that ∇ × Y × ) * Z = YB* .∇
* ZA **** . ∇
* − (A *** ))
* + N )
* −)* N
10. Prove that any vector function can be expressed as the sum of lamellar vector and solenoidal
vector.
11. Derive the equation of continuity
+ div J = 0
ï
ï
12.

14. Show that ¢* × is solenoidal if ¢


* and are irrotational.
13. And show that how this equation express charge conservation.

15. State and proof Gauss’s divergence theorem.


16. State and prove Stoke’s theorem in vector analysis.

59
MSCPH501

18. Verify Green’s theorem in a plane for ∮Û(3I / − 8J / )NI + (4J − 6IJ)NJ where c is the
17. State and prove Green’s theorem in a plane.

boundary defined by J = I6// ; J = I / .

1.15.3Numerical question

1. Calculate the dot product of vectors @ = 6 + 7P + 4N u = + 3P + 2 .


2. A particle moves from the position (3 + 3P + 2 )q to another position (-2 + 2P +
4 )q under the influence of a force Ä = 3 + 2P + 4 newton. Calculate the work

3. Obtain the projection of a vector @ = 3 + 4P + 5 along a line which originates at a point


done by the force.

4. Find the unit vector in the direction of resultant vectors of @ = 6 + 7P + 4N u = +


(2, 2, 0) and passing through another point (-2, 4, 4).

3P + 2 .

60
MSCPH501

UNIT - 2 MATRIX

STRUCTURE:

1.0 Objectives
1.1 Introduction
1.2 Matrices
1.2.1 Definition of a Matrix
1.2.2 Notations
1.2.3 Order of a matrix
1.2.4 Equality of two Matrices
1.2.5 Transpose of a matrix
1.2.6 Conjugate of a matrix
1.2.7 Trace of a matrix
1.3 Types of Matrices:
1.3.1 Zero matrix or Null matrix
1.3.2 Rectangular matrix:
1.3.3 Square matrix
1.3.4 Diagonal matrix
1.3.5 Identity or unit matrix:
1.3.6 Triangular matrix
1.3.7 Single element matrix
1.3.8 Scalar matrix
1.3.9 Symmetric and skew-symmetric matrix
1.3.10 Orthogonal matrix:
1.3.11 Nilpotent matrix
1.3.12 Idempotent matrix
1.3.13 involuntary matrix
1.3.14 Singular matrix
1.3.15 unitary matrix
1.3.16 Hermitian and skew Hermitian matrix
1.4 Properties of a matrix
1.4.1 Addition of matrices
1.4.2 Subtraction of matrices
1.4.3 Multiplying a scalar to matrix
1.4.4 Multiplication of matrices
1.4.4.1 Properties of matrix multiplication
1.4.4.2 Determinant of a matrix
1.5 Important properties of Determinants
1.6 Minor of a matrix

61
MSCPH501

1.7 Cofactors of a matrix


1.8 Adjoint of a matrix
1.9 Properties of Adjoint
1.10 The inverse or reciprocal of a matrix
1.11 The rank of a matrix
1.12 Normal form (Canonical form)
1.13 Eigen Values
1.14 Cayley-Hamilton theorem
1.15 Summary
1.16 References
1.17 Exercise

1.0 Objectives

62
MSCPH501

After studying this unit, you should be able to-

• Knowledge on matrices
• Knowledge on matrix operations
• Matrix as a tool of solving linear equations with two or three unknowns
• Solve application problems that can be modeled by systems of linear equations.

1.1 Introduction: The understanding of matrices is essential in various field of mathematics.


Matrices are one of the most powerful tools in mathematics. This mathematical tool simplifies
our work to a great extent when compared with other straight forward methods. The evolution of
concept of matrices is the result of an attempt to obtain compact and simple methods of solving
system of linear equations. Matrices are not only used as a representation of the coefficients in
system of linear equations, but utility of matrices far exceeds that use. Matrix notation and
operations are used in electronic spreadsheet programs for personal computer, which in turn is
used in different areas of business and science like budgeting, sales projection, cost estimation,
analyzing the results of an experiment etc. Also, many physical operations such as magnification,
rotation and reflection through a plane can be represented mathematically by matrices. This
mathematical tool is not only used in certain branches of sciences, but also in genetics,
economics, sociology, modern psychology and industrial management. In this chapter, we shall
find it interesting to become acquainted with the fundamentals of matrix and matrix algebra.

1.2 Matrices

1.2.1 Definition of a Matrix

“A rectangular array of real or complex numbers is called a matrix”.

The horizontal arrays of a matrix are called its rows and the vertical arrays are called its columns.
A matrix having m rows and n columns is said to have the order m× n.

A matrix A of order m× n can be represented in the following form:

66 6/ … … 6©
/6 // … … /©
⋮ ⋮ ⋮ ⋮ ⋮
⋮ ⋮ ⋮ ⋮ ⋮
A= .

6 / … … ©

Where, `b is the entry at the intersection of the ith row and jth column.
In a more concise manner, we also denote the matrix A by [ `b ] by suppressing its order.

63
MSCPH501

1 3 7
».
4 5 6
Let A =º
Then 66 = 1, 6/ = 3, 6_ = 7, a/6 = 4, a// = 5, and a/_ = 6.

“A matrix having only one column is called a column vector; and a matrix with only one row is
called a row vector”.

Remarks:-
1. A matrix is a collection of objects of numbers over a field of numbers, the elements of the
field being called the scalars.
2. It has no numerical value.
3. A matrix cannot be equal to a number.

1.2.2 Notations

corresponding small letters `b , “`b , `b ,……


Matrices are denoted by capital letters A, B, C,.. and their elements are denoted by the

parentheses i.e. ( ) and double bars i.e. ‖ ‖ are also sometimes used to indicate a matrix.
Generally, we have used only a pair of brackets i.e. [ ] to denote a matrix, but a pair of

1.2.3 Order of a matrix


The order of a matrix is defined in terms of its number of rows and columns.
Order of a matrix = No. of rows × No. of columns.

Two matrices A = [aØô ] and B = [bØô ] having the same order m× n are equal if aØô = bØô for each i
1.2.4 Equality of two Matrices

= 1, 2. . . m and j = 1, 2, . . . , n.
In other words, two matrices are said to be equal if they have the same order and their
corresponding entries are equal.
1.2.5 Transpose of a Matrix: If in given m × n matrix A = [aØô ], we interchange the rows and
the corresponding columns, the new matrix obtained is called the transpose of the matrix A.

2 3 4 2 0 5
The transpose of A is denoted by A’ or .

For example, if A = 0 9 7 and its transpose of A = ′ = 3 9 6 .


5 6 2 4 7 2

Thus, the transpose of a row vector is a column vector and vice-versa.


1.2.6 Conjugate of a matrix:-

1+ 2−3 4
Let A= º ».
7+2 − 3−2

1− 2+3 4
Conjugate of matrix A is ̅ = º ».
7−2 3+2

1.2.7 Trace of a matrix:-The sum of all elements in the principal diagonal is called the trace of

Trace of A= tr A= ∑©`Å6 `` .
the matrix.

64
MSCPH501

2 8 0
Let A= 4 3 7.
3 6 9
The trace of matrix A is = 2+3+9=14.

1.3 Types of Matrices:

1.3.1 Zero matrix or Null matrix: A matrix in which each entry is zero, called a zero-
matrix, denoted by 0.

For example,

0 0
A= º » is a null matrix denoted by O or O/×/ .
0 0

2 1 5
1.3.2 Rectangular matrix: Any m×n matrix is called a rectangular matrix, if m≠n.
For example = º ».
6 8 4
1.3.3 Square matrix: A matrix having the number of rows equal to the number of columns

3 7 2
is called a square matrix. Thus, its order is m× m (for some m) and is represented by m only.

For example A = 4 5 7 is a square matrix of order 3. In a square matrix, A = [aØô ],


3 6 9
of order n, the entries a66 , a// , . . . , a"" are called the diagonal entries.

1.3.4 Diagonal matrix: A square matrix A = [aØô ] is said to be a diagonal matrix if aØô =
0 for i≠ j. In other words, all its non-diagonal elements are zero.

1 0 0
For example, A = 0 3 0.
0 0 4

1.3.5 Identity or unit matrix: A square matrix A = [ `b ] with `b =(1 if i = j and 0 if i≠ j


is called the identity matrix, or in other words if all the diagonal elements are unity and
diagonal elements are zero. It is denoted by I.

For example,

1 0 0
1 0
I= 0 1 0 ¢4 q I Q N 3, 4N º » ¢4 q I Q N 2.
0 1
0 0 1

1.3.6 Triangular matrix: A square matrix A = [aij], all of whose elements below the
leading diagonal are zero, is said to be an upper triangular matrix or in other
words `b = 0 for i> j.

65
MSCPH501

1 3 2
For example- A= 0 4 1.
0 0 6

A square matrix A = [ `b ], all of whose elements above the leading diagonal are zero, is
said to be a lower triangular matrix or in other words aØô = 0 for i< j.

2 0 0
For example- A = 4 1 0 .
5 6 7

1.3.7 Single element matrix: A matrix j `b l of order 1×1 is defined to be equal to a scalar
‘a’.
1.3.8 Scalar matrix: A diagonal matrix, in which all the diagonal elements are equal to a

2 0 0
scalar, is called a scalar matrix.

For example- A= 0 2 0 is a scalar matrix of order 3 and is also written as diag [2, 2,
0 0 2
2].
.

1.3.9 Symmetric and skew-symmetric matrix: A square matrix A is called symmetric if


transpose of A i.e. A’= A and skew-symmetric if A’= −A.

ℎ t 0 −ℎ −t
#ℎ “ Q$ Jqq 4N # ℎ −Q $ Ú Jqq .
t Q t Q 0
For example,

1.3.10 Orthogonal matrix: A square matrix A is said to be orthogonal if the product of the
matrix A and the transpose matrix A’ is an identity matrix i.e., AA’= A’A = I.
#
1.3.11 Nilpotent matrix: The matrices A for which a positive integer k exists such that =
0 are called nilpotent matrices. The least positive integer k for which # = 0 is called the
order of nilpotency.

Let A = º “/ “/ » , /
= º “/ “/ » æ “ “/
ç=º
0 0
».
− − “ − − “ – / − “ 0 0
/
1.3.12 Idempotent matrix: The matrices that satisfy the condition that = A are called
Idempotent matrices.

For example-

66
MSCPH501

2 −2 −4 2 −2 −4 2 −2 −4 2 −2 −4
= −1 3 4 , /
= −1 3 4 −1 3 4 = −1 3 4 .
1 −2 −3 1 −2 −3 1 −2 −3 1 −2 −3

1.3.13 Involuntary matrix: A matrix A will be called an involuntary matrix, if / = %(unit


matrix).

1 2
1.3.14 Singular matrix: If the determinant of the matrix is zero, then the matrix is known as
» is singular matrix because | | = 6 − 6 = 0.
3 6
singular matrix e.g. A=º

1.3.15 Unitary matrix: A square matrix A is said to be unitary if its product with Transpose

= %.
of the conjugate gives the Identity matrix.
Ü
Ü
Where denotes the transpose of the conjugate of matrix A.

6 ` 56 ` 65` 65`

Let = #6/ ` /
65`
$, Ü
= #565`
/ /
6 `
$ , Ü
= %.
/ / / /

matrix if transpose of the conjugate of A i.e. Ü = ( ̅)′= A and skew-Hermitian if


1.3.16 Hermitian and skew-Hermitian matrix:-A square matrix A is called Hermitian

Ü
= ( ̅)′=−A.

1.4 Properties of Matrix:-


(a) The commutative law-
If A and B are two matrices of the same order, say m×n.

If = „ `b … 4N ) = „“`b … = 1,2, … … … , q
A+B = B+A

Then A+B = „ `b … + „“`b … = „ `b + “`b …


j=1,2,… …… ….,n.

= „“`b + `b … 4 “`b 4N `b r
=„“`b … + „ `b … = ) +
i.e. the commutative law of addition holds.

(b) The associative law-


If A, B and C are three matrices of the same order, then

Let = „ `b …, ) = „“`b … 4N p = j `b l
(A+B)+C=A+ (B+C)

(A+B)+C= Y„ `b … + „“`b …Z + „ `b …
= („ `b + “`b … + j `b l)
= „Y `b + “`b Z + `b …

67
MSCPH501

=„ `b + Y“`b + `b Z…, `b , “`b 4N `b r .


= „ `b … + Y„“`b + `b …Z
=A+ (B+C)
i.e. the associative law of addition holds.

(c) The Distributive law-

k (A+B) = j `b + “`b l
If A and B are two matrices of the same order m×n and k is a scalar, then

= j ( `b + “`b )l
= „ `b … + „ “`b …
= „ `b … + „“`b …, r .
= kA+kB.
The distributive law of addition holds.

(d) Existence of Additive identity-


If A be a matrix of any order, say, m×n and O a null matrix of the same order such that
when it is added to A leaves it unchanged.
A+O = A

Proof: if = „ `b … 4N ‡ 4¢rr q I
Then O is said to be the additive identity of A.

Then A+O= j `b + ‡l
= „ `b … since a zero added to any scalar leaves it unchanged.
= A.
Hence O is said to be an additive identity of A.

(e) Existence of Additive Inverse-


If A be a matrix of any order say m×n, and there exists a matrix –A of the same order
such that if it is added to A, gives a null matrix O.
A+ (-A) =O

Let A=j `b l
(-A) is said to be the additive inverse of A.

-A= -„ `b … = „− `b …
A+ (-A) = „ `b … + „− `b … = „ `b − `b … = 0.
Hence (-A) is said to be an additive inverse of A.

(f) The cancellation law-


If A, B and C are three matrices conformable for addition then the relation
A+B=A+C.

Let A= „ `b …, ) = „“`b … 4N p = „ `b … i=1, 2,……..m


If B = C.

j=1, 2,……..n.
Then the relation

`b + “`b = `b + `b .
A+B = A+C

68
MSCPH501

Which yields, “`b = `b 4 `b , “`b , `b all are scalars.


th th
i.e. (i, j) element of B=(i,j) element of C, for all values of i and j.
As such B = C.
Hence the relation, “A+B = A+C” holds if and only if B = C.

1.4.1 Addition of Matrices: let A = [ `b ] and B = [“`b ] be are two m×n matrices. As the
sum A + B is defined to be the matrix C = [ `b ] with `b = `b + “`b .

We define the sum of two matrices only when the order of the two matrices is same.

4 2 5 1 0 2
Thus if A = º »,) = º »
1 3 −6 3 1 4

4+1 2+0 5+2 5 2 7


A+B = º »=º ».
1 + 3 3 + 1 −6 + 4 4 4 −2

1.4.2 Subtraction of Matrices:- let A = [ `b ] and B = [“`b ] be are two m×n matrices. Then the
difference A - B is defined to be the matrix C = [ `b ] with `b = `b − “`b .

4 7 8 1 2 5
Thus if A= º »,) = º »,
5 3 6 3 1 4
4−1 7−2 8−5 3 5 3
A-B = º »=º ».
5−3 3−1 6−4 2 2 2

1.4.3 Multiplying a Scalar to a Matrix:- Let A = [ `b ] be an m × n matrix. Then for any


element k ∈ R, we define kA = [k `b ].
1 4 5
»
0 1 2
5 20 25
For example, if A =º

».
0 5 10
and k = 5, then 5A =º

1.4.4 Multiplication of Matrices: The multiplication of two matrices A and B is only possible if
the number of columns in A is equal to the number of rows in B.
Let A = [ `b ] be an m × n matrix and B = [“`b ] bean n × r matrix. Then the product AB is a
matrix C = [ `b ] of order m× r, with
cØô = aØ6 b6ô + aØ/ b/ô + aØ_ b_ô + ⋯ + aØ" b"ô.

1.4.4.1 Properties of matrix multiplication: Suppose that the matrices A, B and C are so
chosen that the matrix multiplications are defined.
1. Then (AB)C = A(BC). That is, the matrix multiplication is associative.
2. For any k ∈ R, (kA)B = k(AB) = A(kB).
3. Then A(B + C) = AB + AC. That is, multiplication distributes over addition.
4. If A is an n × n matrix then AI= IA = A, where I is identity matrix.

69
MSCPH501

1.4.4.2 Determinant of a matrix: In linear algebra, the determinant is a value that can be
computed from the elements of a square matrix. The determinant of a matrix A is denoted
det(A), det A, or |A|. It can be viewed as the scaling factor of the transformation described by the
matrix.


In the case of a 2 × 2 matrix the specific formula for the determinant is:
| |=' '= ad – bc.
N

Similarly, suppose we have a 3 × 3 matrix A, and we want the specific formula for its
determinant |A|:


Q N Q N
| | = ±N Q±= ' '−“( (+ ( ( = aei-afh-bdi+bfg+cdh-ceg.
ℎ t t ℎ
t ℎ

Each determinant of a 2 × 2 matrix in this equation is called a "minor" of the matrix A. The
same sort of procedure can be used to find the determinant of a 4 × 4 matrix, the determinant of a
5 × 5 matrix, and so forth.

1.5 Important Properties of Determinants

a. The value of a determinant is not altered if its rows are written as columns in the same order.

3 1 4 3 6 7
±6 2 1± = ±1 2 0±.
7 0 5 4 1 5
b. If any two rows (or two columns) of a determinant are interchanged, the value of the
determinant is multiplied by –1.

3 1 4 6 2 1
±6 2 1± = − ±3 1 4±.
7 0 5 7 0 5

3 8 1 3 2 1
c. A common factor of all elements of any row (or column) can be placed before the determinant.

±5 4 2± = 4 ±5 1 2±.
1 12 3 1 3 3

d. If each element of a row (or a column) of a determinant can be expressed as a sum of two, the
determinant can be written as the sum of two determinants.

3 1 4 −1 + 4 1 4 −1 1 4 4 1 4
±6 2 1± = ± 3 + 3 2 1± = ± 3 2 1± + ±3 2 1±.
7 0 5 5+2 0 5 5 0 5 2 0 5

1.6 Minor of a matrix: The determinant corresponding to any r× submatrix of an m× 4


matrix A is called a minor of order r of the matrix A. For example, if
70
MSCPH501

1 2 3 1 2 2 3 1 3
A=º » then its minorsare ' ',' ',' '.
4 5 6 4 5 5 6 4 6

element `b is denoted by R`b and is defined to be the determinant of the submatrix that remains
1.7 Cofactors of a matrix: If A is a square matrix, (3 × 3) for example, then the minor of

The number (−1)` b R`b is denotd by p`b and is called the cofactor of element `b .
after the ith row and jth column are deleted from A.

3 1 −4
Example, Let A = 2 5 6 .
1 4 8

The minor of element 6/ is

2 6
R6/ = ' '= 2(8) − 6(1) = 10.
1 8

Then the cofactor of 6/ is

p6/ == (−1)6 / R6/ = (−1) × 10 = −10.

1.8 Adjoint of a matrix:-Let A=j `b l be a square matrix of order n and let `b denote the
cofactor of `b in the determinant| |. The transpose of the matrix [ `b l is, then defined as the
adjoint of A and is denoted by Adj (A).

66 6/ ⋯ 6©
// ⋯
Thus, if A= ² ⋮ ⋮ ³
/6 /©

©6 ©/ ©©

66 6/ … 6©
// ⋯
Then „ `b … =² /6 /©
³
… ⋯ ⋯ ⋯
6© /© ⋯ ©©

66 ⋯/6 ©6
// ⋯
Adj A = „ `b … =² /6 ©/
³.
⋯ ⋯ ⋯ ⋯

6© /© ⋯ ©©

1.9 Properties of adjoint:-If A=j `b l is a square matrix of order n, then


(i) adj A‫( =׳‬adj A)‫׳‬
(ii) adj A*= (adj A)‫׳‬
(iii) Adjoint of a symmetric (Hermitian) matrix is symmetric (Hermitian).
(iv) The adjoint of the product of square matrices is the product of their adjoint matrices
taken in reverse order i.e., adj(AB) = adjB.adj A.

71
MSCPH501

1.10 The inverse or reciprocal of a matrix:-


Let A be a square matrix of order n. If there exists a square matrix B of the same order n, such
that
AB = BA = I, where I is identity matrix.
Then B is called inverse of A and is denoted by 56 .
Thus, 56
= 56 = %.

We know that A (adj A) =| |=I

( NP )
= %.
| |

= = %].
56 ˜•b 56
| |
Hence, [since,

1.11 The Rank of a Matrix:


The maximum number of linearly independent rows in a matrix A is called the row rank of A,
and the maximum number of linearly independent columns in A is called the column rank of A.
If A is an m× n matrix, or
A matrix ‘A’ is said to be of rank r, if and only if:
(i) There exist at least one non-zero minor of order r.
(ii) Every minor of order (r+1) and higher, vanishes.

1.12 Normal Form (Canonical Form): By performing elementary transformation, any non-zero
matrix A can be reduced to one of the following four forms, called the normal form of A:

% % 0
( )%• ( )j%• 0l( ) º • » ( ) º • ».
0 0 0

The number r so obtained is called the rank of A and we write ρ (A) = r.

1 2 3 1
Ex. Find the rank of A= 2 4 6 2 .
1 2 3 2

Sol. Since rank of a matrix is not altered by elementary operation, therefore, we have

1 2 3 1
@~ 0 0 0 0 by+/ = +/ − 2+6 and +_ = +_ − +6
0 0 0 1
1 2 3 1
~ 0 0 0 1 by +/ = +/ + +_ 4N ℎ 4 +_ = +_ − +6 .
0 0 00

72
MSCPH501

3 1
No. of non-zero rows are 2, which shows that every minor of 3rd order is zero, while a minor of
'. Hence rank of A is 2.
0 1
second order i.e.,'

1 2 −1 3
4 1 2 1
=² ³
3 −1 1 2
Ex. Reduce the matrix A to its normal form, where
1 2 0 1
and hence find the rank of A.

1 2 −1 3
4 1 2 1
Sol. A~ ² ³
3 −1 1 2
1 2 0 1

1 2 −1 3
0 −7 6 −11
A~ ² ³ j+/ = +/ − 4+6 ; +_ = +_ − 3+6 ; +c = +c − +6
0 −7 4 −7
0 0 1 −2

1 0 0 0 1 0 0 0
0 −7 6 −11 0 −7 6 −11
~² ³ ~² ³ j+_ = +_ − +/ l
0 −7 4 −7 0 0 −2 4
0 0 1 −2 0 0 1 −2

p/ = p/ − 2p6 , p_ = p_ + p6 , pc = pc − 3p6

1 0 0 0 1 0 0 0
0 −7 0 0 0 −7 0 0
~² ³~² ³
0 0 −2 4 0 0 −2 4
0 0 1 −2 0 0 0 0

6 11
= p_ + p/ ; pc = pc − p/
7 7

1 0 0 0 1 0 0 0
0 −7 0 0 0 1 0 0 1 1
pc = pc + 2p_ ~ ² ³~² ³ æ+/ = − +/ ; +_ = − +_ ç.
0 0 −2 0 0 0 1 0 7 2
0 0 0 0 0 0 0 0

Hence, Rank of A = 3.

1.13 Eigen Values:-Let A is a matrix of order 3× 3, X is a column vector and Y is also a


column vector.

73
MSCPH501

66 6/ 6_ I6 J6
/6 // /_ I/ = J/
_6 _/ __ I_ J_

AX = Y. ……. (1)

Here, column vector X is transformed into the column vector Y by means of the square matrix
A.
Let X is a such vector which transforms into λX by means of the transformation (1). Suppose
the linear transformation Y=AX transforms X into a scalar multiple of itself i.e. λX.

AX = Y = λX
AX-λIX = 0
(A-λI)X = 0. ……. (2)

Thus, the unknown scalar λ is known as an Eigen value of the matrix A and the corresponding
non- zero vector X as Eigen vector.
Eigen values are also called characteristic values or proper values or latent values.

2 2 1
= 1 3 1
1 2 2
Let

2 2 1 1 0 0 2−λ 2 1
− λI = 1 3 1 − λ 0 1 0 = 1 3−λ 1 .
1 2 2 0 0 1 1 2 2−λ

(a) Characteristic Polynomial: The determinant | − λI| when expanded will give a
polynomial, which is called characteristic polynomial of matrix A.

2−λ 2 1
For example; ± 1 3−λ 1 ± = (2 − λ)Y6 − 5λ + λ − 2Z − 2(2 − λ − 1) +
/

1 2 2−λ
1(2 − 3 + λ)

= −λ_ + 7λ/ − 11λ + 5.

(b) Characteristic equation:-The equation | − λI| = 0 is called the characteristic equation of

λ_ − 7λ/ + 11λ − 5 = 0.
the matrix A e.g.

(c) Characteristic roots or Eigen values:- The roots of characteristic equation | − λI|=0 are
called characteristic roots of matrix A. e.g.

λ_ − 7λ/ + 11λ − 5 = 0

(λ − 1)(λ − 1)(λ − 5) = 0
Eigen values are λ = 1,1,5.

74
MSCPH501

1.14 CAYLEY- HAMILTON THEOREM:

Statement- A square matrix satisfies its own characteristic equation i.e., if A is an n×m matrix
whose characteristic equation is

+© + p6 +©56 + p/ +©5/ + ⋯ p© %© = 0.

Putting λ=A in the above equation, we have


©
+ p6 ©56
+ p/ ©5/
+ ⋯ p© %© = 0.

1 4
» 4N h 4N 56
.
2 3
Ex. Verify Cayley-Hamilton theorem for A=º

| − +%| = 0
Sol. we know that

1 4 1 0 1−+ 4
'º »−+º »=º » = 0'
2 3 0 1 2 3−+

(1 − +)(3 − +) − (4)(2) = 0

A(+) = +/ − 4+ − 5% = 0.

Replace λ with A

P(A)= /
− 4 − 5% = 0 ………(1)

1 4 1 4 1 + 8 4 + 12 9 16
/
=º »º »=º »=º ».
2 3 2 3 2+6 8+9 8 17

9 16 1 4 1 0
Now from (1)
A( ) = º » − 4º » − 5º »
8 17 2 3 0 1

9 − 4 − 5 16 − 16 − 0 0 0
=º »=º ».
8 − 8 − 0 17 − 12 − 5 0 0

Cayley-Hamilton theorem is verified.

| | = 3 − 8 = −5 ≠ 0 ℎ 4 56
I .
56
To find inverse, multiplying eq. (1) with , we get
56 ( /
− 4 − 5%) = 56 /
−4 56
− 5% 56
=0

= −4I-5 56
=0

75
MSCPH501

− 4% = 5 56
= ( − 4%)
56 6
š
6 1 4 4 0
56
= š (º »−º »)
2 3 0 4
−3 4
= º ».
56 6
š 2 −1
Ans.

7 2 −2
Ex. Verify Cayley-Hamilton theorem for A= −6 −1 2 .
6 2 −1
Sol.

7 2 −2 1 0 0
| − +%| = −6 −1 2 −+ 0 1 0 =0
6 2 −1 0 0 1

7−+ 2 −2
= −6 −1 − + 2 =0
6 2 −1 − +

(7 − +)j(−1 − +)(−1 − +) − 4l − 2j−6(−1 − +) − 12l − 2j−12 − 6(−1 − +)l


=0

(7 − +)j+/ + 2+ − 3l − 2j6+ − 6l + 2j6+ − 6l = 0

7(+/ + 2+ − 3) − +(+/ + 2+ − 3) = 0

−+_ + 5+/ − 7+ + 3 = 0

Or +_ − 5+/ + 7+ − 3 = 0.

Now replace + by A
_
−5 /
+ 7 − 3% = 0 ………… (1)

7 2 −2 7 2 −2
/
= −6 −1 2 −6 −1 2
6 2 −1 6 2 −1

25 8 −8
/
= −24 −7 8
24 8 −7
_
= /

76
MSCPH501

25 8 −8 7 2 −2
_
= −24 −7 8 −6 −1 2
24 8 −7 6 2 −1

79 26 −26
_
= −78 −25 26 .
78 26 −25

Now from (1), we have

79 26 −26 25 8 −8 7 2 −2 1 0 0
−78 −25 26 − 5 −6 −1 2 + 7 −6 −1 2 − 3 0 1 0
78 26 −25 6 2 −1 6 2 −1 0 0 1

79 − 125 + 49 − 3 26 − 40 + 14 −26 + 40 − 14
−78 + 120 − 42 −25 + 35 − 7 − 3 26 − 40 + 14 = 0.
78 − 120 + 42 26 − 40 + 14 −25 + 35 − 7

Hence cayley-hamilton theorem is verified.

1.15 Summary: Hence this chapter deals with the matrices and its properties. Matrix is a
rectangular array of elements, which are very helpful to deal with several variables at once. We
can perform number of operations by organizing the elements in terms of rectangular arrays of
numbers. Then we have found that matrices themselves can under certain conditions be added,
subtracted and multiplied hence they will follow the set of algebraic rules. Another operation on
the matrices is transpose by just reversing the transpose and columns. In another section we have
discussed the various types of matrices like unit matrix, zero matrix, diagonal matrix etc.
Matrices find many applications in scientific fields and apply to practical real life problems as
well, thus making an essential concept for solving many practical problems.

1.16 References:

1. B. S. Rajput, Mathematical Physics, Pragati Prakashan.


2. H.K. Das, Rama Verma. Mathematical Physics, S. Chand.
3. Mary L. Boas, Mathematical Methods in the Physical Science, Wiley; 3rd edition (16
August 2005)
4. Eugene Butkov, Mathematical Physics, Addison-Wesley Pub. Co., 01-Jan-1968

77
MSCPH501

1.17 EXERCISE

1. Write the minors and cofactors of each element of the following determinants and also

42 1 6 1 “
evaluate the determinant in each case:

±28 7 4± (ii) ±1 “ ±.
14 3 2 1 “
(i)

2 1 −1 2
2. Matrices A and B are such that
» and -4A+B=º »
−2 −1 −4 3
3A-2B=º

0 −1 −1 −2
=º », ) = º »
2 −1 4 −1
Find A and B. Ans:

0 1
» choose α and β so that(-% + ä )/ = .
6
−1 0 √/
3. If A=º Ans: α=β=±
1 −2 1
4. (i) Show that the matrix a −2 4 −2 is idempotent.
6

1 −2 1

(1 + )© = 1 + (2© − 1) .
(ii) Show that if A is idempotent, then

6 1+ −1
5. Prove that / º » is unitary.
1+ 1−

6. Show that adj( %© ) = ©56


%© , where k is a scalar.

1 0 −1 2 6 4
7. Find the adjoint and then inverse of the matrix

A= 3 4 5 ,-.: /g 21 −7 −8 .
6

0 −6 −7 −18 6 4

1 2 3 4 1 1 2 2 4 3 −2
8. Reduce the following matrices into normal form and find the Rank:

3 4 1 2 ( ) 1 2 2 (iii) −3 −2 −1 4
4 3 1 2 2 2 3 6 −1 7 2
(i)

,-.: (i) 3 (ii) 3 (iii) 3

4 2 −2
= −5 3 2 .
−2 4 1
9. Find the eigen values of the matrix Ans:λ= 1,2,5

10. Use Cayley-Hamilton Theorem to find the inverse of the matrix

78
MSCPH501

cos , sin , cos θ −sin θ


º » »
− sin , cos , sin θ cos θ
Ans:º

Choose the correct alternative:

1. Transpose of a rectangular matrix is a


(i) rectangular matrix
(ii) diagonal matrix
(iii) square matrix
(iv) scalar matrix.

2. Additive inverse of a matrix A is


˜•b( )
| |
(i) (ii) A²

| |
(iv) A
(ii)

3. The number of non-zero rows in an echelon form is called?


( i) rank of a matrix

ii (iii) cofactor of the matrix

iii (iv) reduced echelon form

iv (v) conjugate of the matrix.

1 2 3
4. Rank of the matrix A= 1 4 2 is
2 6 5
(i) 0 (ii) 1 (iii) 3 (iv) 2.

5. Which of the following matrices are Hermitian:

1 2+ 3− 2 3 1 4 2− 5+2
2+ 2 4− ( ) 4 −1 6 ( ) 2 + 1 2−5
3+ 4+ 3 3 7 2 5−2 2+5 2
(i)

0 3
(iv) −7 0 5 .
3 1 0

6. If λ is an Eigen value of the matrix M then for the matrix (M-λI), which of the following
statement is correct?
(i) Skew-symmetric (ii) Non singular (iii) Singular (iv) None of these.

79
MSCPH501

= ( ) = %.
7. A square matrix is idempotent if:
/ /
(i) A’=A (ii) A’= -A (iii)

8. If A and B are matrices, then which from the following is true?

(i) AB ≠ BA

(ii) (A’)’≠ A

(iii) A+B≠B+A

(iv) all are true.


9. Two matrices A and B are multiplied to get BA if

(i) no of rows of A is equal to no. of columns of B

(ii) no of columns of A is equal to columns of B

(iii) both have same order

(iv) both are rectangular.

10. A matrix having m rows and n columns with m ≠ n is said to be a

(i) scalar matrix

(ii) identity matrix

(iii) square matrix

(iv) rectangular matrix.

Ans: (1) (i) (2) (i) (3) (i) (4) (iv) (5) (iii) (6) (iii) (7) (iii) (8) (i) 9 (ii)
10(iv)

80
MSCPH501

UNIT 3: Complex Analysis

STRUCTURE:
3.0 Objectives
3.1 Introduction
3.2 Definitions
3.2.1 Complex Numbers
3.2.2 Equality of complex numbers
3.2.3 Modulus and Argument of a complex number
3.3 Operation of fundamental laws of Algebra on complex numbers
3.3.1 Addition
3.3.2 Subtraction
3.3.3 Multiplication
3.3.4 Conjugate complex numbers
3.3.5 Modulus properties
3.4 Function of a complex variable
3.5 Set of points
3.6 Neighborhood of a point
3.7 Limit point of a set
3.8 Domain
3.9 Analytic function
3.10 Cauchy-Riemann Equation
3.11 Harmonic Function
3.12 Polar Form of Cauchy-Riemann Equation
3.13 Cauchy Integral Formulas
3.14 Taylor Series
3.15 Laurent Series
3.16 Singularities
3.17 Types of Singularities
3.17.1 Isolated Singularities
3.17.2 Removable Singularities
3.17.3 Poles
3.17.4 Essential Singularities
3.18 Residues
3.19 Cauchy Residue Theorems
3.20 Evaluation of residues
3.21 Evaluation of Integrals
3.22 Summary
3.23 References
3.24 Exercise

81
MSCPH501

3.0 Objectives

After studying this unit, you should be able to-

• Knowledge on Complex Numbers


• Knowledge on operation of fundamental laws of algebra on complex numbers
• Function of a complex variable
• Types of Singularities and Residues
• Evaluation of residues and Integrals

3.1 INTRODUCTION

Cantor, Dedekind and Weierstrass etc., extended the concept of rational numbers to a larger field
known as real numbers which constitute rational as well as irrational numbers. But, the number

number, rational or irrational, which satisfies the equation I / +1 = 0. It was, therefore, felt
system solely based on real numbers is not sufficient for all mathematical needs. There is no real

necessary by Euler Gauss, Hamilton, Cauchy, Riemann and Weierstrass etc. to extend the field
of real numbers to the still large field of complex numbers. Euler for the first time introduced the
symbol i with the property / =-1 and then Gauss introduced a number of the form α+iβ, which
satisfies every algebraic equation with real coefficients. Such a number α+iβ with i= √-1 and α, β
being real, is known as a complex number.

3.2 DEFINITIONS

3.2.1 Complex Numbers: “An ordered pair of real numbers such as (x, y) is termed as a
complex number.” If we write

z= (x, y) or x+iy, where i= √-1, then

x is called the real part of z and y is called the imaginary part of the complex number z and
denoted by,

x= +> or R (z) or Re (z)

y= %> or I (z) or Im (z).

3.2.2 Equality of complex numbers: Two complex numbers (x,y) and (x´,y´) are equal if
x=x´ and y=y´.

82
MSCPH501

3.2.3 Modulus and Argument of a complex number: If z=x+iy be a complex number


then, If we introduce polar co-ordinates (r, θ), we have x = r cosθ and y = r sin θ and then
from equation z = r (cosθ + i sin θ ) = re iθ .Here, r is the modulus of the complex number
y
x+iy and is denoted by x + iy or arg (z).Argument of z; θ = tan −1 ( )
x

3.3 OPERATION OF FUNDAMENTAL LAWS OF ALGEBRA ON COMPLEX


NUMBERS

Taking three complex numbers K6 = (I6 ,J6 ), K/ = (I/ ,J/ ), K_ = (I_, J_ ) we define the
following operations:

3.2.1 Addition: The sum of two complex numbers z6 = (x6 ,y6 ), z/ = (x/ ,y/ ) is defined as
a complex number z = (z1+z2) = (x1+x2, y1+y2) such that its real part is the sum of
real parts and imaginary part is the sum of imaginary parts of the given numbers.

3.3.1 Subtraction:

If z1=(x1, y1) and z2=(x2, y2), then


z1-z2 =(x1-x2, y1-y2).
Multiplication: we have z1z2=(x1+iy1)(x2+iy2)
i.e., z1z2=(x1x2-y1y2, x1y2+x2y1)
3.3.2 Conjugate complex numbers :
If z= x+iy, then its conjugate complex number is z = x − iy
Evidently, z1 + z 2 = z1 + z 2
z1 z 2 = z1 z 2
2
z z = ( x + iy )( x − iy ) = x 2 + y 2 = z
.

3.3.3 Modulus properties:

z1 + z 2 ≤ z1 + z 2
z1 + z 2 ≥ z1 − z 2
z1 z
= 1
z2 z2
3.4 Function of a complex variable: All the elementary functions of real variables may
be extended into the complex plane replacing the real variable x by the complex variable
z. Before giving a formula definition of functions of a complex variable, let us define
some useful terms.

83
MSCPH501

3.5 Set of points: The set of points in Argand diagram is a collection of points finite or
infinite in number.

points such that |z − a| < є, where є, is an arbitrary chosen small positive number, is
3.6 Neighborhood of a point: Let ‘a’ be a point in the Argand diagram. A set of all the z

defined as neighborhood of point ‘a’.

3.7 Limit point of a set: A point ‘a’ every neighborhood of which contains a point of set

example, each point on the circumference of circle|K| = is a limit point of set|K| < .
S other than ‘a’ is defined as the limit point of the set S of points in the Argand plane. For

These points do not belong to the set. But each point inside the circle is also a limit point
that belongs to the set. Thus the limit point of a set may not necessarily be the point of the
set. If ‘a’ is a limit point of the set S such that in the neighborhood of ‘a’, there exist
entirely the point of the set S, it is defined as interior or inner point. If all the points in the
neighborhood do not belong to the set S, it is said to be the boundary limit point.
A set is said to be closed if all its limit points (inner or boundary points) belong to the set.
If a set consists of entirely the interior points, it is known to be an open set.

3.8 Domain: If every pair of points of a set of points in Argand diagram can be
connected by a polygonal are every point of which is the point of the set then the set is
said to be domain or region. Open domain is open connected set of points. When the
boundary points of the set are also added to an open domain, it becomes a closed domain.
We may now give a formula definition of a function of complex variables. Let x and y be
a pair of real variables such that z= x+iy, and let u and v be a pair of real functions such
that w=u+iv, then w is said to be the function of complex variable z and written as
w=f(z), if to every value of z in a certain domain D, there correspond one or more definite
value of w. In case w has only one value for each value of z in the given domain D, w is
said to be uniform or single valued function of z and if it takes more than one value for
some or all value of z in D, then w is known as a many valued or multiple valued
function of z. thus the function w=u+iv of complex variable z=x+iy is ordered pair of real
functions of real variable,

i.e. w = f(z) = u(x,y) + iv(x,y).

3.9 Analytic function: A function f(z) is said to be analytic at a point z=a, if f(z)is
differentiable not only at ‘a’ but at every point of some neighborhood of ‘a’. A function
f(z) is analytic in a domain if it is analytic at every point of the domain D of the function.
The points at which the function is not differentiable are called singular points or a
singularity of the function.An analytic function is also known as “holomorphic”,
“regular”, and “monogenic”.

84
MSCPH501

3.10 Cauchy-Riemann equation


A necessary condition for a function f(z) such that w= f(z)= u(x,y)+ iv(x,y) to be analytic
in domain D is that u and v satisfy Cauchy- Riemann equation given by
∂u ∂v ∂u ∂v
= and =−
∂x ∂y ∂y ∂x
Or ux=vy and uy=-vx

These two equations are called the Cauchy- Riemann differential equations.

3.11 Harmonic Function: A function u(x, y) is called harmonic function if its first and
second order partial derivatives are continuous and it satisfy Laplace equation

2 ∂ 2v ∂ 2v
i.e., ∇ u = 2 + 2 = 0
∂x ∂y

3.12 Polar form of Cauchy-Riemann equation- If f(z)= u+iv is an analytic function and
z= r `Ü then the Cauchy-Riemann equations are given by-

∂u 1 ∂v ∂v 1 ∂u
= and =−
∂r r ∂θ ∂r r ∂θ

Proof:-

∂u ∂v ∂u ∂v
Let f (z) = u+iv is an analytic function, so = and =− ….(1)
∂x ∂y ∂y ∂x

y
For polar co-ordinate system, we know that- x= r cosθ, y = r sinθ and θ = tan −1 ( )
x

r 2 = x2 + y2

r = ( x 2 + y 2 )1 / 2

∂r x ∂r y
= = cos θ , = = sin θ
∂x r ∂y r

∂θ y r sin θ sin θ
=− 2 2
=− 2 =−
∂x x +y r r

∂θ x r cos θ cos θ
= 2 2
= =
∂y x + y r2 r

85
MSCPH501

∂u ∂u ∂r ∂u ∂θ ∂u sin θ ∂u
= + = cosθ − ….(2)
∂x ∂r ∂x ∂θ ∂x ∂r r ∂θ
∂u ∂u cos θ ∂u
Similarly, = sin θ + ’ …(3)
∂y ∂r r ∂θ

∂v ∂v sin θ ∂v
= cosθ − …(4)
∂x ∂r r ∂θ
∂v ∂v cos θ ∂v
= sin θ + …(5)
∂y ∂r r ∂θ

Substituting these values in equation (1), we get


∂u sin θ ∂u ∂v cosθ ∂v
cosθ − = sinθ + … (6)
∂r r ∂θ ∂r r ∂θ

∂u cosθ ∂u ∂v sin θ ∂v
sinθ + = -cosθ + ... (7)
∂r r ∂θ ∂r r ∂θ
∂u 1 ∂v
Multiplying (6) by cosθ, (7) by sinθ and adding, we get =
∂r r ∂θ
∂u ∂v
Again multiplying (6) by sinθ and (7) by cosθ and subtracting, we get = −r
∂θ ∂r
Hence polar form of Cauchy- Riemann equations are

∂u 1 ∂v ∂u ∂v
= = −r
∂r r ∂θ , ∂θ ∂r

3.13 CAUCHY INTEGRAL FORMULA:

If f (z) is analytic within and on a closed contour c and ‘a’ is any point within c.
Then,
1 f ( z)
f (a ) = ∫
2πi z − a
dz

Proof- Let z=a , is a point within a closed contour c. Draw a circle γ, with centre at the
point z=a and radius ρ such that it lies entirely within c.
f ( z)
Consider a function ϕ ( z ) = is analytic in region between γ and c.
z−a

86
MSCPH501

As we know- ∫ ϕ ( z )dz = ∫ ϕ ( z )dz


c γ

f ( z) f ( z)
∫ z − a dz = ∫γ z − a dz
c

f ( z) f ( z)
∫ z − a dz = ∫γ z − a dz
c

f ( z) f ( z ) + f (a) − f (a )
∫ z − a dz = ∫γ
c z−a
dz

f ( z) f ( z ) − f (a) f (a)
∫ z − a dz = ∫γ
c z −a
dz + ∫
γ z−a
dz ...(1)

Now, equation of circle, z−a =ρ

z − a = ρe iθ (Since, e iθ =1)
dz = iρe iθ dθ and 0≤θ≤2π


f ( z) f (a)
∫γ z − a ∫0 ρe iθ iρe dθ

So, dz =

87
MSCPH501


= ∫ if ( a ) dθ = 2πif ( a )
o

Putting this value in equation (1)

f ( z) f ( z ) − f (a )
∫ z − a dz = ∫γ
c z−a
dz + 2πif (a)

f ( z) f ( z ) − f ( a)
∫c z − a dz − 2πif (a) = ∫γ z − a dz

Taking modulus on both sides

f ( z) f ( z ) − f (a ) f ( z ) − f (a )
∫c z − a dz − 2πif (a) = ∫γ dz ≤ ∫ dz
z−a γ z−a

Now by the definition of continuity,


f ( z ) − f ( a ) 〈ε , z − a = ρ and ∫γ dz = perimeter = 2πρ
Hence,
f ( z)
∫ z − a dz − 2πif (a) 〈 2πε
c

Making є → 0, we get
f ( z)
∫ z − a dz − 2πif (a) 〈0
c

f ( z)
∫ z − a dz − 2πif (a) = 0
c

f ( z)
dz = 2πif (a )
z−a
1 f ( z)
f (a) = ∫
2πi c z − a
dz

This is Cauchy integral formula.

3.14 TAYLOR SERIES: If a function f(z) is analytic at all points inside a circle C, with
its centre at the point a and radius R, then at each point z inside C.

88
MSCPH501

f '' ( z − a ) 2 f n (a)
f ( z ) = f (a) + f ' (a)( z − a) + + ............................... ( z − a) n + ......
2! n!

PROOF: Take any point z inside C. Draw a circle p6 with centre a, enclosing the point
z. Let w be a point on circlep6 .

1 1 1
= =
w − z w − a + a − z w − a − ( z − a)

−1
1 1 1  z−a
= = 1 − 
( w − a) z−a w−a w−a
(1 − )
w−a
Applying binomial theorem

2 n
1 1  z−a  z−a  z−a 
= 1 + +   + .......... .. +   + .....
w − z w − a  w − a  w − a  w−a 

1 1 ( z − a) ( z − a) 2 ( z − a) n
= + + + ....... + + ........... …… (1)
w − z w − a ( w − a) 2 ( w − a) 3 ( w − a) n +1

z−a
As z − a < w − a → <1
w−a

so the series converges uniformly. Hence the series is integrable.

Multiplying eq.(1) by f(w).

89
MSCPH501

f ( w) f ( w) ( z − a) f ( w) ( z − a) 2 f ( w) ( z − a) n f ( w)
= + + + ....... + + ....
w− z w−a ( w − a) 2 ( w − a) 3 ( w − a) n +1

On integrating with respect to “w” we get

f (w) f (w) f (w) f (w) f (w)


∫ w− z dw= ∫ w−a dw+(z −a)∫ (w−a)
c1 c1
2
dw+ (z −a)2 ∫
c1 (w− a)
3
dw+.....+(z −a)n ∫
c1 (w− a)
n+1
dw

We know that,

f ( w) f ( w)
∫ w − z dz = 2πif ( z ) and ∫ w − a dz = 2πif (a)
c1 c1

f ( w)
∫ ( w − a)
c1
2
dz = 2πif ' (a) and so on.

Substituting these values in (2) we get


f '' ( z − a) 2 f n (a)
'
f ( z) = f (a) + f (a)(z − a) + + ............................... ( z − a) n + ......
2! n!

This is Taylor’s series.

Examples
/> & 6
>& >
Que: Find Taylor expansion of f(z)= about the point z=1.
Ans:-

2z 3 + 1
f ( z) = , singularities are given by z=0, -1
z ( z + 1)

If centre of the circle is at z=1, then the distance of the singularities z=0 and z=-1 from
the centre are 1 and 2. Hence, if a circle is drawn with centre z=1 and radius 1, then

90
MSCPH501

within the circle z − 1 = 1 , the given function f(z) is analytic and therefore, it can be
expanded in a Taylor series within the circle z − 1 = 1 .

2z 3 + 1 1 1
= 2z − 2 + +
z ( z + 1) z +1 z
1 1
= 2z − 2 + + [ z − 1 ] < 1]
z −1+ 2 z −1+1

−1
1 z −1 −1
= 2 z − 2 + 1 +  + [1 + ( z − 1)]
2 2 

1   z −1  z −1  z −1 


2 3

= 2z − 2 + 1 −   +   −   + .... + [1 − ( z − 1) + ( z − 1) 2 − ( z − 1) 3 + .....]
2   2   2   2  

3 3  z −1 9 2 17 3
= 2z − 2 + −   + ( z − 1) − ( z − 1) + ....
2 2 2  8 16

Which is required expansion.

Que: Expand cos z in a Taylor series about z=Ù/4.


Sol. Heref(z)= cos z, f‫(׳‬z)= -sin z, f‫(׳׳‬z)= -cos z, f‫(׳׳׳‬z)= sin z, …..

Here cos z= f(z)


2 3
 π  π
z −  z − 
π   π  π  4 π  4 π 
= f   +  z −  f '  +  f ''  +  f ' ' '   + ...
4  4 4 2! 4 3! 4

1   
2 3
π  1 π 1 π
= 1 −  z −  −  z −  +  z −  + ...
2   4  2!  4 3!  4 

Which is required expansion.

 sin z 
Que: Expand the function   about z = π .
 z −π 
Sol. - Putting z-π= t, we have

91
MSCPH501

sin z sin(π + t ) sin t


= =−
z −π t t
1 t3 t5  t3 t5
= −  t − + − ...... = −1 + − + ...
t 3! 5!  3! 5!
(z − π )3 ( z − π )5
= −1 + − + ...
3! 5!
Which is required expansion.

3.15 LAURENT’S SERIES: If we are required to expand f(z) about a point where f(z)
is not analytic, then it is expanded by Laurent’s series and not by Taylor’s series.

Statement: If f(z) is analytic on c1 and c2 and the annular region R bounded by the two
concentric circles c1 and c2 of radii r1 and r2 (r2 < r1 ) and with centre at a, then for all z in
R.

b1 b2 bn
f ( z ) = a 0 + a1 ( z − a ) + a 2 ( z − a) 2 + ....a n ( z − a) n + + + ... +
z − a ( z − a) 2
( z − a) n
1 f ( w)
Where a n = ∫
2πi c1 ( w − a) n +1
dw,

1 f ( w)
bn = ∫
2πi c2 ( w − a) −n +1
dw

92
MSCPH501

Proof: By introducing a cross cut AB, multi-connected region R is converted to a simply


connected region. Now f(z) is analytic in this region.
Now by Cauchy’s integral formula

1 f ( w) 1 f ( w) 1 f ( w) 1 f ( w)
f ( z) = ∫
2πi c1 w − z
dw + ∫
2πi AB w − z
dw − ∫
2πi c2 w − z
dw + ∫
2πi BA w − z
dw

Integral along c2 is clockwise, so it is negative. Integrals along AB and BA cancel.

1 f ( w) 1 f ( w)
f ( z) = ∫
2πi c1 w − z
dw − ∫
2πi c2 w − z
dw ….(1)

can be expanded exactly as in Taylor’s series as z lies on c1 .


f ( w)
for the first integral,
w− z

1 f ( w) 1 f ( w) z−a f ( w) ( z − a) 2 f ( w)
2πi c∫1 w − z 2πi c∫1 w − a 2πi c∫1 ( w − a) 2 ∫ ( w − a)
dw = dw + dw + dw + ...
2πi c1
3

= a 0 + a1 ( z − a ) + a 2 ( z − a ) 2 + ...... …..(2)

 1 f ( w) 
2πi c∫1 ( w − a) n +1 
a n = dw


In the second integral, z lies on c2 . Therefore


w−a
w − a < z − a or <1
z−a

1 1 1
So here = =
w − z w − a + a − z (w − a) − ( z − a )

−1
1 1 1  w−a
=− =− 1 − 
z−a  w−a z−a z−a 
1 − 
 z−a 

Using binomial expansion

93
MSCPH501

1  w−a w−a
n +1
2
w−a 
=− 1 + +  + ... +   + ...
z − a  z − a  z − a   z−a  

f ( w)
Multiplying by − , we get
2πi

1 f ( w) 1 f ( w) 1 ( w − a ) f ( w) 1 (w − a) 2
− = + + f ( w) + ...
2πi w − z 2πi z − a 2πi ( z − a ) 2 2πi ( z − a) 3

1 1 1 1 f ( w) 1 1 f ( w)
= f ( w) + + + .....
( z − a ) 2πi 2πi ( z − a ) ( w − a )
2 −1
2πi ( z − a ) ( w − a ) − 2
3

Integrating, we have

1 f (w)  1  1 1 1 f (w) 1 1 f (w)


− ∫
2πi c2 w− z
dw=   ∫ f (w)dw+
 z − a  2πi c2
2 ∫
2πi (z − a) c2 (w− a) −1
dw+ 3 ∫
2πi (z − a) c2 (w− a)−2
dw+...

b1 b2 b3  1 f ( w) 
2πi c∫2 ( w − a) −n +1 
= + 2
+ + ... ..…. (3) bn = dw 
z − a ( z − a) ( z − a) 3 

Substituting the values of values of both integrals from (2) and (3) in (1), we get

f ( z ) = a 0 + a1 ( z − a) + a 2 ( z − a) 2 + .... + b1 ( z − a) −1 + b2 ( z − a) −2 + ...
n =∞ n =∞
bn
f ( z ) = ∑ a n ( z − a) n + ∑ n
n =0 n =1 ( z − a )

This is Laurent Theorem.

1
Que: Expand f ( z ) = for 1< z <2
( z − 1)( z − 2)

1 1 1
Sol:- f ( z ) = = −
( z − 1)( z − 2) z − 2 z − 1

In first bracket z < 2 we take out 2 as common and from second bracket z is taken out
common as 1 < z .

94
MSCPH501

   
1  1  1  1 
  
f ( z) = − −
2 z  z 1
1−  1− 
 2  z
−1 −1
1 z 1 1
= − 1 −  − 1 − 
2 2 z z

1  z z2 z3  1 1 1 1 
= − 1 + + + + ... − 1 + + 2 + 3 + ...
2 2 4 8  z z z z 

1 z z2 z3 1 1 1 1
=− − − − − .... − − 2 − 3 − 4 ....
2 4 8 16 z z z z

This is the required expansion.

Que: Find the Laurent series expansion of

z
f ( z) = valid for z − 1 > 1.
( z − 1)( z − 2)

z 1 2 1 2
Sol. f ( z ) = =− + =− +
( z − 1)( z − 2) z −1 z − 2 z −1 z −1−1

−1
1 2 1 1 2  1 
=− + =− + 1 − 
z −1 z −1 1 z −1 z −1 z −1
1−
z −1

1 2  1 1 1 
=− + 1 + + 2
+ 3
+ ...
z −1 z −1 z − 1 ( z − 1) ( z − 1) 

95
MSCPH501

⋯Ans
1 2 2 2
=− + 2
+ 3
+ + ....
z − 1 ( z − 1) ( z − 1) ( z − 1) 4

3.16 SINGULARITY: A singular point or singularity of a function is the point of a


function at which the function ceases to be analytic.

1
For example:- If f ( z ) =
z−2

Then z=2 is a singularity of f(z).

3.17 Types of singularities:

3.17.1 Isolated singularity-If the function f (z) has a singularity at z = a and in a


neighborhood of ‘a’ (i.e. a region of the complex plane which contains a) there are no
other singularities then ‘a’ is an isolated singularity of f (z).

For example: If f(z)= 1/z then z=0 is an isolated singularity of f(z).

3.17.2 Removable singularity- In this type, if f(z) has a singularity at z=a then we can
remove this singularity.

sin z
For example, consider the function f (z) = , f (z) has an isolated singularity at
z
z=0.
1 z3 z5  z2 z4
f ( z) =  z − + − ........ = 1 − + − .....
z 3! 5!  3! 5!
Since no negative power of z occurs in the expansion. Hence z=0 is a removable
singularity.

3.17.3 Poles-Poles and Zeros of a function are the values for which the value of the
denominator and numerator of function becomes zero respectively. If the number
of terms are ‘m’ then z = a is said to be a pole of order m. A pole of order 1 is
called a simple pole.

1
If f ( z ) = then z=0 is a simple pole, z=5 is a pole of order 2 and z=4
z ( z − 5) 2 ( z − 4) 3
is a pole of order 3.

96
MSCPH501

3.17.4 Essential singularity-


1 1 1
If f ( z ) = e1 / z = 1 + + 2 + 3 + ........
z z .2! z .3!
Since in the expansion there is an infinite series of negative powers of z thus z=0 is
the essential singularity of f (z).

3.18 Residue:
For a function f (z), the Laurent expansion is-

∞ ∞
f ( z ) = ∑ a n ( z − a ) n + ∑ bn ( z − a ) − n
n=0 n =1

If this function f (z) has a pole of order m at z=a then its principal part is given by-

∑b n =1
n ( z − a) −n

1 f ( z) 1 f ( z)
Where, a n = ∫
2πi c ( z − a) n +1
dz, bn = ∫
2πi c ( z − a) −n+1
dz

1
2πi ∫c
Evidently, b1 = f ( z )dz

The coefficient “6 is called residue of f (z) at the pole z=a.

1
2πi ∫c
Res (z = a) = b1 = f ( z )dz

When z = a is a simple pole then,


b1
f ( z) = ∑ an ( z − a) n +
n =0 z−a


( z − a ) f ( z ) = ∑ an ( z − a ) n +1 + b1
n =0

lim( z − a ) f ( z ) = b1
z→a

1. Hence, for simple pole


Res (z=a) = lim( z − a ) f ( z )
z →a

97
MSCPH501

1 d m −1
2. For pole of order m , Re s( z = a) = lim m −1
[( z − a) m f ( z )]
z → a ( m − 1)! dz

3. Residue at infinity (a) Re s ( z = ∞) = lim − zf ( z ) if limit exists.


z →∞
(b) Res (z = ∞) = negative of the coefficient of 1/z in the
expansion of f (z)
3.19 CAUCHY RESIDUES THEOREM: If f (z) is analytic within and on a closed contour
c, except at a finite number of poles z1 , z 2 , z 3 ,........, z n within c, then,
n

∫ f ( z )dz = 2πi ∑ Re s ( z = z r ) = 2πi ∑ R +


c r =1

Where ∑R +
= sum of residues of f (z).

Proof-
Consider c1 , c 2 , c3 ,......, c n are the circles with centre a1 , a 2 , a3 ,......, a n respectively and
radii so small that they lie within closed contour c and do not overlap.
Since f(z) is analytic within the annulus bounded between these circles and the
contour c, then we know-

∫ f ( z )dz = ∫ f ( z )dz + ∫ f ( z)dz + .......... + ∫ f ( z)dz


c c1 c2 cn

Dividing by 2πi

1 1 1 1

2πi c
f ( z )dz = ∫
2πi c
f ( z )dz + ∫
2πi c2
f ( z )dz + ........ +
2πi c∫n
f ( z )dz
1

…….(1)

By the definition of residue,

1
2πi ∫c
Residue of f ( z ) = f ( z )dz

1
2πi c∫1
Re s( z = z1 ) = f ( z )dz

Hence, from equation (1), we get

98
MSCPH501

n
1
f ( z )dz = Re s ( z = a1 ) + Re s( z = a 2 ) + ............... + Re s( z = a n ) = ∑ Re s( z = a r )
2πi ∫c r =1

∫ f ( z )dz = 2πi ∑ Re s( z = a r )
c r =1

This is Cauchy residues theorem.

3.20 EVALUATION OF RESIDUES

1 − 2z
Question: Find the order of each pole and residue of .
z ( z − 1)( z − 2)
Ans:-
1 − 2z
Let f ( z ) =
z ( z − 1)( z − 2)

The poles of f(z) are given by z(z-1)(z-2)=0


z=0, 1, 2 all are simple poles.

z (1 − 2 z )
Residue of f (z) at (z=0) = lim( z − 0) f ( z ) = lim
z →0 z →0 z ( z − 1)( z − 2)

1 − 2z 1
= lim =
z →0 ( z − 1)( z − 2) 2

( z − 1)(1 − 2 z ) 1 − 2z
Residue of f (z) at (z=1)= lim( z − 1) f ( z ) = lim = lim =1
z →1 z →1 z ( z − 1)( z − 2) z →1 z ( z − 2)

( z − 2)(1 − 2 z ) 1 − 2z 3
Residue of f (z) at (z=2)= lim( z − 2) f ( z ) = lim = lim =−
z →2 z → 2 z ( z − 1)( z − 2) z → 2 z ( z − 1) 2

Hence, the residue of f (z) at z=0, 1 and z=2 are ½, 1 and -3/2 respectively.

z2
Question: Evaluate the residue of at 1,2,3 and infinity and show
( z − 1)( z − 2)( z − 3)
that their sum is zero.

99
MSCPH501

z2
Ans: Let f ( z ) =
( z − 1)( z − 2)( z − 3)

Re s ( z = 1) = lim( z − 1) f ( z )
z →1

z2 1
= lim( z − 1) =
z →1 ( z − 1)( z − 2)( z − 3) 2

z2
Res(z = 2) = lim(z − 2) f (z) = lim = −4
z→2 z→2 (z −1)(z − 3)

z2 9
Res(z = 3) = lim =
z →3 ( z − 1)(z − 2) 2
Res(z = ∞) = lim− zf (z)
z →∞

− z3
= lim
z →∞ ( z − 1)( z − 2)( z − 3)

− z3
= lim = −1
z →∞ 1 2 3
z 3 (1 − )(1 − )(1 − )
z z z

Sum of residues = 1 -4+ 9 -1 = 0


2 2

z+3
Question:-Evaluate the residue of f ( z ) = 2
z − 2z
Ans:
z +3
f ( z) =
z ( z − 2)

Poles are z=0 and z=2

( z − 0)( z + 3) z +3 3
Re s( z = 0) = lim = lim =−
z →0 z ( z − 2) z →0 z − 2 2
( z − 2)( z + 3) z +3 5
Re s( z = 2) = lim = lim =
z →2 z ( z − 2) z →2 z 2

1
Question:-Find the residue of f ( z ) = at z=ia.
(z + a 2 )2
2

100
MSCPH501

1
Ans: f ( z) =
(z + a 2 )2
2

1
=
( z + ia) ( z − ia ) 2
2

Poles are z=ia, -ia of order 2.

1 d  1 
Re s( z = ia) = lim  ( z − ia) 2 2 2 
z →ia ( 2 − 1)! dz ( z + ia) ( z − ia ) 

d  1 
= lim  2 
z →ia dz  ( z + ia) 
2 1
= lim − 3
=
z →ia ( z + ia) 4ia 3

z3
Question:-Find the residue of at z=∞.
z 2 −1

z3
Ans:-Let f ( z ) =
z2 −1

−1
z3  1 
f ( z) = = z 1 − 2 
 1   z 
z 2 1 − 2 
 z 
 1 
= z1 + 2 − ........
 z 
1
= z + − .............
z

Res (z=∞) = negative coefficient of 1/z = -1

3.21 EVALUATION OF INTEGRALS

(a) If f (z) is analytic in a closed curve C, except at a finite number


of poles within C, then ∫ f ( z )dz = 2πi i (sum of residues at the
c
poles within C).

101
MSCPH501

1+ z
Question:-Evaluate the following integral using residue theorem ∫ dz , where c is
c
z (2 − z )
the circle z = 1 .

1+ z
Ans. Let f(z)=
z (2 − z )

Poles are z=0, 2.


The integrand is analytic on z = 1 and all points inside
except z=0, as a pole at z=0 is inside the circle z = 1 .
Hence by residue theorem,

1+ z
∫ z (2 − z ) dz = 2πi[resf (0)]
c

z (1 + z ) 1
Residue f(0)= lim =
z →0 2−z 2
Putting the value of Residue f (0) in eq.(1), we get

1+ z 1
∫ z (2 − z ) dz = 2πi[ 2 ] = πi
c

4 − 3z
Question: Evaluate the following integral using residue theorem ∫ dz
c
z ( z − 1)( z − 2)
3
Where, c is the circle z = .
2
Ans: The poles of the function f(z) are given by
z=0, 1, 2
The function has poles at z=0, 1, 2 of which the given circle encloses the pole at z=0 and
z=1.
Residue of f (z) at the simple pole z=0 is

z (4 − 3z ) (4 − 3z ) 4−0
= lim = lim = =2
z →0 z ( z − 1)( z − 2) z → 0 ( z − 1)( z − 2) (−1) * (−2)

Residue of f(z) at the simple pole z=1 is

4 − 3z 4 − 3z
= lim( z − 1) = lim
z →1 z ( z − 1)( z − 2) z →1 z ( z − 2)

102
MSCPH501

4−3
= = -1
1(−1)

By Cauchy’s integral formula

∫ f ( z )dz = 2πi (sum of the residue within c)


c

=2πi (2-1) = 2πi.



(b) Evaluation of ∫ f ( x)dx
−∞
Let f (z) be a function such that-
(i) f (z) is analytic throughout the upper half plane except at certain points which are
its poles.
(ii) f (z) has no poles on the real axis i.e., if R→∞ (R being the radius of semi-circle),
then it will cover entire upper half plane.
(iii) zf(z) →0, uniformly as z →∞ for0≤arg z≤ π.
∞ 0
(iv) ∫
0
f ( x) dx and ∫ f ( x)dx both converges then
−∞

∫ f ( x)dx = 2πi∑ R Where ∑ R + denotes


+

−∞
the sum of the residues of f(z) at its pole in the upper
half plane.

Question: prove that ∫ dx 2 = π
01 + x 2

1
Ans: let f ( z ) = = π /2
1 + z2
Only z=i lies inside the contour c.
Res (z=i) = lim( z − i ) f ( z )
z →i

1 1
= lim( z − i ) =
z →i ( z + i )( z − i ) 2i

1
Hence, ∫−∞∞ f ( z )dz = 2πi ∑ R + = 2πi
2i

∫− ∞ f ( x ) dx = π
∞ dx
∫−∞ =π
1 + x2
∞ dx = π
∫0
1 + x2 2

103
MSCPH501

∞ x 2dx
Question:-Evaluate ∫ 2 2
−∞ (1 + x )( x + 4)

z 2dx
Ans: Consider ∫ 2 2
= ∫ f ( z )dz
c (1 + z )( z + 4) c

Where, C is the contour consisting of the semi-circle pÈ of radius R together with the
part of the real axis from –R to +R.

The integral has simple poles at


z = ±i, z = ±2i , of which z=i, 2i only lie inside C.

( z − i) z 2
The residue (at z = i) = lim
z →i ( z + i )( z − i )( z 2 + 4)

z2 1 1
lim =− =−
z →i ( z + i )( z 2 + 4) 2i (−1 + 4) 6i

( z − 2i ) z 2
The residue (at z=2i) = lim
z → 2i ( z 2 + 1)( z + 2i )( z − 2i )

z2 (2i ) 2
lim = = 1 / 3i
z →2i ( z 2 + 1)( z + 2i ) (−4 + 1)(2i + 2i )

By residue theorem,
∞  1 1 π
+
∫ f ( z ) dz = 2πi ∑ R = 2πi  − +  =
−∞  6i 3i  3

∞ log(1 + x 2 )
Question: Prove that ∫ dx = π log z
0 1 + x2

2
Ans: Let f (z) = log(1 + 2z )
1+ z

Poles of f (z) are given by

1 + z 2 = 0 , only z=i lies inside the contour c.

log( z + i )
Res (z=i) = lim ( z − i) f ( z ) = lim ( z − i) = log(2i) / 2i
z →i z →i ( z + i )( z − i )

104
MSCPH501

log(2eiπ / 2 )
= = (log 2 + log eiπ / 2 ) / 2i
2i

log 2 + iπ / 2
=
2i

Hence ∫ log( z +2 i ) dz = 2πi log 2 + iπ / 2 = π log 2 + iπ 2 / 2
−∞ 1+ z 2i

∞ log( x + i )
2∫ dx = π log 2 + iπ 2 / 2
2
0 1 + x
1
Using formula log (α+iβ) = log(α 2 + β 2 ) + i tan −1(β / α )
2

∞  log(1 + x 2 ) 2i tan −1( β / α ) 


∫  +  dx = π log 2 + iπ 2 / 2
 2 2 
0  1 + x 1 + x

Equating real parts, we get

∞  log( 1 + x 2 ) 
∫   dx = π log 2
2
0  1 + x 


(c) Integration round unit circle of the type ∫ f (cos θ , sin θ )dθ
0

Here ∫ f (cos θ , sin θ )dθ is a rational function of cosθ and sinθ.
0
Convert cosθ, sin θ into z.
Consider a circle of unit radius with centre at origin, as contour.

eiθ + e −iθ 1  1
cos θ = = z − 
2i 2i  z
e iθ − e − iθ 1  1
sin θ = = z + 
2 2i  z
As z = reiθ = 1 * eiθ = eiθ

As we know, z = eiθ → dz = eiθ idθ = izdθ

The integrand is converted into a function of z.


Then apply Cauchy’s residue theorem to evaluate the integral.



Question: Evaluate the integral ∫ 5 − 3 cos θ
0

105
MSCPH501

2π 2π
dθ dθ dz
Ans. ∫0 5 − 3 cos θ = ∫ [Let e iθ = z , dθ = ]
0 e iθ
+e − iθ
 iz
5 − 3 
 2! 

[c is the unit circle z = 1 ]

2 dz dz
=− ∫
i C (3z − 1)( z − 3)
= 2i ∫
C
(3z − 1)( z − 3)


dθ 2 dz 1 2dz
I= ∫ 10 − 3e θ
0
i
− 3e − iθ
=∫
C 10 − 3 z −
= ∫
3 iz i C 10 z − 3z 2 − 3
z

Poles of the integrand are given by (3z − 1)(z − 3)=0


i.e., z=1/3, 3.There is only one pole at z=1/3 inside the unit circle c.

Residue at z=1/3
 1  1
Re s z =  = lim  z −  f ( z )
 3  Z→ 1 3
3

 1
 z − 2i
 3 2i 2i i
= lim = lim = =−
1 (3 z − 1)( z − 3) 1 3( z − 3)
Z→ Z→ 1  4
3 3 3 − 3 
3 
Hence by Cauchy residues theorem
I = 2πi (sum of the residues within contour) = 2πi (-i/4) = π/2 Ans.


cos 3θ
Question:-Evaluate contour integration of the real integral ∫ 5 − 4 cos θ dθ .
0

2π 2π
cos 3θ e 3iθ
Ans: ∫0 5 − 4 cosθ dθ = Real part of ∫0 5 − 4 cos θ dθ

e 3iθ dz
= Real part of ∫0 5 − 2(e iθ + e −iθ ) dθ Let z = e iθ and dθ =
iz

z3  dz 
= Real part of ∫   
1   iz 
c is the unit circle.
C
5 − 2 z + 
 z

106
MSCPH501


1 z3
i ∫0 2 z 2 − 5 z + 2
= Real part of − dz

z3
= Real part of i ∫ dz
(2 z − 1)( z − 2)

Poles are given by (2z-1) (z-2) =0 i.e. z=1/2, z=2


z=1/2 is the only pole inside the unit circle.

 1
i z −  z 3
2
Residue (at z=1/2) = lim1 
Z → ( 2 z − 1)( z − 2)
2
3
iz i
= lim1 =−
Z→ 2( z − 2) 24
2


cos 3θ  i  π
∫ 5 − 4 cos θ dθ = Real part of 2πi  − 24  = 12 .
0
Ans.



Question: Evaluate the integral ∫ 2 + cosθ .
0

2π 2π 2π
dθ dθ 2dθ
Ans. Let I = ∫0 2 + cosθ = ∫0 eiθ + e−iθ = ∫ 4+eθ −e
i −iθ
2+ 0
2i

dz
Put eiθ = z so that eiθ (idθ ) = dz, dθ =

2 dz
I =∫ iz = 1 2 dz
C 4+ z +

1 i C z + 4z + 1
2

The poles are given by

− 4 ± 16 − 4 − 4 ± 2 3
z 2 + 4 z + 1 = 0 or z = = = −2 ± 3
2 2

The pole within the unit circle C is a simple pole at z= − 2 + 3 . Now we calculate the
residue at this pole.

107
MSCPH501

Residue at (z= − 2 + 3 ) =
1 ( z + 2 − 3) 2 2 2 1
lim = lim = =
Z → ( −2 + 3 ) i ( z + 2 − 3 )( z + 2 + 3 ) Z →( −2+ 3) i( z + 2 + 3 ) i (−2 + 3 + 2 + 3 ) 3i

Hence by Cauchy’s residues theorem, we have



∫ 2 + cosθ =2πi (sum of the residues within the contour)
0

1 2π
=2πi = . Ans
i 3 3

3.22 Summary: This chapter introduces imaginary and complex numbers. Complex
numbers are numbers of the form a + ib, where i = − 1 and a and b are real numbers. They
are used in a variety of computations and situations. Complex numbers are useful for our
purposes because they allow us to take the square root of a negative number and to calculate
imaginary roots.

In the beginning of this chapter, we have discussed the complex plane, along with the
algebra and geometry of complex numbers, and then we have made our way via
differentiation, integration, complex dynamics, power series representation and Laurent
series into territories at the edge of what is known today. Complex Integration now includes
a new and simpler proof of the general form of Cauchy's theorem. There is a short section on
the concept of Singularities, residues, poles and Evaluation of Integrals by using Cauchy
residue theorem.

3.23 References:

1. B. S. Rajput, Mathematical Physics, Pragati Prakashan.


2. H.K. Das, Dr. Rama Verma., Mathematical Physics, S. Chand.
3. Joseph Bak, Donald J. Newman, Complex Analysis, Springer.
4. Stephen D. Fisher, Complex Variables, 2 ed. (Dover, 1999).
5. Markushevich, A.I., Theory of Functions of a Complex Variable (Prentice-Hall, 1965).

3.24 EXERCISE

z z 
1. If z=a cosθ+ ia sinθ, prove that  + = 2 cos θ  .
z z 
z −1
2. Prove that = 1.
z −1
3. Test the analyticity of the function w= sin z.

108
MSCPH501

ez
4. ∫ dz Where c is the circle z = 2 . Ans: 2πie
c
z −1
2z 2 + z
5. ∫c z 2 − 1 dz , where c is the circle z − 1 = 1 Ans: 3πi

z
6. Expand in 1 < z < 2
( z − 1)( z 2 + 4)
2

1  2 2 2  z z
3

Ans: 
 + 3 + 5 + .....  −  + + ..... 
10  z z z  2 8 
2z 3 + 1
7. Find Taylor Expansion of f ( z ) = 2 about the point z=i.
z +z
 i 3  i ∞  1 1 
Ans:  −  +  3 + ( z − i ) + ∑ ( −1) n  n +1
+ n +1 ( z − i ) n
2 2  2 n=2  (1 + i ) (i ) 

8. Find the poles or singularity of the following function


1
(sin z − cos z )
Ans: Simple pole at z=π/4

1
9. Evaluate ∫1+ x
0
2
dx Ans: π/2


1
10. Evaluate ∫1+ x
−∞
4
dx Ans: π/ 2

Choose the correct alternative:

3
1. If z = r (cos θ + i sin θ ) then z is equal to:

r3
(i) (cosθ + i sin θ ) 3
(ii) r (cosθ + i sin θ )
3 3
(iii) (iv) r 3 .
2

1 αx
2. If f ( z ) = log( x 2 + y 2 ) + i tan −1 be an analytic function if α is equal to:
2 y
(i) 1 (ii) -1 (iii) 2 (iv) -2 .
z 2 − z +1
3. The value of ∫ dz , c being z = 1 is:
c
z − 1 2

109
MSCPH501

1
(i) 2πi (ii) (iii) 0 (iv) πi.
2πi

1
4. Let f ( z ) = , then z=2 and z=-3 are the poles of order:
( z − 2) ( z + 3) 6
4

(i) 6 and 4 (ii) 2 and 3 (iii) 3 and 4 (iv) 4 and 6.

e2z
5. What is the value of ∫ , where C is circle z = 1 ?
c ( z + 1) 4
4πi − 2 8πie −2
(i) Zero (ii) 4πie −2
(iii) e (iv) .
3 3

cos 2θ
6. The value of integral ∫ 5 + 4 cosθ dθ
0
is :

π π π
(i) π (ii) (iii) (iv) .
2 6 3
z
7. Find the sum of residues at all poles of function .
cos z

(i) π (ii) –π (iii) zero (iv) π/2.

8. Evaluate ∫ z dz
c
where the contour C is straight line from z= -i to z=+i

(i)zero (ii)1 (iii) –i (iv) i.


∞ 2
9. Evaluate ∫ −∞
e − x cos 2 xdx :

(i) π (ii) √π/e (iii) πe (iv) none of these.


e ikz
10. What is the residue at all poles of the function ?
a2 + z2
sinh( ka) i sinh( ka ) i sinh( ka )
(i) (ii) (iii) − (iv) zero.
a a a

Ans: (1)(iv) (2) (i) (3) (iii) (4) (iv) (5)(iv) (6) (iii) (7) (iii) (8)(iv) (9) (ii)
(10)(ii).

110
MSCPH501

__________________________________________________
UNIT 4: TENSOR
STRUCTURE
4.1 Objectives
4.2 Introduction
4.3 Tensors

4.4 Co-Ordinate Transformations

4.5 Indical and Summation Conventions

4.5.1 Indicial convention

4.5.2 Einstein’s summation convention

4.6 Dummy and Real Indices

4.7 Kronecker Delta Symbol

4.7.1 Some properties of Kronecker delta

4.7.2 Generalised Kronecker Delta

4.8 Scalars, Contravariant and Covariant vectors

4.8.1 Scalars

4.8.2 Contravariant vectors

4.8.3 Covariant vectors

4.9 Tensors of Higher Ranks

4.9.1 Contravariant tensors of second rank

4.9.2 Covariant tensor of second rank

4.9.3 Mixed tensor of second rank

4.9.4 Tensor of higher ranks, rank of a tensor

4.10 Symmetric and Antisymmetric Tensors

4.10.1 Symmetric Tensors

111
MSCPH501

4.10.2 Anti- symmetric tensors or skew symmetric tensors

4.11 Invariant Tensors

4.11.1 Levi-Civita symbol

4.12 Algebraic Operations on Tensors

4.12.1 Additional and subtraction

4.12.2 Equity of tensors

4.12.3 Outer product

4.12.4 Contraction of tensors

4.12.5 Inner Product

4.12.6 Quotient law

4.12.7 Extension of rank

4.13 Riemannian Space: Metric Tensor

Fundamental Tensors Gjk, Gjk And δö


ô
4.14

4.14.1 Covariant fundamental tensor gjk

4.14.2 Contravariant fundamental tensor gjk

4.14.3 Mixed fundamental tensor g ö or δ1ô


ô

4.15 Christofell’s 3-Index Symbols

4.16 Geodesics

4.17 Covariant derivative of a contravariant vector

4.18 Summary

4.19 Glossary

4.20 References

4.21 Suggested Readings

4.22 Terminal Questions

4.22.1 Short Answer type

112
MSCPH501

4.22.2 Long Answer type

4.22.3 Numerical Answer type

4.1 OBJECTIVES
After studying this unit, you should be able to-

• Define rank of tensor


• Define Covariant and Conravariant vectors
• Define Covariant and Conravariant tensors
• Define mixed tensor
• Define contraction of tensor
• Define summation and subtraction of tensor
• Define inner and outer product
• Define geodesics
• Define Christofell’s symbol

4.2 INTRODUCTION
In three dimensional space a point is determined by a set of three numbers called the co-ordinates
of that point in particular system. Tensor analysis is intimately connected with the subject of co-
ordinate transformations. Number of indices present in a physical quantity is called its rank. A
Tensor of rank zero is said to be scalar or invariant. A Tensor of rank one is said to be vector. A
Tensor having indices in superscript is said to be contravariant while Tensor having indices in
subscript is said to be covariant. A Tensor having indices both in subscript and superscript is
called mixed Tensor. If two contravariant or covarint indices can be interchanged without
altering the tensor, then the tensor is said to be symmetric with respect to these two indicas. A
tensor, whose each component alters in sign but not in magnitude when two contravariant or
covariant indices are interchanged, is said to be skew symmetric or anti- symmetric with respect

said to be invariant tensors. The Levi-Civita symbol is defined as a quantity εØôö in three
to these two indices. The tensor which has the same components in all co-ordinate systems are

dimensional space which is antisymmetric in all its indices. The sum or difference of two tensors
of the same rank and same type is also a tensor of the same rank and same type. Two tensors of
the same rank and same type are said to be equal if their components are one to one equal. The
algebraic operation by which the rank of a mixed tonsor is lowered by 2 is known as contraction.
An expression which are express the distance between two adjacent point is called a metric or
line element. The path of extremum (maximum or minimum) distance between any two points in

113
MSCPH501

Riemannian space is called the geodesic. The quadratic differential form gjkdxjdxk is independent
of the coordinates system and is called the Riemannian metric for n dimensional space. The
space which is characterised by Riemannian metric is called Riemannian space.

4.3 TENSORS: Tensors are important in physics as they provide a concise mathematical
framework for formulating and solving physics problems in areas such as mechanics (stress,
elasticity, fluid mechanics, moment of inertia etc.) and in electrodynamics (electromagnetic
tensor, Maxwell tensor, permittivity, magnetic susceptibility etc.) or general relativity (curvature
tensor, stress- energy tensor etc.).

In applications, it is common to study situations in which a different tensor can occur at each
point of an object; for example the stress within an object may vary from one location to another.
This leads to the concept of a tensor field. In some areas, tensor fields are so ubiquitous that they
are often simply called "tensors".

Number of indices present in a physical quantity is called its rank. A Tensor of rank zero is said
to be scalar or invariant. A Tensor of rank one is said to be vector.

4.4 CO-ORDINATE TRANSFORMATIONS


Tensor analysis is connected with the subject of co-ordinate transformations.

Consider two sets of variables (x1, x2, x3, …,xn) and žx6 , x / , x _ , … x " Ÿin two different frames of
reference which determine the co-ordinates of point in an n-dimensional space. Let the two sets
of variables be related to each other by the transformation equations

x6 = P6 (x6 , x / , x _ , … x " )

x / = P / (x6 , x / , x _ , … x " )

… … … …

… … … …

x " = P " (x6 , x / , x _ , … x " )

or briefly x 4 = P 4 (x6 , x / , x _ , … , x Ø , … , x " ) …(4.1)

(i = 1, 2, 3, …, n)

where function P 4 are single valued, continuous differentiable functions of co-ordinates. It is


essential that the n-function P 4 be independent. Equations (4.1) can be solved for co-ordinates xi
as functions of x 4 to yield

114
MSCPH501

x Ø = AØ (x6 , x / , x _ , … , x 4 , … , x " ) …(4.2)

Equations (4.1) and (4.2) are said to define co-ordinate transformations.

From equations (4.1) the differentials dx 4 are transformed as

dx 4 = dx6 + dx / + ⋯ + dx
5 5 5
ï ï ï "
ï Á ï & ï 6

= x i, (μ = 1, 2, 3, …, n) .
µ
n
∂dx

i =1 ∂x
i
…(4.3)

4.5 INDICAL AND SUMMATION CONVENTIONS


The summation convention implies the sum of the term for the index appearing twice in that
term over defined range. An index repeated as sub and superscript in a product represents
summation over the range of the index.

We can define two types of convention:

4.5.1 Indicial convention-Any index, used either as subscript or superscript will take all
values from 1 to n unless the contrary is specified. Thus, equations (4.1) can be written as

x = P 4 Yx Ø Z.
4
…(4.4)

The convention reminds us that there are n equations with μ = 1, 2, …n and A4 are the
functions of n-co-ordinates with (i = 1, 2, …, n).

4.5.2 Einstein’s summation convention-If any index is repeated in a term then a


summation with respected to that index over the range 1, 2, 3, …, n is implied. This convention
is called Einstein’s summation convention.
n

According to this conversation instead of expression ∑a


i =1
j xj
i
we write ai x .

Using above tow conversation eqn. (4.3) is written as


4
dx = dx Ø .
5
ï
ï 8
…(4.5a)

Thus, the summation convention means the drop of sigma sign for the index appearing twice in a
given term.

4.6 DUMMY AND REAL INDICES

115
MSCPH501

Any index which is repeated in a given term, so that the summation convention implies, is called

For example i is a dummy index inaØ x Ø .


a dummy index and it may be replaced freely by any other index not already used in the term.
4

4
dx = ï 9 dx ö = ï : dx ; .
5 5
ï ï
…(4.5b)

given term is called a real index. For example μ is a real index inpØ x Ø . A real index cannot be
Also two or more dummy indices can be interchanged. Any index which is not repeated in a
4

replaced by another real index, e.g.

pØ x Ø ≠ p<Ø x Ø .
4

SAQ 1: What is difference between real and dummy indices?

4.7 KRONECKER DELTA SYMBOL


In mathematics, the Kronecker delta (named after Leopold Kronecker) is a function of two
variables, usually just non-negative integers. The function is 1 if the variables are equal and 0

1 if j = k=
otherwise:

The symbol Kronecker delta δö =


ô
0 if j ≠ k
… (4.6)

The Kronecker delta δij is a piecewise function of variables i and j. For example, δ1 2 = 0,

whereas δ3 3 = 1.

4.7.1 Some properties of Kronecker delta


(i) If x1, x2, x3, …xn are independent variables, as

= δö .
ï > ô
ï 9
… (4.7)

(ii) An another property of Kronecker delta symbol is

δö P ô = P ö .
ô
… (4.8)

Since by summation convention in the left hand side of this equation the
summation is with respect to j and by definition of kronecker delta, the only surviving
term is that for which j = k.

(iii) If we are dealing with n dimensions, then

δô = δöö = n.
ô
…(4.9)

116
MSCPH501

By summation convention

δô = δ66 + δ// + δ__ + ⋯ + δ""


ô

= 1 + 1 + 1 + ⋯ + 1 = n.

δô δö = δØö .
ô ô
(iv) …(4.10)

By summation convention

δØô δö = δ6Ø δ6ö + δØ/ δ/ö + δØ_ δ_ö + ⋯ + δØØ δØö + ⋯ δØ" δ"ö
ô

= 0 + 0 + 0 + ⋯ + 1. δØö + ⋯ + 0

= δØö .

= = δö .
8
ï > ï ï > ô
8
ï ï 9 ï 9
(v) …(4.11)

4.7.2 Generalised Kronecker Delta: The generalised Kronecker delta is symbolized as


j j …j
δ6 / ?
k6 k / … k ?

and defined as follows:

(i) The subscripts and superscripts can have any value from 1 to n.
(ii) If either at least two superscripts or at least two subscripts have the same value or
the subscribts are not the same set as super-scripts, then the generalised Kronecker
delta is zero. For example
δØôö1 kk = δ1?? = δö1? = 0.
Øôö Øôö

(iii) If all the subscripts are separately different and the subscripts are the same set of
numbers as the superscripts, then the generalised Kronecker delta has value +1 or -1
according to whether it requires as even or odd number of permutations to arrange
the superscripts in the same order as the subscripts.

For example

δ6/_
6/_ = δ/_6 = δc6/š = +1
6/_ 6cš/

and δ6/_
/6_ = δ6_/ = δc6š/ = −1.
6/_ 6cš/

It should be noted that

117
MSCPH501

δ6Á , δ/& , δ_…" δôÁ , δ/ô& , δ_…"


ôÂ …ô6 = δôÁ , δô& , δôÂ …ô6 .
Ø Ø Â 6 6 Ø …Ø Á & Ø Ø
 6 Ø …Ø

4.8 SCALARS, CONTRAVARIANT VECTORS and

COVARIANT VECTORS

4.8.1 SCALARS: Consider a function ϕ in a co-ordinate system of variables xi and let his
function have the value ϕ in another system of variables x . If
4

ϕ = ϕ.

Then the function ϕ is said to be scalar or invariant or a tensor of the order zero.

The quantity

δØØ = δ66 + δ// + δ__ + ⋯ + δ"" = n.

Is a scalar or an invariant.

4.8.2 CONTRAVARIANT VECTORS: Consider a set of n quantities P6 , P/ , P_ , … P" in


6 / _ "
a system of variables xi and let these quantities have values P , P , P , … P in another co-ordinate
system of variablesx . If these quantities obey the transformation relation
4

4
P = = PØ
5
ï
ï 8
…(4.12)

where the quantities P Ø are said to be the components of a contravariant vector or a contravariant
tensor of first tank.

variables x .
Any n functions can be chosen as the components of a contravariant vector in a system of
4

and taking the sum over the index μ from 1 to n, we get


ï >
Multiplying equation (4.12) by ï 5

4
P =ï PØ = ï 8 PØ = Pô
5
ï > ï > ï ï >
5 5
ï ï 8

4
Pô = ï 5 P .
ï >
or …(4.13)

Equations (4.13) represent the solution of equations (4.12).

The transformation of differentials dx Ø and dx in the systems of variables xi and x


4 4

respectively, from eqn. (8.5a), is given by

118
MSCPH501

dx = dx Ø .
5
4 ï
ï 8
…(4.14)

As equations (4.12) and (4.14) are similar transformation equations, we can say that the
differentials dxi form the components of contravariant vector, whose components in any other
system are the differentials dx of that system. Also we conclude that the components of a
4

contravariant vector are actually the components of a contravariant tensor of rank one.

Let us now consider a further change of variables from x


4
to x’q, then the new
components P’q must be given by

P @A = P4 = . PØ
5
ï BC ï BC ï
5 5
ï ï ï 8
(using 4.12)

= PØ.
ï ,C
ï 8
…(4.15)

This equation has the same from as eqn. (4.12). This indicates that the transformations of
contravariant vectors form a group.

system of variables xi and let these quantities have values P6 , P/ , P_ , … P" in another
4.8.3 COVARIANT VECTORS: Consider a set of n quantities P1, P2, P3, …Pn in a

system of variables x . If these quantities obey the transformation equations


4

P= PØ.
ï 8
5
ï
…(4.16)

where the quantities Pj are said to be a covariant tensor of rank one.

Any n functions can be chosen as the components of a covariant vector in a system


of variables xi and equations (4.16) determine the n-components in the new system of
variables x .
4

and taking the sum over the index μ from 1 to n, we


5
ï
ï 8
Multiplying equation (4.16) by
get

P = PØ = P = Pô
5 5
ï ï ï 8 ï 8
ï > 4 ï > ï
5
ï > Ø

Pô = P4 .
5
ï
ï >
Thus, …(4.17)
4

components PA@ must be given by


Let us now consider a further change of variables fromx to x’q. Then the new

119
MSCPH501

PA@ = P = 5P
5 5
ï ï ï 8
ï ,C 4 ï ,C ï

= A.
ï 8
ï ,C 1
…(4.18)

This equation has the same form as eqn. (4.16). This indicates that the
transformation of contravariant vectors form a group.

= = .
ïý ïý ï 8 ï 8 ïý
5 5 5
ï ï 8ï ï ï 8
As

ïý
ï 8
It follows from (4.16) that form the components of a contravariant vector, whose
ïý
5
ï
components in any other system are the corresponding partial derivatives . This
convariant vector is called grad ψ.

4.9 TENSORS OF HIGHER RANKS

The laws of transformation of vectors are given by following formulas


4
Contravariant …P =
5
ï
ï 8
… (4.12)

PØ .
4
Covariant …P =
ï 8
5
ï
… (4.16)

4.9.1 CONTRAVARIANT TENSORS OF SECOND RANK-


Let us consider (n) 2 quantities Pij (here i and j take the values from 1 to n independently)
in a system of variables xi and let these quantities have values P
4<
in another system of
4
variablesx . If these quantities obey the transformation equations

P PØô
4<
=
5 D
ï ï
ï 8 ï >
…(4.19)

where the quantities Pij are said to be the components of a contravariant tensor of second
rank.

4.9.2 COVARIANT TENSOR OF SECOND RANK- If (n)2 quantities Aij in a


system of variables xi are related to another (n)2 quantities A4< in another system of
variables x by the transformation equations
4

120
MSCPH501

P4< = PØô
ï 8 ï >
5 D
ï ï
…(4.20)

where the quantities Pij are said to be the components of a covariant tensor of second
rank.

4.9.3 MIXED TENSOR OF SECOND RANK: If (n) 2


quantities PôØ in a system of
variables xi are related to another (n) 2 quantities P< in another system of variables x by
4 4

the transformation equations

P< = D Pô
4 ï
5
ï >
Ø
ï > ï
…(4.21)

where the quantities PôØ are said to be component of a mixed tensor of second rank.

SAQ 2: Show that Kronecker deltamixed tensor of rank 2?

4.9.4 TENSOR OF HIGHER RANKS, RANK OF A TENSOR-The tensors having


ranks more than two are called tensor of higher rank. Tensor of higher ranks are defined by
similar laws. The rank of a tensor only indicates the number of indices attached to its per
component. For example P1? are the components of a mixed tensor of rank 5; contravariant of
Øôö

rank 3 and covariant of rank 2.

They transform according to the equation

4<E
PA = P
5 D F G
ï ï ï
Øôö ï
ï 8 ï > ï 9ï C 1
…(4.22)
4<E
Where PA andP1
Øôö
are tensors of rank 4.

4.10 SYMMETRIC AND ANTISYMMETRIC TENSORS

4.10.1 SYMMETRIC TENSORS- If two contravariant or covarint indices can be


interchanged without changing the tensor, then the tensor is said to be symmetric with respect to
these two indices.

For example if

P
= =P
Øô ôØ

PØô = PôØ
or …(4.23)

then the contravariant tensor of second rank Pij or covariant tensor Pij is said to be symmetric.

121
MSCPH501

For a tensor of higher rank P1


Øôö
if

P1 = P1
Øôö ôØö

where the tensor P1


Øôö
is said to be symmetric with respect to indices i and j.

So if a tensor is symmetric with respect to two indices in any co-ordinate system, it remains
symmetric with respect to these two indices in any other co-ordinate system.

This can be seen as follows:

If tensor P1
Øôö
is symmetric with respect to first indices i and j, we have

P1 = P.
Øôö
…(4.24)

4<E
PH = P1
5 D F G
ï ï ï ï Øôö
ï 8 ï > ï 9 ï I
We have

= P jik.
5 D F
ï ï ï ï G
J
ï 8 ï > ï 9 ï
(using 4.24)

Now interchanging the dummy indices i and j, we get


4<E <4E
P = P jik = P
5 D F
ï ï ï ï G
J
ï > ï 8 ï 9ï

i.e., given tensor is gain symmetric with respect to first two indices in new co-ordinate system.
Thus, the symmetry property of a tensor is independent of coordinate system.

Let P1 be symmetric with respect to two indices, one contravarient i and the other covariant l,
Øôö

then we have

P1 = PØ
Øôö 1ôö
…(4.25)
4<E
P = P1
5 D F
ï ï ï ï G Øôö
J
ï 8 ï > ï 9 ï
We have

= J PØ .
5 D F
ï ï ï
1ôö ï G
ï 8 ï > ï 9ï
[(using 4.25)]

Now interchanging dummy indices i and l, we have


4<E
P = P1
5 D F
ï ï ï ï G Øôö
J
ï 8 ï > ï 9 ï

= P1 .
D F 5
ï 8ï ï ï Øôö
J
ï ï > ï 9 ï G
…(4.26)

122
MSCPH501

According to tensor transformation law,


<E
P4 = P1 .
J D F
ï ï ï ï G Øôö
5
ï 8 ï > ï 9 ï
…(4.27)

Comparing (4.26) and (4.27), we see that


4<E <E
P ≠ P4

i.e., symmetry is not preserved after a change of co-ordinate system.


"(" 6)
/
A symmetric tensor of rank 2 in n-dimensional space has at most independent
components.

SAQ 3: Show that the symmetry property of a tensor in independent of co-ordinate system used?

4.10.2 ANTISYMMETRIC TENSORS OR SKEW SYMMETRIC TENSORS-


A tensor, whose each component alters in sign but not in magnitude when two
contravariant or covariant indices are interchanged, is said to be skew symmetric or anti-
symmetric with respect to these two indices.

For example if

P
= = −P
Øô ôØ

PØô = −PôØ
or …(4.28)

then contravariant tensor Pij or covariant tensor Pij of second rank is antisymmetric or for a tensor
of higher rank P1 if
Øôö

P1 = −P1
Øôö Øöô

where tensor P1
Øôö
is antisymmetric with respect to indices j and k.

If tensor P1
Øôö
is antisymmetric with respect to first two indices i and j.

We have

P1 = −P1
Øôö ôØö
…(4.29)
4<E
P = P1
5 D F
ï ï ï ï G Øôö
J
ï 8 ï > ï 9 ï
and

=−ï P1 .
5 D F
ï ï ï ï G ôØö
J
8 ï > ï 9 ï
[using (4.29)]

Now interchanging the dummy indices i and j, we get

123
MSCPH501

4<E <4E
P = P1 = −P
5 D F
ï ï ï ï G Øôö
J
ï > ï 8 ï 9ï

i.e., given tensor is again antisymmetric with respect to first two indices in new co-ordinate
system. Thus, antisymmetry property is retained under co-ordinate transformation.
"("56)
/
An antisymmetric tensor of rank 2 in n-dimensional space has independent components.
Any tensor having either two contravariant or two covariant indices can be expressed as a sum
parts, one symmetric and the other antisymmetric.

P Øô = YP Øô + P ôØ Z + YP Øô + P ôØ Z
6 6
/ /
Thus, …(4.30)

the first term on the right is the symmetric part and the second is the antisymmetric part. The
symmetric part is a symmetric tensor and the antisymmetric part is an antisymmetric tensor.

The process of writing a tensor as a sum of symmetric and antisymmetric parts not only holds for
tensors of rank 2 but is quite general. For example a tensor Pö1 can be written as
Øô

Pö1 = / „Pö1 + Pö1 … + / „Pö1 + Pö1 …


Øô 6 Øô ôØ 6 Øô ôØ

Symmetric part Antisymmetric part

Pö1 = „Pö1 + Pö1 … + „Pö1 − P1ö …


Øô 6 Øô ôØ 6 Øô Øô
/ /
or
Symmetric part Antisymmetric part

SAQ 4: Show that the skew-symmetry property of a tensor of is independent of co-ordinate


system used?

4.11 INVARIANT TENSORS


The tensor which has the same components in all co-ordinate systems are said to
be invariant tensors.

Kronecker delta symbol and Levi Civita symbol (Epsilon tensor) are the important
examples of such tensors.

Kronecker delta: The kronecker delta symbol; is defined as

0 if i ≠ j=
δØô =
1 if i = j
.

We shall now prove that Kronecker delta is an invariant tensor.


4
Let δØô be the components of Kronecker delta in a system of variables xi and δ< as the
corresponding components in another system of variables x .
4

124
MSCPH501

If Kronecker delta is a mixed tensor of rank two then it must transform according to the
rule.
4
δ< = δØô
5
ï ï >
D
ï 8 ï
…(4.31)

= = .
5 5
ï ï >ï 8 ï ï 8
D D
ï 8 ï ï > ï 8 ï
…(4.32)

Since new variables x are the functions of old variables xi which in turn are the
4

functions of new variablesx , we have by chain rule


<

=ï = .
5 5
ï ï ï 8
D D
ï 8 ï

This gives the changes δx consequent upon change δx .


4 <

Since x and x are the coordinates of the same system, hence their variations are
4 <

independent of each other unless μ = v in which case

δx = δx
4 <

1 for μ = v=
∴ =
5
ï
ï
D
0 for μ ≠ v
.

Therefore, by definition of Kronecker delta, we have

= ï D δ< .
5 5
ï ï 8 ï 4
D
ï 8 ï
…(4.33)

4.11.1 LEVI-CIVITA SYMBOL (OR EPSILON TENSOR OR

in three dimensional space in a tensor of rank 3 and is denoted by εØôö while in four dimensional
ALTERNATING TENSOR OR PERMUTATION TENSOR): Levi-Civita symbol

space it is a tensor of rank four and denoted by εØôö1 . The Levi-Civita symbol is defined as a
quantity εØôö in three dimensional space which is antisymmetric in all its indices. Thus, the only
non-vanishing components of εØôö are those for which all the indices are different and they are
equal to +1 or -1 according as (i, j, k) is an even or odd permutation of (1, 2, 3), i.e.,

+1 if (i, j, k)is an even permutation of (1, 2, 3)


εØôö =# −1 if(i, j, k)is an odd permutation of (1, 2, 3) =
0 otherwise (contain two or more repeated indices).
…(4.34)

125
MSCPH501

4.12 ALGEBRAIC OPERATIONS ON TENSORS

4.12.1 ADDITION AND SUBTRACTION: The tensors are added and subtracted only
if the tensors have some rank and same type. Same type means the same number of contravarient
and covariant indices. To add or subtract two tensors the corresponding elements are added or
subtracted.

The sum or difference of two tensors of the same rank and same type is also a tensor of the same
rank and same type.

If there are two tenors Pö and Qö of the same rank (3) and same type (mixed with two indices in
Øô Øô

contravariant and one in covariant), then the laws of addition and subtraction are given by

Pö + Qö = R ö (Addition)
Øô Øô Øô
…(4.35)

Pö − Qö = Sö (Subtraction)
Øô Øô Øô
…(4.36)

where R ö and Sö are the tensors of the same rank (3) and same type (mixed with two indices in
Øô Øô

contravariant and one in covariant) as the given tensors.

The transformation laws for the given tensors are


4<
PE = F Pö
5 D
ï ï
Øô ï 9
ï 8 ï 8ï
…(4.37)

4<
QE = Qö
5 D
ï ï ï 9 Øô
F
ï 8 ï 8 ï
and …(4.38)

Adding (4.37) and (4.38), we get


4<
SE = F Sö
5 D
ï ï
Øô ï 9
ï 8 ï 8ï
…(4.39)

where is a transformation law for the sum and is similar to transformation laws for Pö and Qö
Øô Øô

given nby (4.37) and (4.38). Hence the sum R ö Y= Pö + Qö Z is itself a tensor of the same rank
Øô Øô Øô

and same type as the given tensors.

Subtracting eqn. (4.38) from (4.37), we get


4< 4<
PE − QE = (Pö − Qö )
5 D
ï ï ï 9 Øô Øô
F
ï 8 ï 8 ï

4<
SE = Sö .
5 D
ï ï ï 9 Øô
F
ï 8 ï 8 ï
or …(4.40)

126
MSCPH501

which is a transformation law for the difference and is again similar to the transformation law for
Pö and Qö . Hence the difference Sö Y= Pö − Qö Z is itself a tensor of the same rank and same
Øô Øô Øô Øô Øô

type as the given tensors.

SAQ 5: Show that sum and difference of tensor of same rank and same type is also a tensor of
the same rank and same type.

4.12.2 EQUITY of TENSORS: Two tensors of the same rank and same type are said to be
equal if their components are one to one equal, i.e., if

Pö = Qö for all values of the indices.


Øô Øô

If two tensors are equal in one co-ordinate system, they will be equal in any other co-
ordinate system.

SAQ 6: Show that two tensors are equal in one co-ordinate system, they will be equal in any
other co-ordinate system.

4.12.3 OUTER PRODUCT: The outer product of two tensors is a tensor whose rank is the
sum of the ranks of given tensors.

Thus, if t and t’ are the ranks of two tensors, then rank of their outer product will be (t + t’).

For example if Pö and Q1? are two tensors of ranks 3 and 2 respectively, then
Øô

Pö Q1? = R (say)
Øô
…(4.41)

is a tensor of rank 5 (= 3 + 2).

For proof of this statement we write the transformation equations of the given tensors as
4<
PE − Q E = Pö
5 D
4< ï ï ï 9 Øô
F
ï 8 ï 8 ï
…(4.42)

Q; = Q1? .
J
ï ï N
ï G
ï
: …(4.43)

Multiplying (8.42) and (8.43), we get


4<
PE − Q; = Pö Q1?
5 D J
ï ï ï 9ï ï N Øô
F
ï 8 ï > ï ï G
ï
:

4<
RE; = R ö?
5 D J
ï ï ï ï 9ï N Øô1
F
ï 8 ï > ï G ï ï
:
or …(4.44)

127
MSCPH501

which is a transformation law for tensor of rank 5. Hence the outer product of two tensors Pö and
Øô

Q1? is a tensor R ö? of rank (3 + 2 =) 5.


Øô1

The outer product of tensors is commutative and associative.

4.12.4 CONTRACTION OF TENSORS: The algebraic operation by which the rank of


a mixed tensor is lowered by 2 is known as contraction.

For example consider a mixed tensor P1? of rank 5 with contravariant indices i, j, k and
Øôö

covariant indices l, m.

The transformation law of the given tensor is


4<
PE; = P1? .
5 D F
ï ï ï ï Gï N Øôö
J
ï 8 ï > ï 9ï ï :
…(4.45)

To apply the process of contraction, we put λ = σ and obtain


4<E
P = P1?
5 D F
ï ï ï ï G ï N Øôö
E ï 8 ï > ï 9 ï
J
ï
F

= P1?
5 D F
ï ï ï Gï ï N Øôö
J F
ï 8 ï >ï ï 9 ï

= ö P1?
δ?
5 D
ï ï ï G Øôö
J
ï 8 ï > ï

žsince substitution operator ï = δ?


ö Ÿ
F
ï ï N
F
9 ï

4<E
P = J P1ö
5 D
ï ï Øôö ï G
i.e., E ï 8 ï >ï
…(4.46)

which is a transformation law for a mixed tensor of rank 3.

SAQ 7: Show that contraction of a tensor of rank 7 is tensor of rank 5?

4.12.5 INNER PRODUCT: The outer product of two tensors followed by a contraction
results a new tensor called and inner product of the two tensors and the process is called the
inner multiplication of two tensors.

Example (a) Consider two tensors Pö and Q1?


Øô

The outer product of these two tensors is

Pö Q1? = R ö? (say)
Øô Øô1

Applying contraction process by setting m = i, we obtain

128
MSCPH501

Pö Q1? = R ö? = Sö (a new tensor)


Øô Øô1 ô1

The new tensor Sö is the inner product of the two tensors Pö and Q1? .
ô1 Øô

(b) An another example consider two tensors of rank 1 as P Ø and Qô . The outer product of
P Ø and Qô is

P Ø Qô = RØô .

Applying contraction process by setting i = j, we get

P Ø Qô = R ô (a scalar or a tensor of rank zero).


ô

Thus, the inner product of two tensors of rank one is a tensor of rank zero. (i.e.,
invariant).

4.12.6 QUOTIENT LAW: Quotient law provided a direct method to find out if the given
entity is a tensor or not. Quotient law states that:

An entity whose inner product with an arbitrary tensor (contravariant or covariant) is a tensor, is
itself a tensor.

quotient law let us consider an arbitrary tensor Q1ôö whose inner product with P (i, j, k) is a tensor.
Example: Let P (i, j, k) be the given entity to be tested whether it is a tensor or not. To apply

i.e.,

P (i, j, k)Q1ôö = R1Ø

We have to show that P (i, j, k) is a tensor. In the other system of variables x , we must
4

have

P(μ, v, σ)Q<E = R4 .

Q<E = Q1ôö
J
ï ï > ï 9
D F
ï G ï ï
Now

R4 = Rô .
J
ï ï 8 ô
5
ï G ï
and

4.12.7.EXTENSION OF RANK:
4 4
R< = P Q< = PØ ï D Qô
5
ï ï >
ï 8
So that

= PØ Qô = D Rô .
5 5
ï ï > ï Øï >
D
ï 8 ï ï 8ï

129
MSCPH501

The rank of a tensor can be extended by differentiating its each component with respect to
variables xi.

As an example consider a simple case in which the original tensor is of rank zero, i.e., a
ïQ
scalar S (xi) whose, derivatives relative to the variables xi are ï 8 . In other system of variables
x the scalar is SYx Z, such that
4 4

= = .
ïQ ïQ ï 8 ï 8 ïQ
5 5 5
ï ï 8 ï ï ï 8
…(4.49)

ïQ
ï 8
This shows that , transforms like the components of a tensor of rank one. Thus, the
differentiation of a tensor of rank zero gives a tensor of rank one. In general we may say that the
differentiation of a tensor with respect to variables xi yields a new tensor of rank one greater than
the original tensor.

The rank of a tensor can also be extended when a tensor depends upon another tensor and
the differentiation with respect to that tensor is performed. As an example consider a tensor S of
rank zero (i.e., a scalar) depending upon another tensor Pij, then

= QØô = a tensor of rank 2.


ïQ
ï 8>
…(4.50)

Thus, the rank of the tensor of rank zero has been extended by 2.

4.13 RIEMANNIAN SPACE: METRIC TENSOR


An expression which are express as the distance between two adjacent point is called a
metric or line element. In three dimensional space the line element, i.e., the distance between two
adjacent points (x, y, z) and (x + dx, y + dy, z + dz) in Cartesian coordinates is given by

ds2 = dx2 + dy2 + dz2.

In terms of general curvilinear coordinates, the line element becomes


3 3
ds 2 = ∑∑ g jk du j duk =g jk du j duk (Using summation convention).
j =1 k =1

This idea was generalised by Riemann to n-dimensional space.

The distance between two neighbouring points with coordinates xj and xj + dxj is given
by
n n
ds 2 = ∑∑ g jk dx j dx k =g jk dx j dx k …(4.54)
j =1 k =1
(Using summation convention)

130
MSCPH501

where the coefficients gjk are the functions of coordinates xj, subject to the restriction g =
determinant of gjk, i.e, g ôö ≠ 0.

The quadratic differential form gjkdxjdxk is independent of the coordinates system and is
called the Riemannian metric for n dimensional space. The space which is characterised by
Riemannian metric is called Riemannian space. Hence the quantities gjk are the components of a
covariant symmetric tensor of rank two, called the metric tensor or fundamental tensor.

(dx1)2 + (dx2)2 + (dx3)2 + … + (dxn)2 or dxjdxk,

the space is called n-dimensional Euclidean space. It is now obvious that Euclidean spaces are
the particular cases of Riemannian space.

In general theory of relativity (four dimensional space), the line element is given
by

Ds2 = gjkdxjdxk (j, k = 1, 2, 3, 4).

In special theory of relativity, the line element is given by

(dx1)2 + (dx2)2 + (dx3)2 + … + (dxn)2 or dxjdxk.

the space is called n-dimensional Euclidean space. The Euclidean spaces are the
particular cases of Riemannian space.

In general theory of relativity (four dimensional space), the line element is given
by

ds2 = gjkdxidxjdxk (j, k = 1, 2, 3, 4).

In special theory of relativity, the line element is given by

ds2 = (dx1)2 + (dx2)2 + (dx3)2 [with x4 = ict, i = √ (-1)]

= dxjdxj (j = 1, 2, 3, 4).

As ds2 = gjkdxjdxk has been defined in general space, it is independent of the


coordinate system, i.e., dx2 = gjkdxjdxk is an invariant.

4.14 FUNDAMENTAL TENSORS gjk, gjk AND RS


T

4.14.1 COVARIANT FUNDAMENTAL TENSOR gjk: The line element or interval


ds in Riemannian space is given by

ds2 = gjkdxjdxk. …(4.55)


131
MSCPH501

As dxjdxk are contravariant vectors and ds2 is invariant for arbitrary choice of vectors dxj
and dxk, it follows from quotient law that gjk is a covariant tensor, we have

ds2 = gjkdxjdxk in system of variables xj


4 <
= g4< dx dx in system of variables x
4

4 <
i.e., = g4< dx dx = gjkdxjdxk. …(4.56)

Now applying inverse transformation law to dxj and dxj, i.e.,

dx ô = ï dx etc.
ï > 4
5

4 < ∂x ô 4 ∂x
ö
g4< dx dx = g ôö 4 dx dx
<
∂x ∂x
<

∂x ô ∂x ö
= g ôö dx dx
4 <
∂x ∂x
4 <

Ug4< − g ôö V dx dx = 0.
ï > ï 9 4 <
5 D
ï ï
i.e., …(4.57)

As dx and dx are arbitrary contravarient vectors, we must have


4 <

∂x ô ∂x ö
g4< − c =0
∂x ∂x
4 <

∂x ô ∂x ö
g4< = g ôö .
∂x ∂x
4 <

Hence g ôö is a covariant tensor of rank 2.

g ôö may be expressed as

1 1
g ôö = Yg ôö + g öô Z + Yg ôö − g öô Z
2 2
= Pôö + Qôö …(4.58)

where Aôö = / Yg ôö + g öô Z is symmetric tensor


6
= W
and Bôö = / Yg ôö − g öô Z is symmetric tensor
6 …(4.59)

∴ ds2 = gjkdxjdxk = (Ajk + Bjk) dxjdxk. …(4.60)

132
MSCPH501

We have

Bjkdxjdxk = Bkjdxkdxj (interchanging dummy indices j and k)

= – Bjkdxjdxk

(since Bjkis antisymmetric i.e., Bjk= – Bkj)

i.e., 2Bjk dxjdxk= 0.

As dxj and dxk are arbitrary vectors, we have

Bjk = 0

Yg ôö + g öô Z = 0
6
/
i.e.,

i.e., g ôö + g öô = 0

i.e., g ôö is symmetric.

So, we can write g ôö as

g 4< . = / Yg 4< + g <4 Z.


6

Thus, we have proved that the metric tensor gjk is covariant symmetric tensor of rank 2.
This is called covariant fundamental tensor of rank 2.

4.14.2 CONTRAVARIANT FUNDAMENTAL TENSOR gjk


Let us define gjk as

g ôö
XYZ[X Y YZ \>9 Ø" \
\
…(4.61)

where g is the determinant of gjk, i.e.,

g66 g6/ g6_ … g6"


g /6 g // g /_ … g /"
… … … … … ®.
g = g ôö = ®®

… …
… …
… … …®
… …
g "6 g "/ g "_ … g ""

Since gjk is symmetric, g is symmetric which implies cofactor of gjk in g is symmetric and
jk
so g is symmetric.

Let Pj be an arbitrary contravariant vector, then by quotient law,

Pk = gjkPj …(4.62)
133
MSCPH501

is an arbitrary covariant vector.

Now multiplying eqn. (4.62) by gkl, we get

gklPk = gjkgklPj. …(4.63)


XYZ[X Y YZ \9G Ø" \
\
But gjkgkl = gjk (using 4.61)

= δ1ô (by theory of determinants). …(4.64)

Therefore, equation (4.63) yields

gklAk = δ1ô Pj = Pl …(4.65)

i.e., the inner product of gkl with an arbitrary covariant vector Pk yields a contravariant vector.
Hence by quotient law gkl is a contravariant tensor of rank 2.

4.14.3 MIXED FUNDAMENTAL TENSOR ] S or R^T


T

From equation (4.64), we have

gjkgkl =δ1ô . …(4.66)

As gjk and gkl are covariant and contravariant tensors of rank 2 respectively, therefore,
from quotient law δ1ô is also a tensor of rank 2; it is a mixed tensor, contravariant in l and
covarian in j and is known as mixed fundamental tensor.

4.15 CHRISTOFELL’S 3-INDEX SYMBOLS


We now introduce two expressions formed of the fundamental tensors, known as
Christofell’s symbols of first and second kind.

Christofell’s symbol of first kind

jjk, ll = Γ1,ôö = ž + − Ÿ.
6 ï\G> ï\9G ï\>9
/ ï 9 ï > ï G
…(4.67)

Christofell’s symbol of second kind.

l
= Γ 1 .ôö = / g 1? ž ï >N + ï9N −ï N Ÿ.
6 ï\ ï\ ï\>9
jk 9 > …(4.68)

From the symmetry property of gjk it follows that

jjk, ll = jkj, llorΓ1,ôö = Γ1.öô …(4.69)

134
MSCPH501

l l
= or Γ 1 .ôö = Γ 1 öô
jk kj
and …(4.70)

there by indicating that Christofell’s symbols Γ1.ôö and Γ 1 ôö are symmetrical with respect to
indices j and k.

Relations between Christofell’s symbols of first and second kind

(i) Replacing l by m in eqn. (4.67), we get

Γ?,ôö = ž + − Ÿ.
6 ï\N> ï\9N ï >9
/ ï 9 ï > ï N

Multiplying both sides of above equation by glm, we get

g 1? Γ?,ôö = / g 1? ž ï >N
9 +
−ï N Ÿ
6 ï\ ï\9N ï\
>9
ï >
(since gjm = gmj)

= Γôö1 [Using (4.68)]

i.e., Γôö1 = g 1? Γ?,ôö . …(4.70)

(ii) Interchanging l and m in eqn. (4.68), we get

Γôö? = / g 1? žï + − Ÿ.
6 ï\>G ï\9G ï\>9
9 ï > ï G

Multiplying above equation by glm, we get

g 1? Γôö? = / g 1? g 1? žï + − Ÿ
6 ï\>G ï\9G ï\>9
9 ï > ï G

= / žï + − Ÿ (since glmgml = δ11 = 1)


6 ï\>G ï\9G ï\>9
9 ï > ï G

= Γ1,ôö .

4.16 GEODESICS
In Euclidean three dimensional space the path of shortest distance between two fixed
points is a straight line. Here we shall generalise this fundamental concept to Riemannian space.

The path of extremum (maximum or minimum) distance between any two points in
Riemannian space is called the geodesic. Thus, a geodesic is determined by the condition that
the path between two fixed points A and B given by be extremum, i.e.,
B

∫ ds
A
B
extremum (or stationary), …(4.71)
∫ ds
A
135
MSCPH501

B
δ ∫ ds = 0
i.e., A …(4.72)

Where δ represents the variation symbol.

In Riemannian space, we have

ds2 = gjkdxjdxk …(4.73)

2 ds δ (ds) = δ (gjk) dxjdxk + gjkδ (dxj) dxk + gjkdxjδ (dxk)

δx ? + gjkdxkδ (dxj) + gjkdxjδ (dxk).


ï\>9
= dxjdxkï N

Dividing both sides by 2 ds and using the relation

δž Ÿ= Yδx ô Z.
ï > `
` `

We get(ds) = / U ` + −ï N δx
+ g ôö Yδx ô Z + g ôö Yδx ö ZV ds.
6 ` > ` 9 ï\>9 ? ` 9 ` ` > `
` ` ` ` `

Substituting the value of δ(ds) from (4.73) in (4.72), we get

1  dx j dx k ∂g jk m
B
dx j d dx k d 
∫ 
2 A  ds ds ∂x m
δ x + g jk
ds ds
( δ x ) + g jk
k

ds ds
( δ x j )ds = 0

On changing the dummy indices in the last two terms, we get

1  dx j dx k ∂g jk m  
B
dx j dx k  d m 
∫  m
δ x + g
 jm + g mk  . (δ x ) ds = 0.
2 A  ds ds ∂x  ds ds  ds 

Integrating the second term by parts and remembering that the variation δ is zero at the
fixed end points A and B,

1  dx j dx k ∂g jk d  dx k   m
B
dx j
2 ∫A  ds ds ∂x m ds 
 − g
 jm + g mk  δ x ds = 0.
ds ds  

As the infinitesimal displacements δx ? are arbitrary, therefore for the integral to be


stationary the coefficient of δx ? in the integrand must vanish at all points on the path, i.e.,

1  dx j dx k ∂g jk d  dx j dx k  
 − g
 jm + g mk  = 0
2  ds ds ∂x m ds  ds ds  

136
MSCPH501

1 dx j dx k ∂g jk 1 dg jm dx j 1 d 2 xk
− − g mk = 0.
i.e., 2 ds ds ∂x m 2 ds ds 2 ds 2

1 dg mk dx k 1 d 2 xk
− − g mk = 0.
2 ds ds 2 ds 2 …(4.74)

But we have

dg jm ∂g jm dx k dg mk ∂g mk dx j
= and =
ds ∂x k ds ds ∂x j ds

With these substitutions, equation (4.74) becomes

1 dx j dx k  ∂g jk ∂g jm ∂g mk  1 d 2x j d 2 xk 
 − −  − g
 jm + g mk  = 0.
2 ds ds  ∂x m ∂x k ∂x j  2 ds 2 ds 2 
i.e.,

Replacing the dummy indices j and k and l in the second bracketed terms, we get

1 dx j dx k  ∂g jk ∂g jm ∂g mk  1 d 2 xl d 2 xl 
 − −  − g
 jm 2 + g mk  = 0.
2 ds ds  ∂x m ∂x k ∂x j  2 ds ds 2 

Using symmetry property of glm (i.e., glm = gml) above equation mau be written as

1 dx j dx k  ∂g mj ∂g km ∂g jk  d 2 xl
 − −  lm 2 = 0.
+ g
2 ds ds  ∂x k ∂x j ∂x m  ds

Now multiplying throughout by gmp, we get

1 dx j dx k mp  ∂g mj ∂g km ∂g jk  2 l
mp d x
g  k + − +
 lmg g =0
2 ds ds  ∂x ∂x j ∂x m  ds 2

1 dx j dx k mp d 2 xl
g Γ m. jk + δ l p =0
or 2 ds ds ds 2

d 2 x p dx j dx k mp
+ g Γ m. jk = 0
i.e., ds 2 ds ds …(4.75)

d 2 x p dx j dx k p
+ Γ jk = 0.
or ds 2 ds ds …(4.76)

137
MSCPH501

4.17 COVARIANT DERIVATIVE OF A CONTRAVARIANT VECTOR


Let Aj be a contravariant vector, then by tensor transformation law
4
A = Aô .
5
ï
ï >
…(4.77)

Differentiating above equation with to x , we get


<

µ µ µ
∂A ∂ x ∂A j ∂x k ∂ 2 x ∂x k j
v
= j + A
∂A ∂x ∂x k ∂ x v ∂x j ∂x k ∂ x v
µ µ µ
∂A ∂ x ∂x k ∂A j ∂ 2 x ∂x k j
v
= j + A
∂A ∂x ∂ x v ∂x k ∂x j ∂x k ∂ x v …(4.78)

The presence of the last term on the R.H.S. of eqn. (4.77) shows that the partial
5
ï > ï
derivativesï 9 or ï D do not transform like the components of tensor.

From eqn. (4.77) interchanging x and x co-ordinates, we get


µ µ σ λ
∂2 x ∂x ∂x ∂x µ
j k
= Γ pjk p − j Γσλ
∂x ∂x ∂x ∂x ∂x k …(4.79)
µ µ µ σ λ
∂A ∂ x ∂x k ∂A j  p ∂ x ∂ x ∂ x µ  ∂x k j
= j 
+ Γ jk p − j Γσλ  v A
∂A
v
∂ x ∂x
v
∂x k
 ∂x ∂ x ∂x k

  ∂x
µ µ σ
∂ x ∂x k ∂A j p ∂ x ∂x
k
µ ∂x
= j v k
+ Γ jk p v
A j − δ vλ Γσλ j
Aj
∂x ∂ x ∂x ∂x ∂ x ∂x
µ µ
∂ x ∂x k ∂A j p ∂ x ∂x
k
µ
= j v k
+ Γ jk p v
A j − Γσ v Aσ
∂x ∂ x ∂x ∂x ∂ x

Interchanging dummy indices j and p in the second term on R.H.S. of above equation, we
obtain
µ µ
∂A ∂ x ∂x k
µ  ∂A j j p 
v
+Γ A= j
σv  k + Γ pk A 
∂A ∂x ∂ x v  ∂x  …(4.80)

Introducing the comma notation

µ ∂A j j
∂ A;k = + Γ pk Ap
∂x k …(4.81)

138
MSCPH501

Eqn. (8.108) can be written in the form


µ
µ∂ x ∂x k j
∂A =;k A; k
∂x j ∂ x v …(4.82)

This equation shows that A;ö defined by (4.82) is a mixed of rank two, called the
ô

covariant derivative of Aj with respect to xk.

4.18 SUMMARY
In this unit you have learned about tensor analysis, rank of tensor, types of tensor etc. You have
learnt coordinate transformation in terms of covariant and contravariant vector and tensor. You
have also learnt reduction of rank of a tensor and algebra rule like addition, subtraction,
multiplication etc. You have also learnt symmetric and skew symmetric tensor. You have also
learnt Christofell’s three index symbol and their relationship. In this unit you have studied
fundamental tensor, Remanian metric, Geodesic. Many solved examples are given in the unit to
make the concepts clear. To check your progress, self assessment questions (SAQs) are given
place to place.

4.19 GLOSSARY
Dummy indices – which can be changed without altering meaning
Covariant – set of quantities remain unchanged
Contravariant - not comparable
Geodesic- the shortest line between two points that lies in a given surface.
Fundamental - basic

4.20 REFERENCES

1. Mathematical Physics, H.K. Das

2. Mathematical Physics, B.S. Rajput

3. Objective Physics, Satya Prakash, AS Prakashan, Meerut

4. Tensor Calculus And Riemanning Geometry, J.K. Goyal,K.P. Gupta

4.21 SUGGESTED READINGS


1. Modern Physics, Beiser, Tata McGraw Hill

139
MSCPH501

2. Physics Part-I, Robert Resnick and David Halliday, Wiley Eastern Ltd

3. Berkeley Physics Course Vol I, Mechanics, C Kittel et al, McGraw- Hill Company

4.22 TERMINAL QUESTIONS

(Should be divided into Short Answer type, Long Answer type, Numerical, Objective type)
4.22.1 Short Answer type
1. What do you understand by dummy and real indices?
2. Explain Kronecker delta. Discuss some properties of kronecker delta.
3. Explain contravariant vectors.
4. Explain covariant vectors.
5. Discuss contravariant tensors of second rank.
6. Discuss Covariant tensor of second rank.
7. What do you understand by mixed tensor of second rank?
8. Explain symmetric and antisymmetric tensors.

4.22.2 Long Answer type


1. Show that the sum or difference of two tensors of the same rank and same type is
also a tensor of the same rank and same type?
2. What do you understand by Contraction of tensors? Discuss it with an example.
3. Explain Metric tensor in Riemannian space.
4. Explain Christofell’s 3-index symbols. Establish Relations between Christofell’s
symbols of first and second kind.

140
MSCPH501

________________________________________________________

UNIT - 5 LINEAR DIFFERENTIAL EQUATIONS OF


FIRST and SECOND ORDER
__________________________________________________________________

STRUCTURE
5.1 Objectives
5.2 Introduction
5.3 Ordinary and Partial Differential Equations
5.4 Linear Differential Equations
5.4.1 Linear Differential Equation of First Order
5.4.2 Solution of a First Order Linear Differential Equation
5.4.2.1 Separation of Variable Method
5.4.2.2 Using Integrating Factor Method
5.4.2.3 Change of Variable Method
5.5 Second Order Linear Differential Equation
5.5.1 Solution of Differential Equation of Second Order
5.5.2 Complementary Functions
5.5.3 Rules for Finding Complementary Functions
5.5.4 Particular Integral and Rules for finding Particular Integral
5.6 Summary
5.7 Glossary
5.8 References
5.9 Suggested Readings
5.10 Terminal Questions
5.11 Answers

141
MSCPH501

UNIT 5:
Differential equations: Linear ordinary differential equations of first and second order

5.1 Objectives
The Learning objectives of this unit are
1. To know the difference between linear and non-linear differential equations.
2. To classify the differential equations according to their order.
3. To find the solution of linear first and second order differential equations using different
approaches.

5.2 Introduction
A great many number of problems in nature, either scientific or non-scientific, involves rate of
change of one quantity with respect to another, this referred to as derivative in mathematics.
A differential equation is an equation expressing a relation between a function and its derivatives
it contains derivatives either ordinary or partial. Most common example of differential equation,
which every one might have come across with, is the Newton’s second law. The equation is F =
ma, where F is the force applied on a particle of mass m and ‘a’ is the acceleration which results
because of that force. We may also write the equation as: h = q •Œ & we can see that the force on
•& =

the body is expressed as a double differential of the position w.r.t time. Other frequently
encountered differential equations are; the Laplace’s equation, the Poisson’s equation and many
others.

5.3 Ordinary and Partial Differential equations

The function which is being described by a differential equation decides whether it is an ordinary
differential equation or partial differential equation. If the function has single independent
variable it is ordinary differential equation, whereas if the function has more than one

expressed as complete derivatives ž•=Ÿ , whereas the partial differential equations are expressed
independent variable then it is partial differential equation. Ordinary differential equations are

œ
as (œ= ).
As an example consider the function y = 2x or any higher powers of x, now the differential
equation expressing the different derivatives of the function would be called an ordinary
differential equation since the function has a single independent variable x.
On the other hand for the function of type y = xz + xz2, the differential equation should contain
terms representing rate of change of y with respect to both the variables x and z, thus this
differential equation is termed as partial differential equation.

Example of ordinary differential equation isNewton’s second law, rate of change equations etc
Examples of Partial differential equations are Schrodinger equation, Maxwell’s equations etc

Order of a differential equation:


142
MSCPH501

The order of a differential equation is the order of the highest derivative of the unknown function

derivatives such as the expression for slope of a line; q = •= , whereas a second order
involved in the equation, for example a first order differential equation contains only first order
•<

differential equation contains at least one second order derivative such as Newton’s second law.
The order of a differential equation does not depend on whether the equation is ordinary or
partial. Some examples of differential equations ca be summarized as

ay’’+ by’ + cy = g (t) (1)

à 4 (J) = (1 − J) + J/ .
•& < •< 5š<
•= & •=
(2)

5.4 Linear Differential equations

The differential equations are further classified as linear differential equations and nonlinear
differential equations. The linear differential equations are those differential equations in which
the dependent variable and their derivatives do not occur in product form or in powers other than
single power. For example a linear differential equation ca be written as

an(t) y(n)(t) + a(n-1)(t) y(n-1)(t)+ - - - - - - - - - - + a1(t) y’(t) + a0(t) y(t) = g(t). (3)

The coefficients a0(t), ….. an(t) and g(t) can be zero or non - zero functions, constant or non - constant
functions, linear or non – linear functions. Only the function y (t) and its derivatives are considered to
determine whether a differential equation is linear or not.If a differential equation cannot be written in the
form of (3) it is a non - linear differential equation as is equation (2). The fundamental equations of
atmospheric physics are non-linear.

For the present unit we will restrict ourselves to linear differential equations of first and second orders
only as these are more frequently involved in studying different physical phenomenon.

We start here with first order ordinary linear differential equation.

5.4.1 Linear Differential equation of First order


A first order ordinary linear differential equation is one which involves only first derivative of the
independent variable.
The standard form of first order ordinary differential equation can be expressed as:

y' + P(x) y = Q(x). (4)

where P and Q are functions of x.


As defined this equation is a first order differential equation as the derivative of the unknown variable y
is of first order only, the equation is also linear because dependent variable y is neither in product form
nor in any power other than single power.

A first order linear differential equation can be encountered in different fields of study such as scientific
research, engineering, and economics etc.

143
MSCPH501

An example of first order linear differential equation we may study the radioactive decay equation.

According to law of radioactivity, a unstable nuclei decays to more stable nuclei and the rate of this decay
is proportional to the initial number of the unstable nuclei, we may express this as

NS
-S
N

NS
= −+ S
N
NS
= −+ N
S
This is of the form of eq 4, a linear differential equation of first order

The solution of this differential equation can be done as

Integrating both sides we get

ln S = −+N + p 4 4 .

Using initial conditions we can solve this to get N = N0e-λt.

Further example of differential equation of this type may be encountered in electronic circuits such as a
simple series circuit as shown below

For the given series circuit above containing resistor R, capacitor C, and inductor L and a source of emf
V, we can study this circuit using a first order linear differential equation as follows
If at time t the current flowing through the circuit is I(t) and the charge on the capacitor is q(t) then I =
dq/dt, The voltage across R is RI, the voltage across capacitor is q/V and the voltage across inductor is
L(dI/dt). At any time t we must have

N% ‚
ƒ + +% + = •.
N p

144
MSCPH501

This is the required equation of which we have to find the solution.

Now we move on to study the techniques to solve a first order linear differential equation.

5.4.2 Solution of a First order linear differential equation

A Solution of a differential equation (in the variables x and y) is a relation between x and y
which, if substituted into the differential equation, gives an identity.

There are three main techniques for solving linear differential equations depending on the form.

5.4.2.1. Separation of variables method: If the first order differential equation is of the form

NJ
= h(I, J).
NI
F(x,y) can be expressed as F(x,y) = X(x)Y(y) (separation of variable form).
Then the equation can be expressed as

NJ
= a(I)b(J)
NI

= a(I)NI.
•<
c(<)

The equation can thus be solved by integrating both sides as

NJ
â = â a(I)NI.
b

Examples

i) J @ = Œ .
1. Solve the following differential equation using separation of variable method
c<

ii) x2yy’= ey, x≠0.

Sol. i) The equation can be solved by separation of variables method we separate the variables as

NJ 4J
=
N

NJ N
= .
4J

Integrating this we get

145
MSCPH501

NJ N
â =â + p1
4b
ln|J|
= r4| | + p6
4

ln|J|
− r4| | = p6
4
6
r 4 (J c ( − r4| | = p6

6
Jc
ln • • = p6

6
Jc
= ŽÁ

6
Jc = ŽÁ

J = p c.

This is the general solution of the given differential equation which defines a family of solution
curves corresponding to various initial conditions.

Sol. ii) x2yy’= ey.


This equation can be written as

J 5<
NJ = I 5/ NI.

Now the variables are separated, now we integrate it

âJ 5<
NJ = â I 5/ NI

−(1 + J) 5<
+ p = −I 56

where C is a constant of integration.


The solution can thus, be expressed implicitly in the form as

I(J + 1) = (1 + pI) <


.

146
MSCPH501

5.4.2.2 Using Integrating factor: This method is used for those first order linear differential
equations, which are in standard for as

J @ + O( )J = ‚( ).

The ODE can be solved using an integrating factor as % = ´ ·(Œ)•Œ .

Next step is to multiply both sides by this integrating factor

´ ·(Œ)•Œ jJ @ + O( )Jl = ´ ·(Œ)•Œ ‚( )

j ´ ·(Œ)•Œ Jl@ = ´ ·(Œ)•Œ ‚( )

´ ·(Œ)•Œ J = â ´ ·(Œ)•Œ ‚( )N + p.

Now solving the integration on R.H.S and dividing both sides by the integrating factor gives the
general solution of the equation.

Examples

i) J @ − 2 J =
Q.2. Solve the following equations using Integrating factor method

ii) J @ + = =Â .
_< dd
=

Sol. i) J @ − 2 J =

´ 5/Œ•Œ

Multiplying both sides by ´ 5/Œ•Œ = 5Œ


We first find the integrating factor as I. F. =
&

NJ
We get
´ 5/Œ•Œ − ´ 5/Œ•Œ 2 J= ´ 5/Œ•Œ
N
N
e ´ 5/Œ•Œ Jf = ´ 5/Œ•Œ
N

´ 5/Œ•Œ J = â ´ 5/Œ•Œ N

5Œ &
J=â 5Œ &
N

J=p −
Œ& 6
/
.

147
MSCPH501

Where C is constant

Sol. ii) J @ + =
_< dd
= =Â
The integrating factor I.F = ´d•= = 1" = = I _ .
 Â

Now we multiply both sides of the differential equation by this integrating factor

3J =
I _ jJ @ + l = I_ _
I I
NJ
I_ + 3I / J = =
NI

(I _ J) =
• =
•=
.

Integrating both sides we get

I_J = â =
NI + p.

Which gives the solution as


=
+p
J= .
I_

5.4.2.3. Change of Variable method:


This method is applied for those differential equations which gets convertible to integrable
forms under proper substitution.

i) •= = =5< ( − )
Examples
•< = <

Sol. Multiplying both sides by ey we get

NJ
<
= ( /=
− = <)
NI
NJ
<
+ = <
= /=
.
NI

= , which gives < •<


= •=
< •”
•=
Put

N
+ =
= /=
.
NI

148
MSCPH501

This equation is linear in v and x.

Put P = ex and Q = e2x

Integrating factor I. F = %. h = ´ ·•= = dd


.

Multiplying both sides of the differential equation by I.F, we get

N
dd
j + =
l= d d /=
NI

N
e dd
f= d d /=
.
NI

Integrating both sides we have

dd
= â d d /=
NI + p

Put =
=

dd
= ⠌
N +

= ( − 1 ) Œ + p = ( = − 1) dd
+p
= = − 1 + p 5d
d

<
= =
−1+p 5d d
.

5.5 Second order linear differential equation


A second order linear differential equation is one in which the highest order derivative occurring in the
equation is 2 and the coefficients are functions of only x. The general form of the second order linear
differential equation can be written as

J @@ + (I)J @ + “(I)J = B(I)

This is an example of inhomogeneous differential equation of second order; the homogeneous differential
equation of second order is obtained if Q(x) becomes zero.

J @@ + (I)J @ + “(I)J = 0

The second order linear differential equations are used to model many situations in physics and
engineering. The behavior of simple models such as spring mass - system and an LCR circuit can
be studied using the differential equations involving both single as double derivatives, these can

149
MSCPH501

then be used to approximate other more complicated situations such as the bonds between atoms
or molecules are often modeled as springs that vibrate, as described by these same differential
equations.

5.5.1 The solution of the differential equations of second order


The two forms of the second order differential equations as described above i.e. non-
homogeneous and homogeneous differential equations are related to each other and there is an
important connection between the solution of a nonhomogeneous linear equation and the solution
of its corresponding homogeneous equation. The two principal results of this relationship are as
follows:

Theorem 1:
If y1, y2 _ _ _ _ _ _ yn are n linearly independent solutions of the differential equation
yn + a1yn-1 + a2yn-2_ _ _ _ + any = 0, then y = c1y1 + c2y2 + _ _ _ _ _ + cnyn is also its solution,
where c1, c2, _ _ _ _ _ _cn are arbitrary constants.

Theorem 2:
If y(x) is any particular equation of the linear non homogeneous equation, and if yn(x) is the
general solution of the corresponding homogeneous equation, then the general solution of the
linear non homogeneous equation is the linear sum of yn(x) and the particular solution of given
non- homogeneous equation.

The general solution of the homogeneous linear differential equation part is called the
complementary function (C.F) of the non - homogeneous equation whereas the other part is the
particular integral (P.I) of the equation.

5.5.2 Complementary function:


For a non-homogeneous differential equation,the complementary function is the solution of the
differential equation with the right hand side term replaced by zero. To find C.F we need to first
find the auxiliary equation.

The Auxiliary equation (Characteristic equation): The equation obtained by equating to zero the
symbolic coefficient of y is called the auxiliary equation.

Steps for finding Auxiliary equation

1. Replace y by 1.
•<
2. Replace •= by m
•& < •g <
3. Replace •= & by m2 and so on replace •= g
4. By doing so we have an equation in m of degree n called auxiliary equation.

5.5.3 Rules for finding the complementary function

150
MSCPH501

1. Write the corresponding characteristic equation for the given differential equation.
2. The auxiliary equation would be an equation in m of degree n, so it will give n values of
m on solving.
3. If m1, m2 , …….. mn are the roots of the auxiliary equation, then the complimentary
function depends upon the nature of the roots of the auxiliary equation. The different
cases which arise are discussed as follows.
CASE I

When the roots of the Auxiliary equation are real and distinct: In this case the general solution of
the homogeneous differential equation comprises of only complementary function.

J = p6 Á = + p/ & = .
The general solution is given as

Example:

Solve the equation J @@ + J @ − 6J = 0.

The auxiliary equation is q/ + q − 6 = 0.

The roots of this equation can be obtained as

q/ + 3q − 2q − 6 = 0 .
Which gives m1 = 2 and m2 = -3.
Since the roots are real and distinct the solution of the differential equation would be

J (I) = p6 /= + p/ 5_= .
This solution can be verified by differentiating and substituting in the original differential
equation to get zero.

CASE II

equation is written as J(I) = p6 = + p/ I = , where m is the common root.


When the roots of the auxiliary equation are equal: In this case the solution of the differential

Solve the equation 4J @@ + 12J @ + 9J = 0.


Example:

The auxiliary equation of this differential equation can be written as

4q/ + 12q + 9 = 0.

Which can be factored as (2q + 3)/ = 0.

The two roots of this equation are same and thus m = -3/2.

Thus the solution of the given differential equation would be

151
MSCPH501

_ _
J(I) = p6 5 =
/ + p/ I 5 =
/ .

CASE III

When two roots of the auxiliary equation are imaginary: In this case the solution of the

q6 = - + ä 4N q/ = - − ä then the solution of the differential equation would be


differential equation involves sine and cosine function, if the roots of the auxiliary equation are

h= (p
J(I) = 6 äI + p/ 4äI).

J @@ − 6J @ + 13J = 0.
Example:
Solve the equation

q/ − 6q + 13 = 0.
The auxiliary equation of the differential equation would be

We can obtain the roots of this equation as

−“ ± √“ / − 4
q= .
2
Which gives

q= = =3 ±2 .
a ±√_a5š/ a ±c`
/ /
The solution of the differential equation would be
_= (p
J(I) = 6 2I + p/ 42I).

CSAE IV

When roots of the auxiliary equation are repeated imaginary: If the two roots of the differential
equation are m1 = m2 = α + iβand m3 = m4 = α –iβ, then the complementary function will be
h= (p
J(I) = 6 + p/ I) äI + (p_ + pc I) 4äI.

This would be the case for fourth order linear differential equation. Since we are discussing
second order linear differential equation this case is not elaborated.

5.5.4 Rules for finding the Particular Integral

For a non -homogeneous second order linear differential equation the solution comprises of both
the complimentary function and the Particular integral part. Depending on the nature of the term
Q(x) on the R.H.S of the differential equation we have the following cases for the solution.

Case I: When Q = eax


First find the P I as : A. %. = B
6
•(i)

152
MSCPH501

= ,O N N Q( ) ≠ 0.
6 ˜= 6 ˜=
•(i) •(˜)

Case II: When Q = sin (ax + b) or cos (ax +b)


Replace D2 by –a2.
If denominator reduces to a constant, it will be the final step in finding P.I.
If denominator reduces to D only, we are then only to integrate the given function Q once.
If denominator reduces to a factor of the form αD + β then operate by its conjugate αD – β on
both numerator and denominator from left hand side such as

-à − ä 1
æ sin( I + “)ç .
-à − ä -à + ä

By doing so, denominator will become - / à/ − ä / which in turn reduces to a constant by


replacing D2 by –a2.
Now operating sin (ax + b) by αD – β we can find the required particular integral.

CASE III: When Q = xm, m being a positive integer.


Here A. % = •(i) I .
6

Take out the lowest degree term from f(D) to make the first term unity (so that Binomial theorem
for a negative index is applicable). The remaining factor will be of the form

Examples:

Q.1. Solve the following differential equations


− 6 •= + 5J = 0 .
•& < •<
•= &
i)
+ 4 J = 0.
•& <
•= &
ii)

Sol.i) First we write the corresponding characteristic equation

k2 -6k +5y = 0.

The roots of this equation are k1 = 1 and k2 = 5.


Since the roots are real and distinct, the solution has the form

y(x) = c1ex + c2e5x.

Sol. ii) The characteristic equation is


k2 + 4i = 0.

The roots of the equation are k1 , k2 = ±2i√i .

153
MSCPH501

These can be expressed in trigonometric form as

i = sin π/2 + i cos π/2 =eiπ/2 which implies √i = eiπ/4 = sin π/4 + i cos π/4.

The roots of the equation can thus, be written as

k1 = 2i (sin π/4 + i cos π/4) and k2 = -2i (sin π/4 + i cos π/4)

k1= 2i (1/√2 + i/√2) and k2 = -2i (1/√2 + i/√2)

k1 = -2/√2 + 2i/√2 and k2 = -2i/√2 + 2/√2

k1= -√2 + √2 i and k2 = √2 - √2i .

The general solution of the given differential equation would thus be

y (x) = C1e (-√2 + √2 i) x + C2e (√2 - √2i) x

where C1 and C2 are arbitrary constants.

5.6 Summary
In the present unit, we studied about forms of the differential equations. The different forms of
the differential equations were introduced, such as linear, non-linear, ordinary and partial
differential equations. The order of the differential equations was defined and difference between
first order and second order differential equation was elaborated. We also presented some first
and second order linear differential equations and studied different approached to determine their
solution.

5.7 Glossary
Equation: relationship between dependent and independent variable.

Differential equation: equations involving dependent variable and their derivatives with respect
to independent variables.

Linear differential equation: no multiplication among dependent variables.

Ordinary differential equation: Equations involving only one independent variable.

Auxiliary equation: An equation obtained from the standard form of a linear differential
equation by replacing the right members by zero.

Complementary Function:general solution of auxiliary equation of linear differentialequation.

Particular Integral: Any solution to a differential equation

154
MSCPH501

5.8 References:
1. Arfken, G. "A Second Solution." §8.6 in Mathematical Methods for Physicists, 3rd
ed. Orlando, FL: Academic Press, pp. 467-480, 1985.

2. Boyce, W. E. and DiPrima, R. C. Elementary Differential Equations and Boundary Value


Problems, 4th ed. New York: Wiley, 1986.

3. Morse, P. M. and Feshbach, H. Methods of Theoretical Physics, Part I. New York: McGraw-
Hill, pp. 667-674, 1953.

4. Boyce, W. E. and DiPrima, R. C. Elementary Differential Equations and Boundary Value


Problems, 5th ed. New York: Wiley, 1992

5.9 Suggested Readings


1. An Introduction to Ordinary Differential equations by Earl A Coddington, Dover Publications
Inc.; New edition (1 March 1989).

2. Differential Equations with Applications and Historical Notes by George Simmons, McGraw Hill
Education; 2nd edition (1 July 2017).

5.10 Terminal Questions


Short Answer type questions

+ 4J = Ã 4I ?
•<
•=
Q.1. What is the order of the differential equation

Q.2. The process of formation of the differential equation is given in the wrong order,
1) Eliminate the arbitrary constants.
2) Differential equation which involves x,y,dydx.
3) Differentiating the given equation w.r.t x as many times as the number of arbitrary constants.
Write the correct order.

Q.3. Consider the differential equation •Œ & − 3 •Œ + 2I = 0, if x =0 at t=0, and x=1 at t=1,
•& = •=

determine the value of x at t=2.

=
•< = <
•= =
Q.4. Find the Particular solution of the differential equation , y(1) = 1.

Q.5. Solution of the second order differential equation •Œ & − 5 •Œ + J = 0 is y = e2t, the value
•& < •<

of k is.

155
MSCPH501

Long Answer type questions

Q.1. Solve the Differential equation I = I / + 3J.


•<
•=

Q.2. Find the Solution of •= & − 2 +J =


•& < •< dd
•= =& 6

Q.3. Find the Solution of •= & + 7J = 0


•& <

Q.4. Find the Solution of 3 −2 − J = 2I − 3.


•& < •<
•= & •=

+ IJ = I, J(0) = −6.
•<
•=
Q.5. Solve the initial value problem

5.11 Answers

Answers to Short Answers type questions


1. First Order
2. 3,1,2
3.e2+ e
4. y = x log |x| + x
5.k = 6

Answer to Long Answer type questions

1. y = -x2 + Cx3
ln(1 + I / ) + I 4I
6 = =
2. y = Aex + Bxex - /
3. J = `√–=
+) 5`√–=

4. J = +) − 2I + 7
Á
= 5 =
Â
d&
5.J = 1 − 7 5
&

156
MSCPH501

__________________________________________________
Unit 6: Partial Differential equations
__________________________________________________________________

Structure
6.1 Objectives
6.2 Introduction
6.3 Partial Derivative
6.4 Examples of Partial Differential equations
6.5 Order of a Partial Differential equation
6.6 Solution of Partial Differential Equation
6.7The Laplace equation
6.7.1 The Dirichlet Boundary Condition
6.7.2 The Neumann or second type boundary condition
6.7.3 Robin’s type condition
6.8 Solution of 2D Laplace equation with Boundary Condition
6.9 The Wave Equation
6.10 The Heat equation
6.11 Summary
6.12 Glossary
6.13 References
6.14 Suggested Readings
6.15 Terminal Questions
6.16 Answers

157
MSCPH501

6.1 Objectives
The learning objectives of this unit are
1. To introduce students with Partial Differential Equations
2. To derive Heat and wave equations
3. To find solutions of PDE using boundary conditions.

6.2 Introduction: Partial differential equations describe the behavior of ,any engineering
phenomena, such as wave propagation, fluid flow (air or liquid), vibration, mechanics of solids,
heat flow, electric field, diffusion of chemicals etc. Many of the problems of mathematical
physics involves the solution of partial differential equations. In fact, a single partial differential
equation may apply to a variety of physical problems.
A partial differential equation is an equation involving functions and their partial derivatives,
such as the heat equation, the wave equation and the Laplace’s equation.
These are termed, as partial differential equations since these involve partial derivatives, which
are derivatives of the functions having more than one variable.
To start with the partial differential equations we first describe the partial derivative and then try
to study some important partial differential equations such as the heat equation, the wave
equation and the Laplace’s equation.

6.3 Partial Derivative: For a function dependent on more than one variables, the change in
the function with respect to one variable keeping the other variables constant is represented by
partial derivative. For example let f be a function of x,y,z and t, represented as f(x,y,z,t), the
change in function f with respect to change in variable x, keeping the other variables constant is
jk
represented by, jl .

6.4 Examples of Partial differential equations:

∂/ U ∂/
− c /
=0
∂t / ∂x /

∂/ U ∂/
+ c /
=0
∂y / ∂x /

›¢ › /¢
(I + J )
/ /
+ − 3¢ = 0
› ›IðJ

› /¢ › /¢ › /I
/
+¼ ½ + = I/ + J/.
›I / ›IJ ›J /

158
MSCPH501

The partial differential equations are classified based on their order, form and nature as first,
second, third or higher orders, linear, non-linear, homogeneous and non-homogeneous PDE’s
respectively.

6.5 Order of a Partial differential equation: The order of the highest derivative
term in any partial differential equation is the order of the PDE. All the above equations given
above are second order partial differential equations. Depending on the nature of the problem,
first or second order PDE’s are used to describe it. For example, the gas flow problem, the traffic
flow problem, the phenomena of shock waves, motion of wave fronts, Hamilton Jacobi theory,
nonlinear continuum mechanics and quantum mechanics etc. can be studied using first order
PDE’s, whereas the problems of fluid mechanics, heat transfer, rigid body dynamics and
elasticity are modelled by second order PDE’s.

Some of the examples of first and second order PDE’s are

ݢ ݢ
+ =0 ; 4 O ‚¢ 4
›I ›J

› /¢ › /¢
+ =0; ƒ Or ‚¢ 4 4 2à
›I / ›J /

›¢ › / ¢
− =0; o ‚¢ 4
› ›I /

› /¢ › /¢
− = 0; p ‚¢ 4.
› / ›I /

Linear PDE: If the dependent variable and all its partial derivatives occur linearly, i.e. degree at
most one, in any PDE then such an equation is called linear partial differential equation,
otherwise a non-linear PDE, in above equations all except the last are linear PDE.

Quasi – Linear PDE: A partial differential equation in which all the terms of the highest order
derivatives of dependent variable occurs linearly. The coefficients of such terms are functions of
only lower order derivatives of the dependent variables.

Homogeneous PDE:A linear differential equation is termed homogeneous if the dependent


variable and its partial derivatives appear in terms with degree exactly one, or it has no,non-
differential terms. If it has one or more non-differential term, it is non-homogeneous or
heterogeneous PDE.

6.6 Solution of a partial differential equation


The solution of Ordinary differential equation contains arbitrary constants, whereas the solution
to partial differential equations contains arbitrary functions. While an ODE of order m has m
159
MSCPH501

linearly independent solutions, a PDE has infinitely many solutions. These are consequences of
the fact that a function of two variables contains immensely more of information than a function
of only one variable. Some of the method for solving PDE are:
Separation of variables method
Integral solutions employing a green function
Use of Integral Transforms
Numerical calculations

We will now discuss some important partial differential equations.

6.7 The Laplace Equation: The steady state of a field that depends on two or more
independent variables, which are typically spatial. The Laplace equation arises as a steady state
=0=
œq œ& q
œŒ œŒ &
problem for the heat or wave equations that do not vary with time so that
Laplace equation in 2D.

The physical problems in which the Laplace equation arises are


i) 2D steady state heat conduction
ii) Static deflection of a membrane
iii) Electrostatic potential.

The general form of a 2D Laplace equation is

› /¢ › /¢
+ = 0.
›I / ›J /
We can relate this equation to the steady state heat equation, which do not vary with time.
Secondly, we can also relate it to the equation of continuity for incompressible potential flow.

The Laplacian represents the flux density of the gradient flow of a function. For instance, the net
rate at which a chemical dissolved in a fluid moves toward or away from some point is
proportional to the Laplacian of the chemical concentration at that point.

The three-dimensional Laplace equation is

› /¢ › /¢ › /¢
+ + = 0.
›I / ›J / ›K /

Since there is no time dependence in the Laplace’s equation, there is no initial condition to
satisfy by their solutions, However there should be certain boundary condition on the bounded
curve in which the differential equation is to be solved.

Typically, there are three types of boundary conditions given as

6.7.1 The Dirichlet Boundary Condition:


This boundary condition when imposed on an ODE or PDE specifies the values that a solution
needs to take along the boundary of the domain.

160
MSCPH501

The solution u (x,y) of the Laplace equation in a domain D is specified by boundary dS as

u (x, y) = f (x, y) on dS

where f (x, y) is a given function. The question of finding solutions to such equations is known
as Dirichlet problem and is expressed as

∇/ ¢(I, J) = 0 4 à; ¢(I, J) = Q(I, J) 4 NÃ.

6.7.2 The Neumann or second type boundary condition:


It is the boundary condition which when imposed on an ODE or PDE, specifies the values in
which the derivative of a solution is applied within the boundary of the domain.

ݢ(I, J)
= t(I, J), Q I, J ∈ NÃ .
›4

In physical terms, the normal component of the solution gradient is known on the boundary. In
steady state heat flow problem, Neumann boundary condition means the rate of heat loss or gain
through the boundary point is prescribed.
The Laplace equation together with Neumann BC are called the Neumann BVP or the Neumann
problem and is written as

ݢ(I, J)
∇/ ¢(I, J) = 0 4 à; = t(I, J) Q (I, J) ∈ NÃ.
›4
The Neumann problem have no solution unless the average value of the function g on dS is
assumes zero. This assumption is known as the compatibility condition.

6.7.3 Robin’s type condition: This boundary condition also called as the third type
boundary condition, when imposed on an ordinary or a partial differential equation, is a
specification of a linear combination of the values of a function and the value of its derivative on
the boundary of the domain.

ݢ(I, J)
+ (¢ − t) = 0.
›4
C is a constant and g is a function, which can vary over the boundary. The Laplace equation
together with Robin’s conditionis known as Robin’s boundary value problem or mixed problem.

6.8 Solution of 2D Laplace equation with Boundary Condition


Consider the geometry of a rectangle given by 0≤ x ≤ L, 0≤ y ≤ H. For this geometry the Laplace
equation along with the four boundary conditions is

N/ ¢ › /¢
∇/ ¢ = + =0
›I / ›J /

u(0.y) = g1(y);

161
MSCPH501

u(L, y) = g2(y)
u(x, 0) = f1(x)
u(x, H) = f2(x).

There is no initial condition here and both the variables are spatial variable and occur in a 2nd
order derivative, so we need two boundary conditions for each variable.
The PDE is both linear and homogeneous but the boundary conditions are only linear but not
homogeneous.
ogeneous. To solve the Laplace equation completely it need to be solved four times. Each
time the equation is solved, one of the boundary condition can be non homogeneous while the
remaining three will be homogeneous.
The four conditions are as represente
represented by the below figure

These four equations can be solved by separation of variable method. We proceed by solving for
u1as follows.

162
MSCPH501

¢6 (I, J) = a(I)b(J).
The Laplace equation is

N/¢ › /¢
+ = 0.
›I / ›J /

a @@ b @@
Substituting the solution in the equation, we get
+ = 0.
a b

On separating the variables we get

a @@ b @@
=− = .
a b

Which gives

a @@ − a = 0

b @@ + b = 0.

Substituting λ2 = -k .
The solution u(x,y) can be written as

¢(I, J) = ( p +I + )à 4 +I)(pp ℎ +J + àà 4ℎ +J).

With the boundary conditions


u(0,y) = 0
u(L, y) = 0

we get

A= 0 and λn = nπ/L, n= 1,2,3

So the solution becomes

4ÙI 4ÙJ 4ÙJ


¢© (I, J) = à 4 ž ©p ℎ + )© à 4ℎ Ÿ.
ƒ ƒ ƒ

The boundary condition u(x, H) = 0 gives

4Ùo
p‡ ℎ
)© = − ƒ
4Ùo ©.
Ã%4ℎ4 ƒ

163
MSCPH501

So the solution can now be written as

r r
4ÙI4ÙJ 4ÙJ
¢© (I, J) = ó ¢© (I, J) = ó à 4 ž ©p
+ )© Ã 4ℎ ℎ Ÿ
ƒ ƒ ƒ
©Å6 ©Å6
4Ù(o − J)
4ÙI Ã 4ℎ
r r

¢© (I, J) = ó = ó © Ã 4 ƒ .
ƒ 4Ùo
Ã%4ℎ4
©Å6 ©Å6 ƒ

The remaining boundary condition u(x, 0) = f1(x) can be used to find the value of An
r
4ÙI
Q(I) = ó ©Ã 4
ƒ
©Å6

2 s 4ÙI
= â Q(I)Ã 4 NI.
©
ƒ g ƒ

Similarly other solutions u2, u3, u4 can be determined and the overall solution can be written as

u = u1+ u2 + u3 + u4 .

6.9 The Wave equation: The Vibrating string


An important second order partial differential equation for description of waves.

Consider a vertical string of length L, that has been tightly stretched between two points at x = 0
œ<
and x = L.Since, the string is tightly stretched, we can assume that the slopeœ= of the displaced
string at any point is small. The string never getsfar away from its equilibrium position
Consider a point x on the string in this equilibrium position i.e. the location of the point at t = 0.
As the string vibrates this point will be displaced both vertically and horizontally, however if we
assume that at any point the slope of the string is small then the horizontal displacement will be
very small in relation to vertical displacement.
So at any point x on the string the displacement will be purely vertical, let this displacement be
u(x,t). we now write the 1D wave equation as
The wave equation in one dimension case can be derived from Hook’s law as follows.
Imagine an array of little weights of mass m interconnected with massless springs of length l and
spring constant k.

164
MSCPH501

Here the dependent variable u(x) measures the distance from the equilibrium of the mass situated
at x, So that u(x) essentially measures the magnitude of a disturbance, travelling in an elastic
material.

The forces exerted by the mass m at the location x + l are

›/
htduŒU© = q. ( ) = q. ¢(I + r, )
› /

hvUU# = h= /` − h= = j¢(I + 2r, ) − ¢(I + r, )l − j¢(I + r, ) − ¢(I, )l

hvUU# = j¢(I + 2r, ) − 2¢(I + r, ) − ¢(I, )l.

Equating the two forces, we get

›/
q. / ¢(I + r, ) = j¢(I + 2r, ) − 2¢(I + r, ) − ¢(I, )l

›/
¢(I + r, ) = j¢(I + 2r, ) − 2¢(I + r, ) − ¢(I, )l.
› / q

If the array of weights consists of N weights spaced evenly over the length L = Nh of total mass
M = Nm, and the total spring constant of the array K = k/N we can write the above equation as

›/ wƒ/ j¢(I + 2r, ) − 2¢(I + r, ) − ¢(I, )l


)
¢(I + r, = .
› / R ℎ/

Under the limits N → ∞ and h → 0, the equation attains the form

› / ¢(I, ) 1 › /¢
= .
› / / ›I /

Where = %xs& is the propagation speed of the wave.

The generalized 1D wave equation can thus be written as

165
MSCPH501

› /¢ 1 › /¢
= .
› / / ›I /

For the wave equation, the only boundary condition is that of the prescribed location of the
boundaries.

The 2 D and 3D version of the wave equation is

› /¢
= ∇ ¢.
/ /
› /

6.10 The Heat Equation: It is the equation, which governs the temperature distribution
in an object. There are in fact several forms of the heat equation; we will focus on one of the
forms in 1D and 3D. The heat equation is, derived using the principle of conservation of energy
and the fact that heat flows from hot regions to cold regions.
The 1D heat equation: Temperature distribution in a rod of length L
Consider a uniform rod of length l with non uniform temperature lying on the x axis from x = 0
to x = l.
Let u(x,t)denotes the temperature at x at a time t, and is assumed constant throughout the rod at
each time t.

By the principle of energy conservation the net change of heat inside the segment between x and
x + Δ x is equal to the net heat flux (influx at x and out flux at x + Δx) across the boundaries and
the total heat generated between x and x + Δ x.
If ‘s’ is the, specific heat capacity of the rod, ‘k’ is the thermal conductivity of the rod, ‘ρ’ the
density of the rod, ‘A’ the cross sectional area of the rod and f(x,t) is the external heat source,
then we can have

Total amount of heat inside the segment between x and x + Δ x at time t =´= Þ ¢(I, )NI.
= ∆=

Net change of heat inside the segment = •Œ ´= Þ ¢(I, )NI = ´= Þ ¢Œ (I, )NI .
• = ∆= = ∆=

j¢(I + ∆ I , ) − ¢(I, )l

•=
Net heat flux across the boundaries =

= „Y¢= (I + ∆ I, ) − ¢= (I, )Z….


Where ¢= = .
•q
•=

166
MSCPH501

´= Q(I, )NI.
= ∆=
Heat generated due to external heat source between x and x + Δx =

By principle of conservation of energy we now have

´= Þ ¢Œ (I, )NI = „Y¢= (I + ∆ I, ) − ¢= (I, )Z… + ´= Q(I, )NI .


= ∆= = ∆=

Applying mean value theorem for integral we have

O ¢Œ (z6 , )ΔI = „Y¢= (I + ∆ I, ) − ¢= (I, )Z… + Q(z/ , )ΔI.

Where z6 , z/ ∈ (I, I + ∆I) hence

„Y¢= (I + ΔI , ) − ¢= (I, )Z… Q(z/ , )


¢Œ (z6 , ) = + .
O ΔI O

In the limit Δ x → 0, we arrive at

¢Œ (I, ) = - / ¢== (I, ) + h(I, ).

Where - / = is called the thermal diffusivity of the rod and h(I, ) = Q(I, )is called the heat
# 6
•· •·
source density.

Now there arises three cases

1. The case when the lateral boundary is not insulated as above, and heat is allowed to flow in and out
across the lateral boundary at a rate proportional to the difference between the temperature of the rod
u(x,t) and the surrounding medium u0, the conservation of heat principle yields

¢Œ = - / ¢== − ä(¢ − ¢U ), ä > 0.

surrounding medium, ä is the constant of proportionality.


The heat loss or gain is proportional to the difference between the temperatures of the rod and

167
MSCPH501

2. If, the material of the rod is uniform then k is independent of x. for some materials the value of k
depends on the temperature u and hence the resulting heat equation is nonlinear and given as:

1 › ›¢
¢Œ = (¢) .
Þ ›I ›I

3. If, the material is non-homogeneous i.e one-half the rod is made of one material and the other half of
different materials, the diffusion within the rod depends on x. The heat equation is written as

¢Œ = - / (I)¢== , 0<I<r.

With


}l = ~}D,
•<I< ,
E=
}E , €/E < I < r,

Where α1 and α2 are the thermal diffusivity of the two materials respectively.

6.11 Summary
In the present unit we studied the partial differential equation and its different forms. We also
studied different boundary value problems and methods to solve the partial differential
equations. The Laplace equation, the Heat equation and the wave equation for one dimensional
and two dimensional cases were discussed in detail.

6.12 Glossary

Partial derivatives: Derivative of a multivariable function w.r.t any one, keeping others fixed.

168
MSCPH501

Linear PDE: Dependent variable and all its derivatives occur in first order.

Quasi-Linear PDE: All terms of highest order derivative of dependent variable occurs linearly.

Homogeneous PDE: A differential equation is homogeneous if it is a homogeneous function of


the unknown function and its derivatives

6.13 References

• R. Courant and D. Hilbert, Methods of Mathematical Physics. Volume 2. Partial


Differential Equations, Wiley-VCH, 1989.
• L. C. Evans, Partial Differential Equations, American Mathematical Society, Providence,
1998.
• D. Bleecker and G. Csordas, Basic Partial Differential Equations, Van Nostrand Reinhold,
New York, 1992.
• E. Kreyszig, Advanced Engineering Mathematics, Wiley, 2011.

6.14 Suggested Readings

• I.N. Sneddon, Elements of Partial Differential Equations, Dover Publications, New


York, 2006.
• Peter J. Olver, Introduction to Partial Differential equations, Springer, New York
2014.

6.15 Terminal Questions

Short Answer type questions

1. The nature of the partial differential equations œ= & + 4 + = 0 is


œ& q œ& q œ& q
œ=œ< œ< &

i) elliptic

169
MSCPH501

ii) parabolic

iii) hyperbolic

iv) none of the above.

=
œ& q /œ q
&

œŒ & œ= &
2. The equation is known as

i) one dimensional heat equation

ii) One dimensional wave equation

iii) Laplace equation

iv) Poisson’s equation.

= 9 œ< &
œ& q œ& q
œ= &
3. Using substitution which of the following are solutions of the PDE

i) Cos (3x-y)

ii) x2 + y2

iii) Sin (3x-3y)

iv) e-3πySin(πy).

4. The three dimensional heat equation among following is

= žœ= & + + œ> & Ÿ


œq œ& q œ& q œ& q
œŒ œ< &
i)

ii) œŒ & = žœ= & + + œ> & Ÿ


œ& q œ& q œ& q œ& q
œ< &

iii) œŒ = žœ= + + œ> Ÿ


œq œq œq œq
œ<

= − žœ=œ< + + œ>œ=Ÿ.
œq œ& q œ& q œ& q
œŒ œ<œ>
iv)

170
MSCPH501

5. The boundary condition which include direct boundary value is

i) Dirichlet boundary condition

ii) Neumann boundary equation

iii) Forced boundary equation

iv) Discrete boundary equation.

Long Answer type questions

1. Solve œ= = 6 + ¢ using the method of separation of variables, if u(x,0) = 10 e-x.


œq œq
œŒ

2. Solve the initial boundary value problem

=3 , 0 ≤ I ≤ Ù, 0 < < ∞
œq œ& q
œŒ œ= &

u(0,t) = u(π,t) = 0, 0 < < ∞

u(x,0) = 3Sin 2x – 6Sin 5x, 0 ≤ x ≤ π.

=
œ& q /œ q
&

œŒ & œ= &
3. Consider the initial value problem , -∞ < x, t >∞ , IC u(x,0) = 0, ut(x,0) = Sin (x)

171
MSCPH501

6.16 Answers
Answer to short answer type questions

1. iii

2.i

3. iv

4. i

5. i.

Solution to Long Answer type questions

1. 10 e-x e-t/3

2.u(x,t) = 3e-12t Sin(2x) – 6 e-75t Sin(5x)

4. ¢(I, ) = − jp (I + )−p (I − )l.


6

172
MSCPH501

______________________________________________
UNIT 7: LEGENDRE’S AND BESSEL’S
DIFFERENTIAL EQUATIONS
Structure
7.1 Objective
7.2 Introduction
7.3 Definitions
7.3.1 Differential equations of second order
7.3.2 Real and analytic function
7.3.3 Regular and Singular points
7.3.4 Leibniz’s rule
7.3.5 Generating function
7.4 Series solution of differential equation
7.4.1 Power series method
7.4.2 Theorem
7.4.3 Frobenius Method
7.5 Legendre differential equation

7.5.2 Legendre Polynomials „… (l)


7.5.1 Solution of Legendre equation

7.5.3 Rodrigue’s Formula

7.6 Properties of Legendre Polynomial „… (l)


7.6.1 Generating function
7.6.2 Orthogonality
7.6.3 Recurrence relations
7.7 Bessel’s differential equation

7.7.1 Solution of Bessel’s equation


7.7.2 Recurrence relations
7.7.3 Generating function
7.7.4 Orthogonality

7.8 Summary

173
MSCPH501

7.9 Answer to SAQs

7.10 References / Bibliography

7.11 Terminal and model Questions

7.1 Objective
The plan of this unit is as follows:

• First, I am going to briefly review linear second order differential equations which have
already been introduced to you in unit 5(Ordinary differential equations of First and
Second order).
• Second, I am going to introduce power series methods to solve differential equations.
• Third, we will apply the power series methods to solve above two special differential
equations.
• We will learn other methods to generate polynomial like generating function, Rodrigues
formula.
• We will discuss various properties of the polynomials such as orthogonality, recurrence
relations.

7.2 Introduction
You have already studied the methods to solve first and second order differential equations. The
solution of which are continuous i.e. exists for all the values over the real line. However there are
certain differential equations whose solution exists only in a defined range. In this unit we are
going to learn how to solve linear and homogeneous second order differential equations using
power series method. Our focus will be to seek solution of two of the four special forms of
second order differential equations which are linear and homogeneous in nature. The differential
equations are:

(1 − I / ) J @@ − 2I J @ + 4(4 + 1)J = 0.
a) Legendre Differential Equation:

b) Bessel’s differential equation:

I / J @@ + I J @ + (I / − † / )J = 0.

The solution of the differential equation turns out to be a polynomial which plays an important
role as a part of solution to various important problems of different fields. We see the existence

174
MSCPH501

method. For weight function as 1the Gauss quadrature method turns out to be Gauss-Legendre
of Legendre polynomials in numerical analysis as a solution of Gaussian quadrature integration

quadrature. Bessel’s polynomial occurs as one of the solution of partial differential equation of a
circular membrane. We also see presence of Bessel polynomial in the intensity distribution of
very interesting phenomena well known as Airy’s pattern (It is diffraction pattern of a star light
when the star is seen through a circular lens/aperture). So let’s proceed to learn detail description
of these complex but beautiful differential equations.

7.3 Definitions
In this section we will review and learn important definitions which will be used in
understanding the mathematical development of the differential equations.

7.3.1Differential equations of second order

A second order linear differential equation is a differential equation that can be written in the
following form:

Q(I)J @@ + t(I)J @ + ℎ(I) J = i(I). (1)

WhereJ @@ = •= & , J @ = 4N Q(I) ≠ 0. In any linear differential equations there is no


•& < •<
•=
products of the function J(I), and its derivatives and no terms of J and its derivatives with
•< /
power other than first power, for example: terms like J / , J •= , ž•= Ÿ J •= & .. are not
•< •& <

part of linear differential equations. The coefficients Q(I), t(I), ℎ(I) 4N i(I) can be zero or
non-zero functions, constant or non-constant functions, linear or non-linear functions. Only ‡(l)
and its derivatives are used to determine whether equation is linear or not.

We can write equation (1) as follows

t(I) @ ℎ(I) i(I)


J @@ + J + J =
Q(I) Q(I) Q(I)

J @@ + O(I)J @ + ‚(I)J = (I). (2)

If (I) = 0 then equation (2) is homogeneous equation.

7.3.2 Real and analytic function

A real function Q(I) is called ANALYTIC at a point I = Ig if it can be represented by a power


series in powers of (I − Ig ) with radius of convergence + > 0, the convergence interval is
(Ig − +, Ig + +).

175
MSCPH501

7.3.3 Regular and Singular points

A regular point of J @@ + O(I) J + ‚(I)J = 0 is a point Ig at which the coefficients O(I) and
‚(I) are analytic i.e. converges to a finite value as I → Ig ) . Then the power series method can
be applied. If Ig is not regular, it is singular point (i.e.lim=→=Ý O(I)and lim=→=Ý ‚(I)do not
converge to finite value).

Again consider a homogeneous second order differential equation of the form

J @@ + O(I)J @ + ‚(I)J = 0.

If at I = Ig the coefficients O(I) and ‚(I) are not analytic i.e. I = Ig is a singular point,

^ˆ‰ (l − l• ) Š(l) → k‹}‹Œ• Ž|€••


however the condition for weak singularity is satisfied, which is as follows
l→l•

^ˆ‰l→l• (l − l• )E •(l) → k‹}‹Œ• Ž|€••


thenI = Ig is called as regular singular point.

E (l − E) l‡ + ‘l ‡ + (l − E)‡ = •
SAQ1: Find the regular singular point of the differential equation
E @@ @

7.3.4 Leibniz’s rule

differential calculus. It expresses the derivative of order4 of the product of two functions.
Leibniz's rule (named after Gottfried Wilhelm Leibniz) is a generalization of the product rule in

Suppose that the functions ¢(I) and (I) have the derivatives up to order4. Then the derivative
of the product of these functions (à is the derivative operator)

The first order derivative will be

à(¢ ) = (à¢) + ¢ (à ).

The second order derivative will be

à/ (¢ ) = (à/ ¢) + 2(à¢)(à ) + ¢ (à/ ).

The third order derivative will be

à_ (¢ ) = (à_ ¢) + 3(à/ ¢)(à ) + 3(à¢)(à/ ) + ¢ (à_ ).

If we keep differentiating with increasing the order of derivative, it is easily visualized that the
terms on the right side is similar to binomial expansion with appropriate exponent on the

176
MSCPH501

derivative operator D. So in general for}Œ’ order derivative of the product of two functions
can be expanded as (with àg ¢ = ¢ and àg = , i.e. no derivative)

“} (•Ž) = (“} •)Ž + }


”D (“}5D •)(“Ž) + }
”E (“}5E •)(“E Ž) + }
”‘ (“}5‘ •)(“‘ Ž)
+ ⋯+ }
”}5D (“•)(“}5D Ž) + } (•)(“}
”} Ž) .

7.3.5 Generating function

sequence of function be •Q© –r


A generating function is a continuous function associated with a given sequence. Consider a
©Åg . The generating function of this sequence of function is defined
as
r

t(K) = ó Q© K ©
©Åg
for|I| < + and + is radius of convergence of the series. The sequence of function Q© appears as
coefficient in the series t(K). It is important that the series has a non-zero radius of convergence,
otherwise t(K) would be undefined for all I ≠ 0.

7.4 Series solution of second order differential equation


There are several methods to find the solution of second order differential equation depending on
their form. Series solution is one of the method applied to normally special kind of second order
differential equation.

7.4.1 Power series method

Power series method is the standard basic method for solving linear differential equations with
variable coefficients. It gives solution in the form of Power series.

A power series is an infinite series of the form


r

ó © (I − Ig )© = g + 6 (I − Ig ) + / (I − Ig )/ + ⋯
©Åg

If Ig = 0 then a power series will have powers of I only i.e.


r

ó © I = g + 6I + /I
/
+ _I
_
+⋯
©Åg

For a given homogeneous differential equation of the form of equation (2):

‡@@ + Š(l)‡@ + •(l)‡ = • with O(I)and ‚(I)are analytic at I = 0, the solution can be found
using power series method as follows:

177
MSCPH501

1. Assume the solution in the form of power series


r

J= ó ©I
©
= g + 6I + /I
/
+ _I
_
+⋯ (3)
©Åg

and the series obtained by term wise differentiations


r

J = ó4
@
©I
©56
= 6 +2 /I +3 _I
/
+⋯ (3.1)
©Å6

J = ó 4 (4 − 1)
@@
©I
©5/
=2 / +6 _I + 12 cI
/
+⋯ (3.2)
©Å/

2. Replace J, J @ 4N J′′ in the given differential equation.


3. Collect the like powers of I and equate the sum of the coefficients of each occurring
power of I to zero. We get equations from which the constants coefficients g , 6 , / , ….
can be determined.
4. Once the coefficients are known put it in assumed power series solution (in step 1)to get
final solution and simplify it if possible.

Let’s take a very simple example of a differential equation whose solution can be obtained from
methods explained in unit 5.

‡ +‡ =•
SAQ2:Using power series method Solve the differential equation
@@

7.4.2 Theorem

Several second order differential equations of great practical importance (e.g. Bessel equation,
etc.) have coefficients that are not analytic but singular and are such that following theorem
holds:

Any differential equation of the form

˜(l) ”(l)
‡@@ + ‡ + E ‡ = •. (™)
l l
Or

178
MSCPH501

lE ‡@@ + l˜(l)‡ + ”(l)‡ = •. (š)

With coefficientsO(I) = and(I) = , such that the function I O(I) and I / ‚(I) are
›(=) Û(=)
= =&
analytic at I = 0,(I = 0 is a regular singular point) has at least one solution that can be
represented in the form
r

J= I ó •
©I
©
= I• ( g + 6I + /I
/
+ _I
_
+ cI
c
+⋯) Ú ℎ g ≠ 0. (6)
©Åg

Where the exponent may be any (real or complex) number (C is chosen so that |• ≠ •).

This provides an extension of power series method called as Frobenius Method.The method was
known for German mathematician F.G. Frobenius (1849 – 1917).

7.4.3 Frobenius Method

To find the solution of equation (4) or (5) first expand “(I) and (I) in power series:

“(I) = “g + “6 I + “/ I / + “_ I _ + “c I c + ⋯

(I) = g + 6I + /I
/
+ _I
_
+ cI
c
+⋯

and consider the solution is given by equation (6)


r r

J = I• ó ©I
©
=ó ©I
© •
.
©Åg ©Åg

Differentiate this term by term


r

J =ó © (4 + )I © = I •56 j + 6( + 1)I + /( + 2)I / + _( + 3)I _ + ⋯ l(œ)


@ •56
g
©Åg

J =ó © (4 + )(4 + − 1)I ©
@@ •5/

©Åg

= I •5/ j g ( − 1) + 6( + 1) I + /( + 2)( + 1)I / + _( + 3)( + 2)I _ + ⋯ l(•)

Insert this in equation (5) we get

I • j g ( − 1) + 6 ( + 1) I + / ( + 2)( + 1)I / + _ ( + 3)( + 2)I _ + ⋯ l


+(“g + “6 I + “/ I / + “_ I _ + ⋯ )I • j g + 6 ( + 1)I + / ( + 2)I / + _ ( + 3)I _ + ⋯ l
+( g + 6 I + / I / + _ I _ + c I c + ⋯ )I • ( g + 6 I + / I / + _ I _ + c I c + ⋯ ) = 0. (ž)

179
MSCPH501

Equate the sum of the coefficients of each power of I i.e. I • , I • 6 , I • / , I • _ , . to zero. This
will give system of equations involving unknown coefficients ` . Here we are interested in only
equation with lowest or smallest power of l. The equation is:

(Coefficient of smallest power of I . . I • )


jC(C − D) + ˜• C + ”• l|• = •. (D•)

From equation (10) it is possible that either g is zero or the other term in square bracket is zero.
We choose C such that |• ≠ •. Therefore, we get a quadratic equation in C
jC(C − D) + ˜• C + ”• l = •. (DD)
called as Indicial equation of differential equation (4).
Let CD 4NCE be the roots of the indicial equation thenbasis of the general solution depends on
three cases:

• CASE I: distinct root not differing by an integer 1,2,3,…

J6 = I •Á ( g + 6 I + / I / + _ I _ + ⋯ ) and
The basis is:

J/ = I •& ( g + 6 I + / I / + _ I _ + ⋯ ).

CASE II: A double root : CD = CE = C = E • .


D5˜

J6 = I • ( g + 6 I + / I / + _ I _ + ⋯ )and
The basis is:

J/ = J6 (I) r4(I) + I • ( 6 I + / I / + _ I _ + ⋯ )for I > 0.

CASE III: Roots differing by an integer 1,2,3,… , let CD > CE


J6 = I •Á ( g + 6 I + / I / + _ I _ + ⋯ )correspond to the larger root and

J/ = J6 (I) r4(I) + I •& ( g + 6 I + / I / + _ I _ + ⋯ ).


Note: In case II and case III the second solution or basis J/ is obtained by Reduction of order
method which is applied when one solution is known.
The general solution is = ”D ‡D (l) + ”E ‡E (l) . is some constant, may turn out to be zero.

SAQ3: Can we use Frobenius method to solve following differential equation


(I / − I) J @@ + (3I − 1) J @ + J = 0

7.5 Legendre differential equation


Any solution of Legendre Differential Equation:

180
MSCPH501

(1 − I / ) J @@ − 2I J @ + q(q + 1)J = 0 ; q 4 4 4 t (DE)

is called a Legendre Function (Polynomial). Rewrite above equation (12) in the form

2I q(q + 1)
J @@ − J @
+ J = 0. (D‘)
1 − I/ 1 − I/
Compare equation (13) with equation (2) we can write

• (I) = 0so Legendre equation is a homogeneous equation.


O(I) = − and ‚(I) = turns out to be singular at l = ±Dand analytic at
/= ( 6)
65= & 65= &
l = •.

Therefore solution lies in the interval −1 < I < 1, i.e. solution should be bounded (have finite
terms) and not divergent.
7.5.1 Solution of Legendre equation
We can apply power series solution to the differential equation with Ig = 0. The expansion of
power series solution given by equation(3) i.e.J = ∑r ©Åg © I and its derivatives, which is given
©

by equations (3.1 and 3.2), put this in equation (12) we get


r r r

(1 − I /)
ó 4(4 − 1) ©I
©5/
− 2I ó 4 ©I
©56
+ q(q + 1) ó ©I
©
=0 (D™)
©Å/ ©Å6 ©Åg

r r r r

ó 4(4 − 1) ©I
©5/
− ó 4(4 − 1) ©I
©
−2ó4 ©I
©
+ q(q + 1) ó ©I
©
= 0 (Dš)
©Å/ ©Å/ ©Å6 ©Åg

Writing each series and arranging each power of I :

2×1 / +3×2 _I +4×3 cI


/
+5×4 šI
_
+ ⋯ + ( + 2)( + 1) • /I

+⋯

+2×1 /I
/
+3×2 _I
_
+4×3 cI
c
+5×4 šI
š
+ ⋯ + ( − 1) • I • + ⋯

−2( 6 I + 2 /I
/
+3 _I
_
+4 cI
c
+ ⋯+ •I

+⋯)

+q(q + 1)( g + 6I + /I
/
+ _I
_
+ cI
c
+ ⋯+ •I

+ ⋯ )(DŸ)

Collect the coefficients of same power of I:

Coefficient of I g ∶ 2 / + q(q + 1) g =0

Coefficient of I ∶ 3×2 _ + j−2 + q(q + 1)l 6 =0

Coefficient of I / ∶ 4×3 c + j2 − 4 + q(q + 1)l / = 0.

181
MSCPH501

So we can write a general term as

Coefficient of I • ∶ ( + 2)( + 1) • / − ( − 1) • −2 • + q(q + 1) • = 0 or,

(¡ + E)(¡ + D)|¡ E − j¡(¡ − D) + E¡ − …(… + D)l|¡ = •. (Dœ)

j ( − 1) + 2 − q(q + 1)l = / − + 2 − q/ − q = ( − q)( + q) + ( − q)


The coefficient of second term of above equation can be written as

= ( − q)( + q + 1) = −(q − )(q + + 1).

So from equation (17) we get

−(… − ¡)(… + ¡ + D)
|¡ = |¡ (D•)
E
(¡ + E)(¡ + D)

called as recurrence relation or recurrence formula. With = 0,1,2,3,4, … we can get all the
coefficients in terms of g and 6 which is an arbitrary constants.

= 0,1,2,3,4, … in equation(18) we get


q(q + 1)
• Put the value of

=0∶ = −
/
2! g

(q − 1)(q + 2)
=1∶ = −
_
3! 6

(q − 2)q(q + 1)(q + 3)
=2∶ =
c
4! g

(q − 3)(q − 1)(q + 2)(q + 4)


=3∶ =
š
5! 6

(q − 4)(q − 2)q(q + 1)(q + 3)(q + 5)


=4∶ = −
a
6! g

(q − 5)(q − 3)(q − 1)(q + 2)(q + 4)(q + 6)


=5∶ = −

7! 6

` Q = 2,4,6, …) in terms of g
coefficients ( ` Q = 3,5,7, …) in terms of 6 .
and so on so that we can write even coefficients ( and odd

Put these coefficients in power series solution


r

J= ó ©I
©
= g + 6I + /I
/
+ _I
_
+ cI
c
+⋯
©Åg
We get
(q − 2)q(q + 1)(q + 3)
r
q(q + 1)
J=ó ©I
©
= − gI
/
+ gI
c
+⋯
g
2! 4!
©Åg

182
MSCPH501

(q − 1)(q + 2) (q − 3)(q − 1)(q + 2)(q + 4)


+ 6I − 6I +
_
6I
š
+⋯ (Dž)
3! 5!

‡ = |• ‡D (l) + |D £E (l) ; ∀ |I| < 1. (E•)


Or the solution of differential equation can be expressed as

q(q + 1) / (q − 2)q(q + 1)(q + 3) c


Called Legendre function with

J6 = 1 − I + I
2! 4!
(q − 4)(q − 2)q(q + 1)(q + 3)(q + 5) a
− I
6!
+⋯ (E•. |)

‡D
j…(… − E)(… − ™) … (… − E¥ + E)lj(… + D)(… + ‘)(… + š) … (… + E¥ − D)l E¥
r

= D + ó(−D)¥ l (E•. |)
(E¥)!
¥ÅD
(q − 1)(q + 2) _ (q − 3)(q − 1)(q + 2)(q + 4) š
J/ = I − I + I
3! 5!
(q − 5)(q − 3)(q − 1)(q + 2)(q + 4)(q + 6) –
− I
7!
+⋯ (E•. ˜)
j(… − D)(… − ‘) … (… − E¥ + D)lj(… + E)(… + ™)(… + Ÿ) … (… + E¥)l E¥
r

‡E = l + ó(−D)¥ l D
(E•. ˜)
(E¥ + D)!
¥ÅD

The series converges for |I| < 1 , therefore, the radius of convergence is unity. The two basis of
the solution J6 and J/ are linearly independent as J6 consists even power of I and J/ consists
odd power of I, so the ratio <Á is not constant.
<
&

= q in equation (18) we see that / = 0 which implies


c = 0, a = 0, ¦ = 0, 6g = 0, … 4N 4
• If put

If … is even: then from equation (20.a and 20.b) we see that J6 reduces to polynomial of degree
q andJ/ diverges. For example
Take q = 0, then Legendre functions
‡D (l) = Dwhich is a polynomial of order zero,
and J/ (I) = I + + + ⋯ = / ln ( ) which diverges.
=Â =§ 6 6 =
_ š 65=
So the bounded solution of differential equation for q = 0 is ‡ = ‡D (l) = D.
Take q = 2, then Legendre functions
‡D (l) = D − ‘ lE a polynomial of order two
and J/ (I) = 2I − − − / ln ( ) which diverges.
=Â _= ¨ 6 6 =
_ – 65=
So the bounded solution of differential equation for q = 2 is = ‡D (l) = D − ‘ lE .
If … is odd: then from equation (20.a and 20.b) we see that J/ reduces to polynomial of degree
q and J6 diverges. For example

183
MSCPH501

Take q = 1, then Legendre functions


J6 (I) = 1 − / I ln ( ) which diverges
6 6 =
65=
and‡E (l) = l which is a polynomial of order one.
So the bounded solution of differential equation for q = 1 is ‡ = ‡E (l) = l.
Take q = 3, then Legendre functions
J6 (I) diverges
and ‡E (l) = l − ‘ l‘ which is a polynomial of order three.
š

So the bounded solution of differential equation for q = 3 is ‡ = ‡E (l) = l − ‘ l‘ .


š

7.5.2 Legendre Polynomials „… (l)


Solution to Legendre’s equation are the polynomials J6 (I) and J/ (I) for various values of q

If J6 (I) or J/ (I) is the solution of differential equation then J6 (I) or J/ (I) ( is a constant)
gives an independent solution.

various q) are normalized to have value unity at I = 1 . . A (1) = 1 ( J(1) = 1). Such
is also a solution. We shall choose the normalization constant such that the polynomials (for

normalized polynomials are called as Legendre Polynomials.

q = 0 ∶ Ag (I) = 1 which is J6 (I) = 1 4N = 1


So the few normalized polynomials are

q = 1 ∶ A6 (I) = I which is J/ (I) = I 4N = 1


q = 2 ∶ A/ (I) = / (3I / − 1) which is J6 (I) = 1 − 3 I / 4N = −/
6 6

q = 3 ∶ A_ (I) = / (5I _ − 3I) which is J/ (I) = I − _ I _ 4N = −/


6 š _

q = 4 ∶ Ac (I) = ¦ (35 I c − 30 I / + 3)which is J6 (I) = 1 − 10I / + 35I c 4N =¦


6 _

1
q = 5 ∶ Aš (I) = (63 I š − 70 I + 15 I) (ED)
8
One canformulate compact or general representation for polynomials in equation (21). To do that

−(¡ + E)(¡ + D)
rewrite recurrence relation as

|¡ = | ; j¡ ≤ … − El. (EE)
(… − ¡)(… + ¡ + D) ¡ E

Then all non vanishing coefficients may be expressed in terms of coefficient |… of highest power
of Iof the polynomial. The coefficient is then arbitrary. Choose = 1 when q = 0 and in

(E…)! D × ‘ × š × … (E… − D)
general

|… = … = . (E‘)
E (…!) E …!
For this choice of |… all those polynomials will have the value 1atI = 1.
Now put = q − 2 (q ≥ 2) in equation (22)

184
MSCPH501

−(q)(q − 1)
=
5/
(2)(2q − 1)
put |… from equation (23) we get
−(q)(q − 1) (2q)! (2q − 2)!
5/ = = −
(2)(2q − 1) 2 (q!)/ 2 (q − 1)! (q − 2)!
Similarly if put = q − 4 (q ≥ 4) in equation (22) and with 5/ we can find
−(q − 2)(q − 3) (2q − 4)!
5c = 5/ = (−1)
/
(4)(2q − 3) 2 2! (q − 2)! (q − 4)!
So in general when … − E€ ≥ •we can write for ¡ = … − E€
(E… − E€)!
|…5E€ = (−D)€ … (E™)
E €! (… − €)! (… − E€)!
so the Legendre polynomials of degree … denoted by „… (l)can be written as
(E… − E€)!
ª

„… (l) = ó(−D)€ l…5E€ (Eš)


E… €! (… − €)! (… − E€)!
€Å•
whereª = E if…is evenandª = E if…is odd i.e. ªis an integer=
… …5D …
E
from equation (25) for integer q :
.

q = 0 ⟹ S = 0 ∶ Ag (I) = 1
q = 1 ⟹ S = 0 ∶ A6 (I) = I
q = 2 ⟹ S = 1: A/ (I) = / (3I / − 1)
6

q = 3 ⟹ S = 1: A_ (I) = / (5I _ − 3I)


6

1
q = 4 ⟹ S = 2: Ac (I) = (35 I c − 30 I / + 3)
8
1
q = 5 ⟹ S = 2: Aš (I) = (63 I − 70 I + 15 I)
š
8
we can cross check that equation (21) is reproduced. We can plot the polynomials given by
equation (25), first few have been plotted in the figure below

185
MSCPH501

We will try to find yet another solution of Legendre differential equation in terms qŒ¬ order
7.5.3 Rodrigue’s Formula

derivative of some function and from this we will obtain Legendre polynomial.

J = (I / − 1) . (EŸ)
Consider a function

NJ
Take derivative of equation (26)
= 2 q I (I / − 1) 56 . (Eœ)
NI
Multiply both side of equation (27) by (I / − 1)
NJ /
(I − 1) = 2 q I (I / − 1)
NI
= J 2 q I. (E•)
Differentiate both side of equation (28) (q + 1) times by Leibniz’s rule taking ¢ = •= , =
•<

(I / − 1) on L.H.S. and ¢ = J, = 2qI on R.H.S..

186
MSCPH501

N 6
NJ / N 6
æ (I − 1)ç = jJ 2 q I l
NI 6 NI NI 6

N /
J N 6
J N J
(I / − 1) + 6 (2I) + 6
2
NI / 6
NI 6 /
NI
N 6
J N J
= (2qI) + 6 (2q)
NI 6 6
NI

N /
J N 6
J N J
(I / − 1) + (2I) − q(q + 1) =0
NI / NI 6 NI

N J
N 6J N J
or,
/
(1 − I / ) − (2I)+ q(q + 1) =0
NI NI 6
/ NI
Let - = ~l… then we can write above equation as
~… ‡

N/ • N•
− (2I) (1 − I / )
+ q(q + 1) •
NI / NI
= 0. (Ež)
Equation (29) is Legendre equation and • is the solution this equation with
~… ‡ ~… (lE − D)…
-= = .
~l… ~l…
Now let the Legendre polynomial be

~… ‡ ~… (lE − D)…
„… (l) = ®- = ® = ® . (‘•)
~l… ~l…

To find p , put I = 1 in equation (30) so that from the definition of Legendre polynomial we

~… (lE − D)…
have

„… (D) = D = ® ¼ ½ .
~l… lÅD

J = (I / − 1) = (I − 1) (I + 1) .
Now as we had considered

Differentiate both sides q times and use Leibniz rule (on R.H.S.) to get
N J N (I / − 1)
=
NI NI
N (I + 1) N 56 (I + 1)
= (I − 1) + q •q(I − 1) 56 –
NI NI 56
N 5/ (I + 1)
+ q(q + 1) •q(q − 1)(I − 1) 5/ – + ⋯
NI 5/
N (I − 1)
+ (I + 1)
NI

187
MSCPH501

N J N (I / − 1) N (I − 1)
¦ § =¼ ½ = 0 + 0 + 0 + ⋯ + 2 (q!) ¯ = q!°
NI =Å6 NI =Å6
NI
±
hence, A (1) = 1 = p ¦ § = p 2 (q!) so we get ® = E… (…!)therefore the
•± Y= & 56Z D
•= ±
=Å6
Legendre polynomials are also obtained from

D ~… (lE − D)…
„… (l) = (‘D)
E… (…!) ~l…
well known as Rodrigue’s Formula.

7.6 Properties of Legendre Polynomial

Since „… (l)is a sequence of polynomials, it may appear as coefficient of some particular series.
7.6.1 Generating function

We observe that on the expansion of a function

D
²(³, l) = (‘E)
√D − El³ + ³E
as power series „… (l)is generated as a coefficient of K © . Hence, equation (32) is expressed as
generating function for Legendre Polynomials.
Proof: Let Ú = 2IK − K / then (K, I) =
6
√65u
, Now binomial expansion of this function is
1 6
= (1 − Ú)5/
√1 − Ú
1 3 1 3 5
1 ž− 2Ÿ ž− 2Ÿ ž− 2Ÿ ž− 2Ÿ (− 2)
= 1 + ¦− § (−Ú) + (−Ú)/ + (−Ú)_ + ⋯
2 2! 3!
1 1×3 / 1×3×5 _ 1 × 3 × 5 × … × (24 − 1) ©
= 1+ Ú+ Ú + Ú + ⋯+ Ú +⋯
2 2×4 2×4×6 2 × 4 × 6 × … × 24

1 1×3 1×3×5
= 1 + (2IK − K / ) + (2IK − K / )/ + (2IK − K / )_ + ⋯
2 2×4 2×4×6
K/ 3 5
= 1 + IK − + (4I / K / + K c − 4IK _ ) + (8I _ K _ + K a − 8I / K c − 4IK š ) + ⋯
2 8 16
1 1 5 5
= 1 + IK + (3I / − 1)K / + ¦−12IK _ + × 8I _ K _ − 4IK š § + ⋯
2 8 2 2
1 1
= 1 + IK + (3I / − 1)K / + (5I _ − 3I)K _ + ⋯
2 2
= Ag (I) + A6 (I)K + A/ (I)K / + A_ (I)K _ + ⋯
r
D
²(l, ³) = (D − El³ + ³ E ) 5E
= ó „} (l) ³} . (‘‘)
}Å•

188
MSCPH501

If „… (l)and „} (l)are two Legendre polynomials then the orthogonality property is defined as
7.6.2 Orthogonality

D • ‹k } ≠ …
â „… (l)„} (l) ~l = ~ E =
‹k } = …
5D E} + D
Proof:

(1 − I / ) J @@ − 2I J @ + 4(4 + 1)J = 0
The Legendre equation is

N NJ
or
(1 − I / ) + 4(4 + 1)J = 0.
NI NI
Since „… (l)and „} (l)are solutions of the Legendre equation, we can write
N NA
(1 − I / ) + q(q + 1)A = 0 (34)
NI NI

N NA©
and
(1 − I / ) + 4(4 + 1)A© = 0 (35)
NI NI
multiply equation (34) by „} (l)and equation (35) by „… (l)and subtract
N NA© N NA
A (1 − I / ) − A© (1 − I / ) + A© A j4(4 + 1) − q(q + 1)l = 0.
NI NI NI NI
Integrating above equation w.r.t. I from −1 to 1

6 6 6
N NA© N NA
âA (1 − I / ) NI − â A© (1 − I / ) NI + (4 − q)(4 + q + 1) â A© A NI
NI NI NI NI
56 56 56
= 0.
The first and second term will cancel each other so we get
6

(4 − q)(4 + q + 1) â A© A NI = 0. (‘Ÿ)
56
Case I: if … ≠ }then term outside the integration in equation (36) is not zero hence
D

â „} „… ~l = • (Q … ≠ }). (‘œ)
5D
Case II: if … = }then equation (36) is zero as term outside the integration is zero, however the
integration may not be zero for this case. To find the value of integration ´5D „E} ~l , start with
D

the generating function


6
(1 − 2 I K + K / )5/ = ó A© K © .
Squaring both side we get

189
MSCPH501

(1 − 2 I K + K / )56 = ´ó A© K © µ ´ó A K µ = ó A©/ K /© + 2 ó ó A© A K © (‘•)


© © © ¶©
The expression on R.H.S. has been separated into two terms, first for q = 4 and second for
q ≠ 4. The factor 2 with the second is due to repeated terms for each q and 4.

q = 1,2 and 4 = 1,2 so R.H.S. is


For example: Let’s look how it would appear, we will evaluate R.H.S. of equation (38) for

/ /

ó A© K ó A K
©
= (A6 K + A/ K / ) (A6 K + A/ K / )
©Å6 Å6
= jA6/6 K / + A// K c l + jA6 K A/ K / + A/ K / A6 K]
= jA6/6 K / + A// K c l + 2 jA6 A/ K _ l
/ / /

=ó A©/ K /© + 2ó ó A© A K ©
©Å6 ©Å6 Å6, ¶©

Integrating equation (38) both side w.r.t. I from −1 to 1 we get

6
1
â NI
(1 − 2 I K + K / )
56
6

= â ´ó A© K © µ ´ó A K µ NI
56 ©
6 6

= óK /©
â A©/ NI + 2ó ó K ©
â A© A NI.
© 56 © ¶© 56
Using orthonality property second term on RHS is zero, so we get
6
1
− jr4(1 − 2 I K + K / )l656 = ó K /© â A©/ NI
2K
© 56
6
1 1
ó K /© â A©/ NI = − jln(1 − K)/ − ln(1 + K)/ l = jr4(1 + K) − r4(1 − K)l
2K K
© 56
1 1+K
= jln ¦ §l
K 1−K

190
MSCPH501

6
2 K_ Kš K– K /© 6
ó K /© â A©/ NI = ÊK + + + +⋯+ +⋯Ë
K 3 5 7 24 + 1
© 56

K/ Kc Ka K /© 1
= 2 Ê 1 + + + + ⋯+ + ⋯ Ë = 2 ´ó K /© ¦ §µ.
3 5 7 24 + 1 24 + 1
©

On comparing the coefficient of ∑© K /© we get


6
2
â A©/ NI =
24 + 1
56
Or we can write
D
E
â „} „… ~l = Q … = }. (‘ž)
E} + D
5D

• ‹k } ≠ …
Combining equation (37 ) and equation (39) we get orthoganility property
D

â „… (l)„ (l) ~l = ~ E =.
‹k } = …
}
5D E} + D

}Å• „} (l) ³ both side w.r.t.³, we


7.6.3 Recurrence relations
Differentiating generating function (D − El³ + ³E )5E = ∑r
D
}

get
r
1 _
− (1 − 2IK + K / )5/ (−2I + 2K) = ó 4 A© (I) K ©56
2
©Å6
r
6
(I − K)(1 − 2IK + K / ) 5/ = (1 − 2IK + K /)
ó 4 A© (I) K ©56 .
©Å6
Use generating function to replace 2nd factor on LHS
r r

(I − K) ó A© (I) K ©
= (1 − 2IK + K /)
ó 4 A© (I) K ©56
©Åg ©Å6
r r

I ó A© (I)K © − ó A© (I)K © 6

©Åg ©Åg
r r r

= ó 4A© (I)K ©56 − 2I ó 4A© (I)K © + ó 4A© (I)K © 6


(™•)
©Å6 ©Å6 ©Å6

Expanding equation (40) both sides

191
MSCPH501

jI YAg (I) + A6 (I)K + ⋯ + A`56 (I)K `56 + A` (I)K ` + ⋯ Zl − j(Ag (I)K + ⋯ + A`5/ (I)K `56
+ A`56 (I)K ` + ⋯ )l
= jA6 (I) + 2A/ (I)K + ⋯ + A` (I) K `56 + ( + 1)A` 6 (I) K ` + ⋯ l
− j2I (A6 (I) K + 2A/ (I) K / + ⋯ + ( − 1)A`56 (I) K `56 + ( )A` (I) K ` + ⋯ )l
+ jA6 (I) K / + 2A/ (I) K _ + ⋯ + ( − 2)A`5/ (I) K `56 + ( − 1)A`56 (I) K ` + ⋯ l
and equating coefficient of ³‹5D
IA`56 (I) − A`5/ (I) = A` (I) − 2I( − 1)A`56 (I) + ( − 2)A`5/ (I)
and replacing by 4
I A©56 (I) − A©5/ (I) = 4A© (I) − 2I (4 − 1)A©56 (I) + (4 − 2)A©5/ (I)
We get one of the recurrence relations from coefficient of ³}5D as
}„} (l) = E(} − D) l „}5D (l) − (} − D)„}5E Q 4 = 2,3,4, … (™D)
if we equate coefficient of ³‹ in the expansion of equation (40) and replacing by 4we get other

(} + D)„} D (l) = E(} + D) l „} (l) − }„}5D Q 4 = 1,2,3,4, … (™E)


recurrence relation

Again first few Legendre polynomialshave been shown in the figurebelow but this time we used
recurence relation (41) to draw the figure. The figure has been generated using Sci-Lab package.

192
MSCPH501

SAQ4: Show that }„} (l) = l„@} (l) − „@}5D (l) where „@} (l) =
~„} (l)
~l

The Bessel’s differential equation of order † (a real and non-negative number) is


7.7 Bessel’s differential equation

lE ‡@@ + l ‡@ + (lE − ·E )‡ = •

D lE − ·E
can be rewritten in the form of equation (2) as

‡@@ + ‡@ + ‡ = •.
l lE
Compare above equation with equation (2) we can write

• (I) = 0so Bessel’s equation is a homogeneous equation.


O(I) = = and ‚(I) = turns out to be singular at l = •.
6 lE 5·E
lE

However in the lim I O(I) and lim I / ‚(I) are finite at l = •.
=→g =→g

Therefore at I = 0 the equation has regular singularity, hencesolution can be obtained using
Frobenious method. The solution will lie in the interval ( 0, ∞).

7.7.1 Solution of Bessel’s equation

Solution: Using Frobenious method we can find the solution of the Bassel’s function, let the
solution be
r

J= ó I •
.
Åg

The first and second derivative of J is


r

J′ = ó (q + ) I •56

Åg

J = ó (q + )(q + − 1)
@@
I •5/
.
Åg

Replace J, J @ 4N J′′ in Bessel’s differential equation

193
MSCPH501

r r r

ó (q + )(q + − 1) I •
+ ó (q + ) I •
+ (I / −† /)
ó I •
=0
Åg Åg Åg

r r r

ó (q + )(q + − 1) I •5/
+ ó (q + ) I •56
+ó I • /

Åg Åg Åg
r

− †/ ó I •
= 0. (™‘)
Åg

• To find indicial equation, equate the coefficient of lowest power of I i.e. I • for q = 0, we get

( − 1) g + g − †/ g
= 0. (™™)

The lowest power of I gives the indicial equation so we get

[C (C − D) + C − ·E l|• =
• (™š)

with choice g ≠0

[C (C − D) + C − ·E l = CE − ·E = (C + ·)(C − ·) = •.

We get the solution for as

CD = · |}~ CE = −·.

• To find |D , equate the coefficient of next lowest power of I i.e. I • 6 for q = 1, to get

( + 1) 6 + ( + 1) 6 − †/ 6 =0 (™Ÿ)

j( + 1) + ( + 1) − † / l 6 = 0. (™œ)

The term in square bracket is not zero for the solution of Cobtained from indicial equation
therefore we get

|D = •.

• To find recursion relation, equate the coefficients of I • •


to zero

( + )( + − 1) • +( + ) • + •5/ − †/ • = 0. (™•)

Rearrange the terms to get recursion relation as

D
|¡ = − | Q = 2,3,4, … (™ž)
((¡ + C)E − ·E ) ¡5E

194
MSCPH501

Now we will find first Bessel function for first solution of indicial equationi.e. C = CD = ·, put
this in equation (49) to get

D D
|¡ = − | = − | Q = 2,3,4, … (š•)
((¡ + ·)E − ·E ) ¡5E
(¡E + E ¡ ·) ¡5E

Since 6 = 0 we get _ = 0, š = 0, – = 0, … …. , so we get all the odd coefficients zero as all


the odd coefficients can be written in terms of 6 from equation (50). So only even coefficients
are non-zero i.e. for = 2q Ú ℎ q = 1,2,2,3 …. . From eq. (50) for ¡ = E… we get

D D
|E… = − |E…5E = − E | (šD)
™…E +™·… E …(· + …) E…5E

= − /& (¸Ý 6)
˜
m =1 : /

= − /& / (¸& = /î (/!)(¸ Ý6)(¸


˜ ˜
m =2 : c /) /)

= − =− =−
˜î ˜Ý ˜Ý
m =3 : a /& _ (¸ _) (/& /î ) (_×/!)(¸ 6)(¸ /)(¸ _) /¹ (_!)(¸ 6)(¸ /)(¸ _)

in general we can write

(−D)… |•
|E… = . Q q = 1,2,3 … (šE)
EE… (…!)(· + D)(· + E)(· + ‘) … … … (· + …)

Now |• is still arbitrary so for simplicity take |• = (E· ·!)put in eq.(52) to get
D

(−D)…
|E… = · (…!)(· + …)!
Q q = 1,2,3, … (š‘)
EE…

The solution forC = CD = ·is denoted by º· (l), we have assumed solution to be J =


∑rÅg I • from Frobenious method. Replace the coefficient by / (as the odd
coefficients are zero) to get the Bessel function

º· (l) = l· ó |E… lE…


…Å•

(−D)… lE… ·
º· (l) = ó . (š™)
EE… · (…!)(· + …)!
…Å•

This is called BESSEL FUNCTION of First Kind of order ·.

195
MSCPH501

Figure 1: Graphical representation of several Bessel’s function. (Source:


http://mathworld.wolfram.com/BesselFunctionoftheFirstKind.html)

Second Bessel function for other solution of indicial equation i.e. C = CE = −·, which can be
obtained by replacing ·by – ·in Eq. (54).

(−D)… lE…5·
º5· (l) = ó E…5· (Q · }»Œ |} ‹}Œ•¼•C) (šš)
E (…!)(−· + …)!
…Å•

This is called BESSEL FUNCTION of second Kind of order −·.

So Q · }»Œ |} ‹}Œ•¼•C the general solution of Bessel differential equation is

‡(l) = ®D º· (l) + ®E º5· (l). (šŸ)

If ·is an integer then Eq. (56) is not a general solution as º· and º5· are not linearly independent
which can be shown as follows

Linear dependence of º· and º5· when · is an integer:


Put = −† + qthen q = + † , put this in Eq.(55) for º5· (l) (with · to be integer,

summation on …will start from ·and not from zero as for any value of… = • Œ» ·the
term(… − ·)! = ½(… − · + D) becomes infinite, hence summation over { will start
from 0).

(−D){ · lE{ · (−D){ lE{ ·


º5· (l) = ó = (−D) ·
ó
EE{ · (…!)({ + ·)! EE{ · (…!)({ + ·)!
{Å• {Å•

º5· (l) = (−D)· º· (l) therefore, If ·is an integer º· and º5· are linearly dependent.

When · in an integer i.e. the roots of indicial equation differ by an integer then the

196
MSCPH501

• First solution is J6 (I) = i¸ (I) and


• second linearly independent solution is given as CASE III of solution of indicial equation
when solution is obtained using Frobenous method
J/ (I) = ¸ ln(I) i¸ (I) + I 5¸ ó #I
# ¸

#Åg

The second solution or basis J/ is obtained by Reduction of order method which is applied when
one solution is known.

Ycos(†Ù) i5¸ (I) − i5¸ (I)Z


For many purposes, it is convenient to take the linear combination

J/ = b¸ (I) =
sin(†Ù)

as the second independent solution instead of i5¸ (I). This is known as the Bessel function of
second kind of order † . Hence the general solution is given as

‡(l) = ®D º· (l) + ®E ¾· (l) (šœ)

Bessel function of order zero (· = •), º• (l):


/
= 0, hence double root
6 = / = 0.
• From indicial equation Eq. (45) we find the solution to be

• From recursion relation Eq. (49)

D
|¡ = − | Q = 2,3,4, … (š•)
(¡ + C)E ¡5E

Hence from Eq. (52) one can write even coefficients as

(−D)… |•
|E… = Q q≥1
(C + E)E (C + ™)E (C + Ÿ)E … … (C + E…)E

(−D)… |•
|E… = E… Q = 0 4N q = 1,2,3 … (šž)
E (…!)E

Therefore, the solution is

(−D)… lE…
‡C = lC |• D + ó
(C + E)E (C + ™)E (C + Ÿ)E … … (C + E…)E
Å6

and for = 0.,

197
MSCPH501

(−D)… lE…
‡CÅ• = |• D + ó
(E)E (™)E (Ÿ)E … … (E…)E
Å6

(−D)… lE…
»C, º• (l) = |• D + ó E… . (Ÿ•)
E (…!)E
Å6

This is Bessel Function of the first kind of order zero.

7.7.2Recurrence relations

Following relations are the recurrence formulae and their proof for Bessel's functions:

1. Yl· º· (l)Z = l· º·5D (l)


@

NYI ¸ i¸ (I)Z
Proof:
N ¸ (−1) I / ¸ N (−D)… lE(… ·)
= ~I ó / ¸ ¿= ~ ó E… · ¿
NI NI 2 (q!)(† + q)! NI E (…!)(· + …)!
Åg …Å•

E(… + ·)(−D)… lE(… ·)5D (… + ·)(−D)… lE… ·5D


YI i¸ (I)Z = ó = I ó E… ·5D
¸ @ ¸
EE… · (…!)(· + …)! E (…!)(· + …)(… + · − D)!
…Å• …Å•
(−D)… E… ·5D
l
= I¸ ó = I ¸ i¸56 (I).
EE… ·5D (…!) (… + · − D)!
…Å•

2. Yl5· º· (l)Z = −l5· º· D (l)


@

Proof:

NYI 5¸ i¸ (I)Z N 5¸ (−1) I / ¸ N (−D)… lE(…)


= ~I ó / ¸ ¿= ~ ó E… · ¿
NI NI 2 (q!)(† + q)! NI E (…!)(· + …)!
Åg …Å•

E… (−D)… lE(…)5D (−D)… lE(…) ·5D


YI i¸ (I)Z = ó = l ó E… ·5D
5¸ @ 5·
EE… · (…!)(· + …)! E (… − D)! (· + …)!
…Å• …Å•

for q − 1 = . .q = +1

(−D){ D lE{ · D (−D){ lE{ · D


YI 5¸ i¸ (I)Z = l5· ó = −l5· ó E{
@
EE{ · D ({)! (· + { + D)! E · D ({)! ({ + · + D)!
{Å• {Å•
= −I 5¸
i¸ 6 (I).

198
MSCPH501

3. º′· (l) = l º· (l) − º· D (l)orl º′· (l) = · º· (l) − l º· D (l)


·

Proof:

NYi¸ (I)Z N (−1) I / ¸ (2q + †)(−1) I / ¸56


= i′¸ (I) = ~ó / ¸ ¿= ó .
NI NI 2 (q!)(† + q)! 2/ ¸ (q!)(† + q)!
Åg Åg

Multiplying both side by I

(2q + †)(−1) I / ¸
I i′¸ (I) = ó
2/ ¸ (q!)(† + q)!
Åg
2q (−1) I / ¸ (−1) I / ¸
= ó / ¸ +† ó / ¸
2 (q!)(† + q)! 2 (q!)(† + q)!
Åg Åg
(−1) I / ¸
= ó ¸56 (q
+ † i¸ (I)
2/ − 1)! († + q)!
Åg

for q − 1 = . .q = +1

(−1)# 6 I /# ¸ /
I i′¸ (I) = ó ¸ 6 ( )! († + 1 + )!
+ † i¸ (I)
2/#
#Åg
(−1)# I /# ¸ 6
= −I ó ¸ 6 ( )! († + 1 + )!
+ † i¸ (I) = −Ii¸ + † i¸ (I).
2/# 6
#Åg

4. º′· (l) = E Yº·5D (l) − º· D (l)Z.


D

l º′· (l) = −· º· (l) + l º·5D (l).


SAQ 5: Show that

5. º· (l) = E· Yº·5D (l) + º· D (l)Z.


l

7.7.3 Generating function

The Bessel polynomials i© (I) can be expressed ascoefficients of Œ} in the series expansion of a
function ÀÁ ¦E l žŒ − Œ Ÿ§ called “Generating function’.
D D

199
MSCPH501

r
D D
ÀÁ ¼ l ¦Œ − §½ = ó º} (l) Œ} (ŸD)
E Œ
}Å5r

Proof:

1 1 =Œ = I 1 I / I 1 I /
exp ¼ I ¦ − §½ = /
5
/Œ = Ê1 + + ¦ § + ⋯ Ë × æ1 − + ž Ÿ − ⋯ ç
2 2 2! 2 2 2! 2
©
the coefficient of in this product is

1 I © 1 I © / 1 I © c
ž Ÿ − ž Ÿ + ž Ÿ − ⋯ = i© (I)
4! 2 (4 + 1)! 2 2! (4 + 1)! 2

as all the integral powers of , both positive and negative occurs, we have

1 1
exp ¼ I ¦ − §½ = ig (I) + i6 (I) + /
i/ (I) + ⋯ + 56
i56 (I) + 5/
i5/ (I) + ⋯
2
r

= ó i© (I) ©
.
©Å5r

Thus, the coefficients of different powers of in the expansion of exp ¦/ I ž − Œ Ÿ§ give Bessel's
6 6

functions of various orders, hence it is said to be the generating function of Bessel's functions.

7.7.4 Orthogonality

If+ andà are the roots of the equation i© (-) = 0 then condition of orthogonality of Bessel’s
function over the interval (0,1) with weight function isI is

• ‹k Ä ≠ Å
D
â l º} (Äl)º} (Ål)~l = Æ D = (ŸE)
• ºE} D (l) ‹k Ä = Å
E
Or both the cases can be written in terms of delta function
D
D E
â l jº} (Ål)lE ~l = º (l)ÇÄÅ
• E } D

Proof: The second order Bessel’s differential equation is

lE ‡@@ + l ‡@ + (lE − }E )‡ = •

200
MSCPH501

Let us change the independent variableIto +I, the resulting equation is


I / J @@ + I J @ + (+/ I / − 4/ )J = 0

With the general solution from equation (57) be p6 i© (+I) + p/ b© (+I).

Let J6 (I) = i© (+I) and J/ (I) = i© (ÃI) are the solutions of the equation then we have

I / J6@@ + I J6@ + (+/ I / − 4/ )J6 = 0 (63)

I / J/@@ + I J/@ + (Ã / I / − 4/ )J/ = 0 (64)


and

multiply equation (63) by J/ and eq.(64) by J6 and subtract to get


N NJ6 NJ/ NJ6 NJ/ N NJ6 NJ/
(à / − +/ )I J6 J/ = I æJ/ − J6 ç + æJ/ − J6 ç = ¦I æJ/ − J6 ç§
NI NI NI NI NI NI NI NI
integrating on I with limit 0 1 we get
6 6
N NJ6 NJ/ NJ6 NJ/ 6
(à / − +/ ) â I J6 J/ NI = â ¦I æJ/ − J6 ç§ NI = ¦I æJ/ − J6 ç§
g g NI NI NI NI NI g
Since J6 (I) = i© (+I)and J/ (I) = i© (ÃI) , replace these in above equation
N i© (+I) N i© (ÃI)
6
¦I æi© (ÃI) − i© (+I) ç§
6 NI NI
â Ii© (+I)i© (ÃI) NI = g
g (Ã − + )
/ /

6
Y+ i© (ÃI)i© (+I) − Ã i© (ÃI)i© (+I)Z
@ @
â Ii© (+I)i© (ÃI) NI = Q +≠ Ã (65)
g à / − +/
Therefore, in order to ensure orthogonality we must have + 4N Ã be zeros of i© (I), i.e.i© (+) =
i© (Ã) = 0 then for + ≠ Ã in eq. (65) we have
6
â Ii© (+I)i© (ÃI) NI = 0.
g
For Ä = Å if we take the limit then we get RHS of eq. (65) of the form g. So we apply L’Hospital
g

rule (differentiate w.r.t. Ã keeping + constatnt) and find


6
/ (+I)
Y+ i©@ (+)i©@ (Ã) − i© (+)i©@ (Ã) − Ãi© (+)i©@@ (Ã)Z
â I i© NI = lim
g È→É 2Ã
@ (+)i@ (+)
Y+ i© − i© (+)i© (+) − +i© (+)i©@@ (+)Z
@
=
©
2+
Since i© (+) = 0 we have
6
i©@ (+)i©@ (+) 1 1
â I i©/ (+I) NI = + = i©@ (+)i©@ (+) = i©/ 6 (+).
g 2+ 2 2
Since i© 6 (+) ≠ 0, thus we have the orthogonality condition.

201
MSCPH501

Steps to show º@} (Ä) = −º} D (Ä)

I i©@ (I) = 4 i© (I) − I i© 6 (I)


With the recurrence relation

change the independent variable I +I

+I i©@ (+I) = 4 i© (+I) − +I i© 6 (+I)


For I = 1we have

+ i©@ (+) = 4 i© (+) − + i© 6 (+) = −+ i© 6 (+) 4 i© (+) = 0.

7.8 Summary
• Legendre differential equation

(1 − I / ) J @@ − 2I J @ + q(q + 1)J = 0 ; q 4 4 4 t

• Legendre polynomials of degree … denoted by „… (l)can be written as


(E… − E€)!
ª

„… (l) = ó(−D)€ l…5E€


E… €! (… − €)! (… − E€)!
€Å•
whereª = E if…is evenand ª = if…is odd i.e. ªis an integer=
… …5D …
E E
.

• Properties of Legendre polynomials

D ~… (lE − D)…
o Rodrigue’s Formula

„… (l) = .
E… (…!) ~l…

r
o Generating function
D
²(l, ³) = (D − El³ + ³ E ) 5E
= ó „} (l) ³} .
}Å•

D • ‹k } ≠ …
o Orthogonality

â „… (l)„} (l) ~l = ~ E =.
‹k }=…
5D E} + D

o Recurrence relations

}„} (l) = E(} − D) l „}5D (l) − (} − D)„}5E Q 4 = 2,3,4, …


(} + D)„} D (l) = E(} + D) l „} (l) − }„}5D Q 4 = 1,2,3,4, …
}„} (l) = l„@} (l) − „@}5D (l)

202
MSCPH501

(}+D) „} D (l) = (E} + D)l„} (l) − } „}5D (l).

lE ‡@@ + l ‡@ + (lE − ·E )‡ = •.
• Bessel’s differential equation

Bessel function of First Kind of order · :


(−D)… lE… ·

º· (l) = ó E… · .
E (…!)(· + …)!
…Å•

−·º5· (l) = ∑…Å•
Bessel function of Second Kind of order
(Q · }»Œ |} ‹}Œ•¼•C)
(5D)… lE…Ê·
EE…Ê· (…!)(5· …)!

(−D)… lE…
• Bessel Function of the first kind of order zero:

º• (l) = |• D + ó .
EE… (…!)E
Å6
• Properties of Bessel function

Yl· º· (l)Z = l· º·5D (l)


@
o Recurrence Formula

Yl5· º· (l)Z = −l5· º· D (l)


@

º′· (l) = l º· (l) − º· D (l)


·

l º@ · (l) = −· º· (l) + l º·5D (l)


º@ · (l) = E Yº·5D (l) − º· D (l)Z
D

º· (l) = E· Yº·5D (l) + º· D (l)Z.


l

r
D D
o Generating function

ÀÁ ¼ l ¦Œ − §½ = ó º} (l) Œ} .
E Œ
}Å5r

• ‹k Ä ≠ Å
o Orthogonality:
D
â l º} (Äl)º} (Ål)~l = Æ D =
• ºE} D (l) ‹k Ä = Å
E

7.9 Answer to SAQs:

SAQ1: From equation we have O(I) = /=(=5/)& = /(=5/)&and (I) = /=(=5/) , so there are two
_= _ 6

singular points, I = 0, I = 2 at which O(I), ‚(I) are not finite.


To check for regular singular point, evaluate weak singularity condition

203
MSCPH501

3 3
lim(I − 2) O(I) = lim(I − 2) = lim → ∞ . .4 Q4 r¢
=→/ =→/ 2(I − 2) / =→/ 2(I − 2)

So I = 2 is not a regular singular point. Now check for I = 0,

3 3I
lim(I − 0) O(I) = lim I = lim → 0 . . Q4 r¢
=→g =→g 2(I − 2) / =→g 2(I − 2)/

1 I
lim(I − 0)/ ‚(I) = lim I / = lim → 0. . . Q4 r¢
=→g =→g 2I(I − 2) =→g 2(I − 2)

Hence I = 0 is a regular singular point or the differential equation has regular singularity
at I = 0.

SAQ2: Step1: Assume the power series solution


r

J= ó ©I
©
= g + 6I + /I
/
+ _I
_
+ cI
c
+⋯
©Åg

Then find derivatives


r

J @@ = ó 4 (4 − 1) ©I
©5/
= 2 / +3×2 _ I+4×3 cI
/
+5×4 šI
_
+⋯
©Å/

Step 2: Replace J 4N J′′ in the given differential equation:


r r

∴ J @@ + J = ó 4 (4 − 1) ©I
©5/
+ ó ©I
©
= 0.
©Å/ ©Åg

Or,

2 / +3×2 _ I+4×3 cI
/
+5×4 šI
_
+⋯+ g + 6I + /I
/
+ _I
_
+ ⋯ = 0.

Step 3: Collect the like powers of I

(2 / + g ) + (3 × 2 _ + 6 )I + (4 × 3 c + / ) I / + (5 × 4 š + _ )I _ + ⋯ = 0
Now, equate the sum of the coefficient of each occurring power of I to zero and find
g, 6, /, _, c, š, …

Coefficient of I g ∶ 2 + =0 ⇒ = − = − /!Ý
˜Ý ˜
/ g / /

Coefficient of I ∶ 3×2 + =0 ⇒ = − _×/ = − _!Á


Á ˜ ˜
_ 6 _

Coefficient of I / ∶ 4×3 + =0 ⇒ = − c×_ = =


& ˜ ˜Ý ˜Ý
c / c c×_×/ c!

204
MSCPH501

Coefficient of I _ ∶ 5×4 + =0 ⇒ = − = − =
˜Â ˜Á ˜Á
š _ š š×c š×c×_×/ š!

Coefficient of I c ∶ 6×5 + =0 ⇒ = − = − = −
˜î ˜Ý ˜Ý
a c a aך aך×c×_×/ a!

=− , = , =
˜Á ˜Ý ˜Á
Similarly we can find – –! ¦ ¦! — —!
and so on. Thus in power series of
J when 4 is odd the coefficients are expressed in term of 6 and when 4 is even the
coefficients are expressed in term of g with alternate signs.

Step 4: Put the value of coefficients in power series of J and simplify the solution by
collecting terms of g and 6 .

J= g + 6I + /I/ + _I
_

+⋯

J= + 6I − I/ − I_ + Ic + Iš − Ia − I– + I¦ + I— − ⋯
g 6 g 6 g 6 g 6
g
2! 3! 4! 5! 6! 7! 8! 9!

I/ Ic Ia I¦ I_ Iš I– I—
J = g ¼1 − + − + −⋯½+ 6¼I− + − + −⋯½
2! 4! 6! 8! 3! 5! 7! 9!

‡ = |• ÌÍ.(l) + |D .ˆ -(l) = 6 J6 (I) + / J/ (I).

This is a well-known solution, and can also be obtained directly from standard methods

(J6 (I) 4N J/ (I) are two basis of the general solution J).
to solve the second order homogeneous ordinary differential equation.

The example is just to convince you that we can apply power series method to find
solution of any differential equation although we apply the method to solve generally
special differential equations.

SAQ3: Comparing differential equation with general form, we have


3I − 1 3I − 1 1
O(I) = = 4N ‚(I) =
I − I I(I − 1)
/ I(I − 1)
so there are two singular points I = 1 4N I = 0.
Check for regular singularity:
3I − 1 3I − 1
lim(I − 1)O(I) = lim(I − 1) = lim →2
=→6 =→6 I(I − 1) =→6 I
1 I−1
lim(I − 1)/ ‚(I) = lim(I − 1)/ = lim →0
=→6 =→6 I(I − 1) =→6 I

205
MSCPH501

3I − 1 3I − 1
lim(I − 0)O(I) = lim I = lim →1
=→g =→g I(I − 1) =→g (I − 1)
1 I
lim(I − 0)/ ‚(I) = limI / = lim →0
=→g =→g I(I − 1) =→g (I − 1)

Hence both I = 0,1 are regular singular points So solution of the differential equation
can be obtained using Frobeneous method.

SAQ4: Step 1: Differentiating generating function given by equation (33)


r
D
(D − El³ + ³ E ) 5E
= ó „} (l) ³}
}Å•
w.r.t. K we get
_
(1 − 2IK + K / )5/ (I − K) = ó 4 K ©56 A© (I).
Step2: Now differentiating generating function w.r.t. I we get
_
K (1 − 2IK + K / )5/ = ó K © A©@ (I).

(I − K)
Step 3: put result of step 2 in step 1 to get
ó K © A©@ (I) = ó 4 K ©56 A© (I)
K
Or
(I − K) ó K © A©@ (I) = ó 4 K © A© (I).
Step4: now compare the coefficient of K © and rearrange terms to get
}„} (l) = l„@} (l) − „@}5D (l).

SAQ 5: Proof:

NYi¸ (I)Z N (−1) I / ¸


= i′¸ (I) = ~ó / ¸ ¿
NI NI 2 (q!)(† + q)!
Åg
(2q + †)(−1) I / ¸56
= ó .
2/ ¸ (q!)(† + q)!
Åg

Multiplying both side by I and write 2q + † = 2q + 2† − †

206
MSCPH501

(2q + 2† − †)(−1) I / ¸
I i′¸ (I) = ó
2/ ¸ (q!)(† + q)!
Åg
2(q + †) (−1) I / ¸ (−1) I / ¸
= ó − † ó
2/ ¸ (q!)(† + q)! 2/ ¸ (q!)(† + q)!
Åg Åg
(−1) I / ¸56 6
= ó ¸56 (q)! († − 1 + q)!
− † i¸ (I)
2/
Åg
(−1) I / ¸56
= −† i¸ (I) + I ó ¸56 (q)! († − 1 + q)!
2/
Åg
= −† i¸ (I) + Ii¸56 (I) .

7.10 References / Bibliography:


1. Advanced Engineering Mathematics, Erwin Kreyszig, John Wiley & Sons, Inc.
2. Intoduction to mathematical physics, Charlie Harper.
3. Mathematical physics, P.P. Gupta, RPS Yadav, GS Malik, NK Kashyap.
4. Essential mathematical methods, K F Riley and M P Hobson
5. Introductory course in Differential equations, Daniel A. Murray.

7.11 Terminal and model Questions :

Q1: Show that (}+D) „} D (l) = (E} + D)l„} (l) − } „}5D (l).
Q2: Proof the following recurrence relations:
1. º′· (l) = E Yº·5D (l) − º· D (l)Z
D

2. º· (l) = E· Yº·5D (l) + º· D (l)Z.


l

Q3: Find a power series solution in powers of Iof the following differential equation:
1. J @@ + I J @ + J = 0
2. J @@ + I J = 0
3. (1 − I / )J @@ − I J @ + 2 J = 0.

1. 2I J @@ + 2 J @ + J = 0
Q4: Find the singularity point and solution of following differential equation:

2. (I + 2)/ J @@ + (I + 2)J @ − J = 0.

207
MSCPH501

______________________________________________
UNIT 8: Hermite and Laguerre differential equations

Structure
8.1 Objective
8.2 Introduction
8.3 Hermite differential equation and polynomial
8.3.1 Solution of Hermite equation
8.3.2 Generating function
8.3.3 Orthogonality
8.3.4 Rodrigue's formula of Hermite function
8.3.5 Recurrence relations of Hermite Polynomial
8.4 Laguerre differential equation and polynomial

8.4.1 Solution of Laguerre’s equation

8.4.3 Recurrence relations of Laguerre’s Polynomial Î} (l)


8.4.2 Generating function

8.4.4 Orthogonality
8.4.5 Rodrigues Formula

8.5 Summary
8.6 Answer to SAQs
8.7 References / Bibliography

8.8 Terminal and model Questions

8.1 Objective
We will apply the mathematical techniques of power series solution and Frobenious method
learned in the previous unit to find the solution of differential equation which is of the form
ofHermite and Laguerre. We will also learn the properties, like generating function,
orthogonality, recurrence relations and Rodrigue’s relation of the Hermite and Laguerre
polynomials and their proofs.

208
MSCPH501

8.2 Introduction
In Unit 7 you have already learned to solve two special differential equations whose solution
exists only in a defined range. In this unit we will seek solution of rest two of the four special
forms of second order differential equations which are linear and homogeneous in nature. The
differential equations are:

J @@ − 2I J @ + 2 + J = 0 ; + > 0 4N 4 4 .
c) Hermite Differential Equation:

d) Laguerre differential equation:

I J @@ + (1 − I)J @ + 4 J = 0.

The solution of the differential equation is a polynomial which arises as a part of solution of
important fields. In numerical analysis we can observe presence of Hermite and Laguerre
polynomials as a solution of Gaussian quadrature integration method. Depending on the weight
function the methods are called as Gauss-Hermite or Gauss-Laguerre quadrature. In physics,
Quantum mechanics, we can see Hermite polynomials arises as eigen states of the quantum
harmonic oscillator, whereas Laguerre polynomials are seen as solution of radial Schrödinger
equation for a one-electron atom like Hydrogen. So it is really beautiful to learn the behavior and
properties of the above differential equations.

8.3 Hermite Differential equation and polynomial


Any solution of Hermite Differential Equation:

J @@ − 2I J @ + 2 + J = 0. (D)

(+is positive constant) is called a Hermite Function or Polynomial o© (I). In the equation
O(I) = −2I and ‚(I) = 2+, which is finite for all I so one can obtain solution of the equation
using power series method.

8.3.1 Solution of Hermite equation


We can apply power series solution to the differential equation with Ig = 0. Put the expansion
of power series given by J = ∑r ©Åg © I and its derivatives J = ∑©Å6 4 © I
© @ r ©56
and J @@ =
∑©Å/ 4 (4 − 1) © I to Eq. (1) we get,
r ©5/

r r r

ó 4(4 − 1) ©I
©5/
− 2I ó 4 ©I
©56
+2+ó ©I
©
= 0
©Å/ ©Å6 ©Åg

209
MSCPH501

r r r

ó 4(4 − 1) ©I
©5/
− 2ó4 ©I
©
+ 2+ó ©I
©
= 0
©Å/ ©Å6 ©Åg
r r r

ó 4(4 − 1) ©I
©5/
− 2 Ïó 4 © − +ó ©Ð I
©
= 0. (E)
©Å/ ©Å6 ©Åg

Expand the series in Eq.(2) and collect the coefficients of same power of I:

Coefficient of I g ∶ 2 + 2+ =0 =
5/É
/ g or / / g

Coefficient of I ∶ 3×2 − 2(1 − +) =0 =


/(65É)
_ 6 or _ _×/ 6

Coefficient of I / ∶ 4×3 − 2 (2 − +) =0 = /.
/(/5É)
c / or c c×_

So we can write a general term as

Coefficient of l¡ ∶ ( + 2) × ( + 1) − 2 ( − +) = 0 or |¡ = (¡ |¡ (‘)
E(¡5Ä)
• / • E E)×(¡ D)

= 0,1,2,3,4, … we can get all the coefficients in terms of g and 6 which is an arbitrary
So we obtain the recurrence relation or recurrence formulain the form of Eq.(3) and with

constants.

= 0,1,2,3,4, … in Eq. (3) we get


−2+
• Put the value of

=0∶ =
/
2 g

2(1 − +)
=1∶ =
_
3! 6

2/ (2 − +) +
=2∶ = −
c
4! g

2/ (1 − +)(3 − +)
=3∶ =
š
5! 6

2_ + (2 − +)(4 − +)
=4∶ = −
a
6! g

2_ (1 − +)(3 − +)(5 − +)
=5∶ =

7! 6

210
MSCPH501

and so on so that we can write even coefficients ( /# ) in terms of g and odd coefficients
( /# 6 ) in terms of 6 Q = 1,2,3,4,5, …. Such that
2# + (2 − +)(4 − +) … … (2 − 2 − +)
/# = −
(2 )! g

2# (1 − +)(3 − +)(5 − +) … … (2 − 1 − +)
=
/# 6
(2 + 1)! 6

‡ = |• ‡D (l) + |D £E (l)
the solution of differential equation can be expressed as

with

2# + (2 − +)(4 − +) … … (2 − 2 − +) /#
r

J6 = 1 + ó − I
(2 )!
#Å6

2# (1 − +)(3 − +)(5 − +) … … (2 − 1 − +) /#
r

J/ = I + ó I 6
.
(2 + 1)!
#Å6

Polynomials of solution when + = 0, 1, 2,3 … (reduce the complete solution to a finite


polynomial)

+ = 0 ∶ J6 (I) = 1
+ = 1 ∶ J/ (I) = I
+ = 2 ∶ J6 (I) = (1 − 2I / )
2 4
+ = 3 ∶ J/ (I) = (I − I _ )+ = 4 ∶ J6 (I) = (1 − 4I / + I c )
3 3
4 _ 4 š
+ = 5 ∶ J/ (I) = ¦I − I + I §.
3 15

Since |¡ = (¡ |¡ for Ä = ¡we have |¡ = •and hence |¡ = |¡ = ⋯ = •.


E(¡5Ä)
E E)×(¡ D) E ™ Ÿ

If ¡is even: solution is the polynomial with even power of I.

If ¡is odd: solution is the polynomial with odd power of I.

=
(56)g ©!
We arbitrarily choose coefficient © g
!
a multiplicative constant so that coefficient of
&
the term I © is 2© . The resulting solution is Hermite polynomial,Ñ} (l)in general expressed as

}! (−D){
ª

Ñ} (l) = ó (El)}5E{ . (™)


{! (} − E{)!
{Å•

211
MSCPH501

Where the integer ª is E when } is even and when } is odd.


} }5D
E

First few Hermite polynomials are

1. og (I) = 1.

2. o6 (I) = 2I .

3. o/ (I) = 4I / − 2.

4. o_ (I) = 8I _ − 12I .

5. oc (I) = 16I c − 48I / + 12.

Figure 2: Graph of Hermite Polynomialsover the domain (−∞, ∞) for n = 1,2,3 and 4.
(Source :https://mathworld.wolfram.com/images/eps-gif/HermiteH_1000.gif)

8.3.2 Generating function

The generating function of Hermite Polynomial is given as

Ñ} (l) }
•ElŒ5Œ = ó Œ . (š)
E

}!
}

Let’s work out how we can write Eq. (5)


r r
1 1
/=Œ
= ó(2I)• •
4N 5Œ &
= ó(−1)• /•
.
! !
•Åg •Åg

Therefore,

212
MSCPH501

r r
1 1 1
/=Œ 5Œ &
= ó(2I) • •
ó(−1)• /•
= ó ó(−1)• (2I)• • /•
! ! ! !
•Åg •Åg • •

put = 4 − 2 in above equation,

1
/=Œ5Œ &
= ó ó(−1)• (2I)©5/• ©
(4 − 2 )! !
©Å/• •Åg

multiply and divide by 4! on R.H.S. in above equation, and change the upper limit of summation
over from ∞ to S = / to avoid the negative factorial term in factor (4 − 2 )!.
©

(−1)• 4! (2I)©5/• Ñ} (l) }


t
1
• ElŒ5ŒE
= ó ~ó ¿ ©
= ó Œ .
4! (4 − 2 )! ! }!
© • }

8.3.3 Orthogonality

The orthogonality property of the Hermite polynomial is given as


r

â •5l Ñ} (l)Ñ… (l) ~l = √Ò E} }! Ç…} . (Ÿ)


E

5r

SAQ 1:Show that orthogonality relation for Hermite polynomial is given by Eq. (6).

8.3.4 Rodrigue's formula of Hermite function

The Rodrigue's formula for Hermite polynomial is given as

~} 5lE
Ñ} (l) = (−D)} •l Y• Z. (œ)
E

~l}

SAQ2: Proof the Rodrigue's relation Eq.(7) of Hermite polynomial.

8.3.5 Recurrence relation for Hermite polynomial (Ñ} (l))

Some of the recurrence relations for Hermite polynomial are

I. o©@ (I) = 2 4 o©56 (I) ; 4 ≥ 1.

213
MSCPH501

2 I o© (I) = 2 4 o©56 (I) + o© 6 (I).


o©@ (I) = 2 I o© (I) − o© 6 (I).
II.
III.

Proof:

(I) from generating function we have

∑©Åg =
vg (=) © /=Œ5Œ &
©!
.

Differentiate both side with respect to I

o©@ (I) o© (I) o© (I)


ó ©
= 2 /=Œ5Œ &
= 2 ó ©
= 2 ó © 6
4! 4! 4!
©Åg ©Åg ©Åg

©
comparing both side the coefficient of

o©@ (I) o©56 (I)


= 2 ; 4≥1
4! (4 − 1)!

4 o©56 (I) o©56 (I)


= 2 == 24 .
4 (4 − 1)! (4)!

Therefore,

o©@ (I) = 2 4 o©56 (I) ; 4 ≥ 1.

(II) from generating function we have

o© (I)
ó ©
= /=Œ5Œ &
4!
©Åg

differentiating both side with respect to

o© (I) o© (I) o© (I)


ó4 ©56
= 2I /=Œ5Œ &
− 2 /=Œ5Œ &
= 2I ó ©
− 2 ó ©
4! 4! 4!
©Å6 ©Åg ©Åg

o© (I) o© (I) o© (I)


ó ©56
= 2I ó ©
− 2ó © 6
(4 − 1)! 4! 4!
©Å6 ©Åg ©Åg

o© (I) o© (I) o© (I)


2I ó ©
= ó ©56
+ 2ó © 6
4! (4 − 1)! 4!
©Åg ©Å6 ©Åg

©
comparing both side the coefficient of

214
MSCPH501

o© (I) o© 6 (I) o©56 (I) o© 6 (I) o©56 (I)


2I = + 2 = + 24 .
4! (4)! (4 − 1)! (4)! (4)!

Hence, we have

2 I o© (I) = 2 4 o©56 (I) + o© 6 (I).

(III) from the first recurrence relation we have

o©@ (I) = 2 4 o©56 (I)

from second recurrence relation we have

2 I o© (I) = 2 4 o©56 (I) + o© 6 (I)

or, 2 4 o©56 (I) = 2 I o© (I) − o© 6 (I)

put this in first recurrence relation on R.H.S., we get

o©@ (I) = 2 I o© (I) − o© 6 (I).

8.4 Laguerre differential equation and polynomial

Laguerre differential equations is

l‡@@ + (D − l)‡@ + }‡ = • (DD)

or

(D − l) @ }
‡@@ + ‡ + ‡ = •.
l l

Thus, O(I) = = and‚(I) = = , implies I = 0 is a regural singular point. So we get the solution
65= ©
8.4.1 Solution of Laguerre’s equation

of the differential equation using Frobeneous method. Assuming solution of the form
r

J= ó I •

Åg

J′ = ó (q + ) I •56

Åg

215
MSCPH501

J′′ = ó (q + )(q + − 1) I •5/


.
Åg

Replace above expressions in Laguerresdifferential equation (11) we get


r r

I Ï ó (q + )(q + − 1) I •5/
Ð + (1 − I) Ï ó (q + ) I •56
Ð
Åg Åg
r

+ 4 Ïó I •
Ð=0
Åg

r r r

~ ó (q + )(q + − 1) + ó (q + )¿ I •56
+ 4 Ïó I •
Ð
Åg Åg Åg
r

− ó (q + ) I •

Åg
= 0. (12)

To find the indicial Equation put the coefficient of lowest power of I . . I •56 for q = 0 to
zero,

Indicial equation :j ( − 1) + l g = 0; g ≠ 0.

Therefore, j ( − 1) + l = 0 and the solution of this equation is j /


= 0l = 0 is a
double root.

To find recursion relation: equate coefficient of I • •


to zero in Eq.(12)

( + + 1)( + ) • 6 + ( + + 1) • 6 +4 • − ( + ) • =0

( + + 1)/ • 6 + (4 − − ) • =0

( + − 4)
= ; = 0,1,2,3, … (13)
• 6
( + + 1)/ •

Now roots are = 0, put this in recursion relation


−4
= (14)
• 6
( + 1)/ •

= 0: 6 = −4 g = (−1)4 g

1−4 4 (4 − 1)
= 1: = = (−1)/
/
2/ 6
2/ g

216
MSCPH501

2−4 4 (4 − 1)(4 − 2)
= 2: = = (−1)_
_
3/ /
3/ × 2/ g

3−4 4 (4 − 1)(4 − 2)(4 − 3)


= 3: = = (−1)c g.
c
4/ _
4/ × 3/ × 2/
In general we can write

4 (4 − 1)(4 − 2) … … (4 − + 1)
= (−1)•

( !)/ g

multiplying and divide by (4 − )! on right hand side

4 (4 − 1)(4 − 2) … … (4 − + 1) × (4 − )!
= (−1)•

( !)/ (4 − )! g

4!
= (−1)• (15)

( !)/ (4 − )! g.

Now put = 0 in the assumed solution J and replace by recursion relation Eq.(15) for =q
r ©
4!
J= ó I = ó (−1) I .
g
(q!)/ (4 − q)!
Åg Åg

If we take g = 4! then solution J is known as Laguerre polynomial (ĩ (I))

(}!)E
}

Î} (l) = ó (−D)… l… . (DŸ)


(…!)E (} − …)!
…Å•

1. ƒg (I) = 1.
First few Laguerre polynomials are

2. ƒ6 (I) = −I + 1.
1
3. ƒ/ (I) = (I / − 4I + 2).
2
1
4. ƒ_ (I) = (−I _ + 9I / − 18I + 6).
6

217
MSCPH501

Figure 3: Graph of few Laguerre polynomials for l ∈ j−D, šland } = D, E, … , š.


(Source: https://mathworld.wolfram.com/images/eps-gif/LaguerreL_1000.gif)

8.4.2 Generating function


r
Î} (l) } D l³
ó ³ = ÀÁÂ ž− Ÿ. (17)
}! D−³ D−³
}Å•

SAQ3: Show that generating function for Laguerre’s polynomial is given by Eq.(17)

8.4.3Recurrence relations of Laguerre’s Polynomial Î} (l)


1. Î} D (l) + (l − E} − D)Î} (l)+ }E Î}5D (l) = •.

Proof: use generating function


r
1 IK ƒ© (I) ©
exp ž− Ÿ= ó K
1−K 1−K 4!
©Åg

Or, exp ž− 65>Ÿ = (1 − K) ∑r K differentiating w.r.t. K


=> sg (=) ©
©Åg ©!

IK I IK ƒ© (I) ©56 ƒ© (I) ©


exp ž− Ÿ − − = (1 − K) ó 4 K −ó K
1−K 1 − K (1 − K)/ 4! 4!

I IK I ƒ© (I) © ƒ© (I) ©56 ƒ© (I) ©


− exp ž− Ÿ=− ó K = (1 − K) ó 4 K −ó K
(1 − K)/ 1−K 1−K 4! 4! 4!

218
MSCPH501

ƒ© (I) © ƒ© (I) ©56 ƒ© (I) ©


−I ó K = (1 − K)/ ó 4 K − (1 − K) ó K .
4! 4! 4!

Equate the coefficients of K ©

ĩ ĩ 6 ĩ ĩ56 ĩ ĩ56
−I = −2 + − +
4! 4! (4 − 1)! (4 − 2)! 4! (4 − 1)!

4! ĩ 4! ĩ56 4! ĩ56
Iƒ© + ƒ© −2 + − ƒ© + =0
6
(4 − 1)! (4 − 2)! (4 − 1)!

So we get

ƒ© 6 (I) + (I − 24 − 1)ƒ© (I)+ 4/ ƒ©56 (I) = 0.

2. Î@} (l) + }Î}5D (l) − }Î@}5D (l) = •.

Proof: use generating function


r
IK ƒ© (I) ©
exp ž− Ÿ = (1 − K) ó K .
1−K 4!
©Åg

differentiating w.r.t. I
r
K IK ƒ@© (I) ©
− exp ž− Ÿ = (1 − K) ó K
1−K 1−K 4!
©Åg

r r
ƒ© (I) © ƒ@© (I) ©
−K ó K = (1 − K) ó K .
4! 4!
©Åg ©Åg

Equate the coefficient of K ©

ƒ©56 ƒ@© ƒ@©56


− = −
(4 − 1)! 4! (4 − 1)!

4! ƒ©56 4! ƒ@©56
+ ƒ@© − =0.
(4 − 1)! (4 − 1)!

So we get

ƒ@© (I) + 4ƒ©56 (I) − 4ƒ@©56 (I) = 0 .

Problem: show that Iƒ@@© (I) + (1 − I)ƒ@© (I) + 4 ƒ© (I) = 0 .

219
MSCPH501

8.4.4 Orthogonality
r
Î… (l) Î} (l)
â •5l ~l = Ç…} . (D•)
…! }!

Proof: From generating function we have


r
ƒ© (I) © IK I
ó K = (1 − K)56 exp ž− Ÿ = (1 − K)56 exp žI − Ÿ … (‡. 1)
4! 1−K 1−K
©Åg

and
r
ƒ (I) I I
ó = (1 − )56 exp ¦− § = (1 − )56 exp žI − Ÿ … (‡. 2)
q! 1− 1−
Åg

multiply Eq. (‡. 1)and Eq. (‡. 2) we get


I I
r r
ƒ© (I) ƒ (I) exp žI − 1 − KŸ exp žI − 1 − Ÿ
óó K =
©
… (‡. 3)
4! q! 1−K 1−
©Åg Åg

multiplying both side of Eq. (‡. 3) by 5=


and integrate on I from 0 to ∞

I I
exp žI − 1 − KŸ exp žI − 1 − Ÿ
r r
ƒ© (I) ƒ (I)
ó ó ´â 5=
NI µ K © = â 5=
NI … (‡. 4)
4! q! 1−K 1−
© g g

I I I I
exp ž2I − 1 − K − 1 − Ÿ exp žI − 1 − K − 1 − Ÿ
r r

RHS = â 5=
NI = â NI
(1 − K)(1 − ) (1 − K)(1 − )
g g
r
1 1 1
= â exp ¼−I ¦ − − 1§½ NI
(1 − K)(1 − ) 1−K 1−
g

= 1 + 65·
6 ·
(65·)
now we can write

220
MSCPH501

r
1 K
RHS = â exp ¼−I ¦1 + − §½ NI
(1 − K)(1 − ) 1−K 1−
g
r
1 1 K
= Ô− K exp ¼−I ¦1 + − §½W
(1 − K)(1 − ) ž1 + − Ÿ 1−K 1−
1−K 1− g
1 (1 − K)(1 − ) 1
= ¯0 + ° =
(1 − K)(1 − ) (1 − K)(1 − ) + K(1 − ) + (1 − K) (1 − K )
= ó K© ©

©Åg

Therefore, we have
r
ƒ© (I) ƒ (I)
ó ó ´â 5=
NI µ K © = ó K© ©
= óó† K©
4! q! ©
© g ©Åg ©

when q = 4 LHS is non zero and the integral is equal to one as the coefficient of K © , otherwise for
q ≠ 4 integral is zero. Therefore the orthogonality relation for Laguerre polynomial is
r
Î… (l) Î} (l)
â •5l ~l = Ç…} .
…! }!

8.4.5 Rodrigues Formula

~} } 5l
Î} (l) = •l (l • ). (Dž)
~l}
Proof: Generating function is given as
r
IK 1 ƒ© (I) ©
(1 − K) 56
exp ž− Ÿ = (1 − K)56 exp æ¦1 − § Iç = ó K … (Õ. D)
1−K 1−K 4!
©Åg

differentiate Eq.(+. 1) 4 times with respect to K (according to Leibnitz theorem) we get

N© I N© ƒ/ / ƒ© © ƒ© 6 ©
=
º(1 − z)56
exp ž− Ÿ» = æƒ + ƒ K + K + ⋯ + K + K 6
+ ⋯ ç . (Õ. E)
NK © 1− z NK © g 6
2! 4! (4 + 1)!

• To evaluate differential factor of LHS of Eq.(+. 2) . . ~³} º(D − Ö)5D ÀÁÂ ž− D5 ÖŸ» we proceed as
~} l

follows:

evaluate derivative for } = D in the limit ³ → •

221
MSCPH501

N I 1−I−K I
º(1 − z)56 exp ž− Ÿ» = exp ž− Ÿ
NK 1− z (1 − K)_ 1− z

N I N
lim º(1 − z)56 exp ž− Ÿ» = (1 − I)e5= = (I 5=
)
>→g NK 1− z NI

evaluate derivative for } = E in the limit ³ → •

N/ I N /
lim º(1 − z)56 exp ž− Ÿ» = (I / − 4I + 2)e5= = (I 5=
)
>→g NK / 1− z NI

evaluate derivative for } = ‘ in the limit ³ → •

N_ I N _
lim º(1 − z)56 exp ž− Ÿ» = (6 − 18I + 9I / − I _ )e5= = (I 5=
)
>→g NK _ 1− z NI

similarlily, we can show and express that nth orderderivative in the limit ³ → •

~} l ~} } 5l
×Ø÷ = •l ^ˆ‰ º(D − Ö)5D
ÀÁÂ ž− Ÿ» = • l (l • ).
³→• ~³} D− Ö ~l}

• To evaluate RHS of Eq.(+. 2)


r
N© ƒ/ ƒ© ƒ© 6 © N© ƒ© (I) ©
+oà = æƒg + ƒ6 K + K / + ⋯ + K © + K 6
+ ⋯ç = © ó K
NK © 2! 4! (4 + 1)! NK 4!
©Åg

proceed as follows:

N © N/ © N_ ©
K = 4 K ©56 ; K = 4 (4 − 1)K ©5/ ; K = 4 (4 − 1)(4 − 2)K ©5_ ;
NK NK / NK _

N# ©
K = 4 (4 − 1)(4 − 2) … (4 − + 1)K ©5# .
NK #

Therefore, we get for =4

³ = }!
~} }
~³}

similarly we can write

N# ©
K •
= (4 + )(4 + − 1)(4 + − 2) … (4 + − + 1)K © •5#
.
NK #

Therefore, again for = 4 we get

³ = (} + C)(} + C − D)(} + C − E) … (C + D)³C = ³ .


~} } C (} C)! C
~³} C!

Hence, using above relations and with the fact that ĩ is funtion of I we can evaluate RHS in the
limt³ → •

222
MSCPH501

N© ƒ/ ƒ© ƒ© 6 © 6 ƒ© / © /
+oà = æƒg + ƒ6 K + K / + ⋯ + K © + K + K + ⋯ç
NK © 2! 4! (4 + 1)! (4 + 2)!
ĩ (4 + 1)! ĩ 6 (4 + 2)! ĩ / /
= j0 + 0 + 0 + ⋯ + 4! + K+ K +⋯
4! 1! (4 + 1)! 2! (4 + 2)!

r
N© ƒ© (I) ©
lim +oà = lim © ó K = ƒ© (I).
>→g >→g NK 4!
©Åg

Hence, in the limt ³ → • we get Rodrigue's formula

~} } 5l
×Ø÷ = •l (l • ) = Î} (l) = ÕÑÙ.
~l}

8.5 Summary

• Hermite differential equation and polynomial

J @@ − 2I J @ + 2 + J = 0 ; + > 0 4N 4 4 .

• Hermite Polynomial
}! (−D){
ª

Ñ} (l) = ó (El)}5E{ .
{! (} − E{)!
{Å•
• Properties of Hermite function

Ñ} (l) }
o Generating function

•ElŒ5Œ = ó Œ .
E

}!
}

r
o Orthogonality

â •5l Ñ} (l)Ñ… (l) ~l = √Ò E} }! Ç…} .


E

5r

E ~
}
o Rodrigue's formula of Hermite function

Ñ} (l) = (−D)} •l Y•5l Z.


E

~l }

1. Ñ@} (l) = E } Ñ}5D (l) ; } ≥ D.


o Recurrence relations of Hermite Polynomial

2. E l Ñ} (l) = E } Ñ}5D (l) + Ñ} D (l).


3. Ñ@} (l) = E l Ñ} (l) − Ñ} D (l).
4. Ñ@@} (l) = E l Ñ} (l) − E } Ñ} (l).
@

l‡@@ + (D − l)‡@ + }‡ = •.
• Laguerredifferential equation and polynomial

• Laguerre’s Polynomial

223
MSCPH501

(}!)E
}

Î} (l) = ó (−D)… l… .
(…!)E (} − …)!
…Å•
• Properties of Laguerre function

r
o Generating function
Î} (l) } D l³
ó ³ = ÀÁÂ ž− Ÿ.
}! D−³ D−³
}Å•
o Recurrence relations of Laguerre’s Polynomial Î} (l)
1. Î} D (l) + (l − E} − D)Î} (l)+ }E Î}5D(l) = •.
2. Î@} (l) + }Î}5D(l) − }Î@}5D (l) = •.

r
Î… (l) Î} (l)
o Orthogonality

â •5l ~l = Ç…} .
…! }!

o Rodrigues Formula

~} } 5l
Î} (l) = •l (l • ).
~l}

8.6 Answer to SAQs


SAQ1: Proof:

We will use generating function to show the orthogonality relation as follows

o© (I) o (I)
/=Œ5Œ &
= ó ©
……..( ) 4N /=Œ5Œ &
= ó … … . . (“)
4! q!
©
multiply Eq. ( ) and Ђ. (“)

o© (I)o (I)
c=Œ5/Œ &
= óó ©
. ………( )
4! q!
©

Multiply both side of Eq. ( )by 5= &

5= &
o© (I)o (I)
5= & c=Œ5/Œ &
= 5= & c=Œ5cŒ & /Œ &
= 5(=5/Œ)& /Œ &
= óó ©
. … (N)
4! q!
©

Integrating Eq.(d) from −∞ ∞ on I, and put J = I − 2 , ℎ 4 NI = NJ then use


definition of Gamma function
& g
´5r 5(=5/Œ) /Œ NI ´5r 5(<) NJ = √Ù = √ Ù ∑r
r & & /Œ & r & /Œ & Y/Œ Z
L.H.S. = = ©Åg ©!

224
MSCPH501

r
(2 / )© o© (I)o (I)
r 5= &
√Ù ó = â óó ©
NI …( )
4! 4! q!
©Åg 5r ©

when … = }; on comparing the coefficient of ŒE} in Eq.( )


r

â •5l YÑ} (l)Z ~l = √Ò E} }!


E E

5r

and … ≠ }; then comparing the coefficient of Œ… }


in Eq.( )
r

â •5l Ñ} (l)Ñ… (l) ~l = •.


E

5r

SAQ2: Generating function for Hermite polynomial is

o© (I)
/=Œ5Œ &
= ó ©
4!
©

o© (I)
= & 5 = & /=Œ5Œ &
= •= & 5 (=5Œ)& –
= ó ©
4!
©

differentiating both side } times partially with respect to , and then put = 0.

=› (
©
›© o© (I)
•= & 5 (=5Œ)& –
)( = Ïó ©
Ð
› © ŒÅg › © 4!
© ŒÅg

›© o6 o/ o©
= ¦o g
+ + /
+ ⋯+ ©
+ …..§
› © g 1! 2! 4! ŒÅg

= o© (I).

Therefore,

›©
o© (I) = =&
¦ ©( 5 (=5Œ)&

› ŒÅg

let K = − I, , NK = N

›© ›©
o© (I) = =&
¦ ©( 5> &
)§ = =&
¦(−1) (© 5= &
)§.
›K >Å5= ›I ©

225
MSCPH501


Hence, we get Eq. (7)

o© (I) = (−1)© =&


Y 5= &
Z.
NI ©

1 IK 1 IK (IK)/ (−1)# (IK)#


SAQ3: R.H.S. (of Eq. (17))

= exp ž− Ÿ= ¼1 − + − ⋯ + + ⋯½
1−K 1−K 1−K 1 − K 2! (1 − K)/ ! (1 − K)#
r r
(−1)# (IK)# (−1)# (IK)#
=ó = ó (1 − K)5#56 .
! (1 − K)# 6 !
#Åg #Åg

Use binomial expansion


r
(−1)# (I)# # ( + 1)( + 2) / ( + 1)( + 2) … ( + r) µ
RHS = ó K Ê1 + ( + 1)K + K + ⋯+ K
! 2! r!
#Åg

+ ⋯ç.

Multiply and divide the term in summation by !


r
(−1)# (I)# # ( + 1)( + 2) / ( + 1)( + 2) … ( + r) µ
RHS = ó K Ê1 + ( + 1)K + K + ⋯+ K + ⋯Ë
! 2! r!
#Åg
!
×
!
r r
(−1)# ( + r)( + r − 1) … ( + 2)( + 1) ! (I)# #
RHS = ó ó K µ
! r! !
#Åg µÅg
r r
(−1)# ( + r)! (I)# # µ
= óó K .
( !)/ r!
#Åg µÅg

Put + r = 4 r = 4 − , so for r = 0, 4 will start from 4 = in the summation, since summation


over starts from = 0 so e can take summation over 4 from 4 = 0 with the condition that 4 is always
greater than to restrict the appearance of any negative factorial term because of replacement of r! =
(4 − )!. Hence we can take summation of from 0 to 4 and summation of 4 from 0 to ∞.
r © r © r
(−1)# (4)! (I)# © 1 (−1)# (4)! (I)# ƒ© (I) ©
RHS = ó ó K = ó ó 4! K © = ó K = L. H. S.
( !) (4 − )!
/ 4! ( !) (4 − )!
/ 4!
©Åg #Åg ©Åg #Åg ©Åg

8.7 References / Bibliography

226
MSCPH501

1. Advanced Engineering Mathematics, Erwin Kreyszig, John Wiley & Sons, Inc.
2. Intoduction to mathematical physics, Charlie Harper.
3. Mathematical physics, P.P. Gupta, R P S Yadav, G S Malik, N K Kashyap.
4. Essential mathematical methods, K F Riley and M P Hobson.
5. Introductory course in Differential equations, Daniel A. Murray.
6. Mathematical Methods for Physicists: Arfken.
7. Mathematical Physics,H.K. Das, S. Chand, Delhi.

8.8 Terminal and model questions

Q1. Show that

N/ I
lim º(1 − z)56 exp ž− Ÿ» = (I / − 4I + 2)e5= .
>→g NK / 1− z
Q2. Show that

N_ I
lim º(1 − z)56 exp ž− Ÿ» = (6 − 18I + 9I / − I _ )e5= .
>→g NK _ 1− z
Q3. Show that
r
N© ƒ© (I) ©
lim © ó K = ƒ© (I).
>→g NK 4!
©Åg

o©@@ (I) = 2 I o©@ (I) − 2 4 o© (I).


Q4. Show that

o©@@ (I) = 4 4 (4 − 1)o©5/.


Q5. Prove that

Q6. Evaluate
r

âI 5= &
o© (I)o (I)NI.
5r

227
MSCPH501

_________________________________________ ____ _____


UNIT 9: FOURIER INTEGRAL AND TRANSFORMS
Structure
9.1 Objectives
9.2 Introduction
9.3 Fourier Series
9.4 Some Important Results
9.5 Fourier Integral
9.6 Fourier Integral Theorem
9.7 Fourier Sine and Cosine Integrals
9.8 Fourier’s Complex Integrals
9.9 Fourier Transforms:
9.10 Fourier Sine Transforms
9.11 Fourier Cosine Transform
9.12 Properties of Fourier Transforms
9.12.1 Linear Property
9.12.2. Change of Scale Property
9.12.3 Shifting Property
9.13 Fourier Transform of Derivatives
9.14 Fourier Transform of Partial Derivative of a Function
9.15 Application to Simple Heat Transfer Equations
9.16 Summary
9.17 Glossary
9.18 Terminal Questions
9.19 Answers
9.20 References
9.21 Suggested Readings

228
MSCPH501

9.1 OBJECTIVES
After studying this chapter we will learn about how Fourier transforms is useful many
physical applications, such as partial differential equations and heat transfer equations. With the
use of different properties of Fourier transform along with Fourier sine transform and Fourier
cosine transform, one can solve many important problems of physics with very simple way. Thus
we will learn from this unit to use the Fourier transform for solving many physical application
related partial differential equations.

9.2 INTRODUCTION

The central starting point of Fourier analysis is Fourier series. They are infinite series
designed to represent general periodic functions in terms of simple ones, namely, cosines and
sines. This trigonometric system is orthogonal, allowing the computation of the coefficients of
the Fourier series by use of the well-known Euler formulas, as shown. Fourier series are very
important to the engineer and physicist because they allow the solution of linear differential
equations and partial differential. Fourier series are, in a certain sense, more universal than the
familiar Taylor series in calculus because many discontinuous periodic functions that come up in
applications can be developed in Fourier series but do not have Taylor series expansions.
The Fourier Transform is a tool that breaks a waveform (a function or signal) into an
alternate representation, characterized by sine and cosines. The Fourier Transform shows that
any waveform can be re-written as the sum of sinusoidal functions.
The Fourier transform is a mathematical function that decomposes a waveform, which is
a function of time, into the frequencies that make it up. The result produced by the Fourier
transform is a complex valued function of frequency. The absolute value of the Fourier transform
represents the frequency value present in the original function and its complex argument
represents the phase offset of the basic sinusoidal in that frequency.
The Fourier transform is also called a generalization of the Fourier series. This term can
also be applied to both the frequency domain representation and the mathematical function used.
The Fourier transform helps in extending the Fourier series to non-periodic functions, which
allows viewing any function as a sum of simple sinusoids.
So for detailed knowledge of Fourier transform one should know about the Fourier series
and Fourier Integral. So we will start the brief review of Fourier series and then I will explain the
Fourier Integral and transforms in detailed.

9.3 FOURIER SERIES:

229
MSCPH501

Fourier series are infinite series that represent periodic functions in terms of cosines and
sines. As such, Fourier series are of greatest importance to the engineer and applied
mathematician. To define Fourier series, we first need some background material. A function f
(x) is called a periodic function if f ( x) is defined for all real x, except possibly at some points,
and if there is some positive number p, called a period of f (x) such that

Q(I + O) = Q(I) Q rr I.

that are not periodic are I, I / , I _ , = , cos ℎI . to mention just a few.


Familiar periodic functions are the cosine, sine, tangent, and cotangent. Examples of functions

If f(x) has a period of p then it has also a period of 2p

Q(I + 2O) = Q•(I + O) + O– = Q(I + O) = Q(I).

Q(I + 4O) = Q(I).


Or in general we can write

A Fourier series is defined as an expansion of a real function or representation of a real function


in a series of sines and cosines such as
r r

Q(I) = +ó cos 4I + ó “© sin 4I.


g
2 ©
©Å6 ©Å6
Where g , © , 4N “© are constants, called the Fourier coefficients of the series. We see that
each term has the period of 2Ù Hence if the coefficients are such that the series converges, its
sum will be a function of period 2Ù.

1 Û
The Fourier coefficients of f(x), given by the Euler formulas

g = â Q(I)NI
2Ù 5Û
1 Û
© = â Q(I) cos 4I NI 4 = 1, 2, 3, ….
Ù 5Û
1 Û
“© = â Q(I) sin 4I NI. 4 = 1, 2, 3, …
Ù 5Û

The above Fourier series is given for period 2Ù. The transition from period 2Ù to be period
O = 2ƒ is effected by a suitable change of scale, as follows. Let Q(I) have period = 2ƒ . Then
we can introduce a new variable v such that, Q(I) as a function of v, has period 2Ù.

O 2Ù Ù
If we set
I= ⇒ = I ⇒ = I.
2Ù O ƒ
This means = ±Ù corresponds to I = ±ƒ. This represent f , as function of v has a period of
2Ù. Hence the Fourier series is

230
MSCPH501

r r

Q( ) = +ó cos 4 + ó “© sin 4 .
g
2 ©
©Å6 ©Å6
= s I Fourier series for the period of (-L, L) is given by
Û
Now using
r r
Ù Ù
Q(I) = + ó © cos 4 I + ó “© sin 4 I.
g
2 ƒ ƒ
©Å6 ©Å6
This is Fourier series we obtain for a function of f(x) period 2L the Fourier series.

= ´5s Q( )N ,
The coefficient is given by
6 s
g s

1 s 4ÙI
= â Q(I) cos NI,
©
ƒ 5s ƒ

1 s 4ÙI
“© = â Q(I) sin NI.
ƒ 5s ƒ

9.4 SOME IMPORTANT RESULTS

´ 4 “I NI = ˜& ( 4 “I − “ “I).
˜= d Üd
›&
1.
´ “I NI = ( “I + “ 4 “I).
˜= d Üd

˜& ›&
2.
´g = NI = / .
r Ø" ˜= Û
3.
´g 5= NI = / .
r & √Û
4.
´5r (=5›)& ˜& NI = 4 “q . jq > 0l
r Ø" = Û̃ 5˜
5.

9.5 FOURIER INTEGRAL

Fourier series are powerful tools for problems involving functions that are periodic or are
of interest on a finite interval only. Since, of course, many problems involve functions that are
non-periodic and are of interest on the whole x-axis, we ask what can be done to extend the
method of Fourier series to such functions. This idea will lead to “Fourier integrals.”

9.6 FOURIER INTEGRAL THEOREM

Fourier integral theorem states that Q(I) = Û ´g ´5r Q( ) cos ¢( − I)N N¢


6 r r

Proof. We know that Fourier series of a function Q (x) in ( -c, c) is given by

231
MSCPH501

Q(I) = + ∑r cos + ∑r
©Å6 “© sin .
˜Ý ©Û= ©Û=
/ ©Å6 © Û Û

Where g , © 4N “© are given by

= ´5Û Q( )N ,
6 Û
g Û

1 Û

© = â Q( ) cos N ,

1 Û

“© = â Q( ) sin N .

Substituting the values of g , © 4N “© in above equation, we get


r
1 Û 1 Û 4Ù 4ÙI
Q(I) = â Q( )N + ó â Q( ) cos N cos
2 5Û 5Û ©Å6
r
1 Û
4Ù 4ÙI
+ó â Q( ) sin N , sin
©Å6 5Û

r
1 Û 1 Û 4Ù 4ÙI 4Ù 4ÙI
Q(I) = â Q( )N + ó â Q( ) æcos cos + sin sin çN
2 5Û 5Û ©Å6

r
1 Û 1 Û 4Ù 4ÙI
Q(I) = â Q( )N + ó â Q( ) æcos ¦ − §ç N
2 5Û 5Û ©Å6

r
1 Û 1 Û 4Ù
Q(I) = â Q( )N + ó â Q( ) ºcos ( − I)» N
2 5Û 5Û ©Å6

r
1 Û 4Ù
Q(I) = â Q( ) ~1 + 2 ó cos ( − I)¿ N .
2 5Û
©Å6

Since cosine functions are even functions i.e., cos (−,) = cos , the expression
r r
4Ù 4Ù
~1 + 2 ó cos ( − I)¿ = ó cos ( − I).
©Å6 ©Å5r

Hence, we have

232
MSCPH501

r
1 Û 4Ù
Q(I) = â Q( ) ~ ó cos ( − I)¿ N
2 5Û
©Å5r

r
1 Û Ù 4Ù
Q(I) = â Q( ) ~ ó cos ( − I)¿ N .
2Ù 5Û
©Å5r

We now let the parameter c approach infinity, transforming the finite interval [-c, c] into the
infinite interval (-∞ to +∞). We set
4Ù Ù
= ¯, 4N = N¯ Ú ℎ → ∞.

Then, we have

1 r r
Q(I) = â Q( ) ¯â Nω cos ¯( − I)° N
2Ù 5r 5r

1 r r
On simplifying

Q(I) = â â Q( ) ¯( − I) N¯ N . A N
Ù g 5r

9.7. FOURIER SINE AND COSINE INTEGRALS

Q(I) = ´g 4 ¯I N¢ ´g Q( ) 4 ¯ N .
/ r r
Û
(Fourier Sine Integrals)

Q(I) = ´g ¯I N¢ ´g Q( ) ¯ N .
/ r r
Û
(Fourier Cosine Integrals)

Proof: We can write

cos ¯( − I) = cos(¯ − ¯I) = cos ¯ cos ¯I + sin ¯ sin ¯I.

Using this expansion in Fourier integral theorem, we have

1 r r
Q(I) = â â cos ¯( − I) Nω N
Ù g 5r

1 r r
⇒ Q(I) = â â Q( )( ¯ cos ¯I + sin ¯ sin ¯I)Nω N
Ù g 5r

1 r r 1 r r
⇒ Q(I) = â â Q( )( ¯ cos ¯I Nω N + â â Q( ) sin ¯ sin ¯I Nω N .
Ù g 5r Ù g 5r

233
MSCPH501

Now to solve the above equation, we have two different cases, using the following conditions

˜
â Q(I)NI = 0. Q NN Q¢4 4

And
˜ ˜
â Q(I)NI = 2 â Q(I) NI. Q 4 Q¢4 4
5˜ g

Case I: when f(t) is even function: this means

⇒ Q( ) sin ¯ NN Q¢4 4 4N

Q( ) cos ¯ . 4 Q¢4 4

Hence

1 r r
â â Q( ) sin ¯ sin ¯I Nω N = 0.
Ù g 5r

And

1 r r 2 r r
⇒ Q(I) = â â Q( )( ¯ cos ¯I Nω N = â cos ¯I Nω â Q( ) ¯ N
Ù g 5r Ù g 5r

2 r r
Q(I) = â ¯I N¢ â Q( ) ¯ N .
Ù g g

This is known as Fourier cosine integral.

Case II: If f(t) is odd function: this means

⇒ Q( ) sin ¯ 4 Q¢4 4 4N

Q( ) cos ¯ . NN Q¢4 4

Hence

1 r r
â â Q( ) cos ¯ cos ¯I Nω N = 0.
Ù g 5r

And

234
MSCPH501

1 r r 2 r r
⇒ Q(I) = â â Q( ) 4 ¯ 4 ¯I N¯ N = â 4 ¯I N¯ â Q( ) 4 ¯ N
Ù g 5r Ù g 5r

2 r r
Q(I) = â 4 ¯I N¢ â Q( ) 4 ¯ N .
Ù g g

This is known as Fourier sine integral.

9.8. FOURIER’S COMPLEX INTEGRALS


We know from Fourier integral theorem

1 r r
Q(I) = â â Q( ) ¯( − I) N¯ N .
2Ù 5r 5r

Now adding
r r
Q(I) = â Q( )N â 4 ¯( − I) N¯ = 0.
2Ù 5r 5r

Since
r
â 4 ¯( − I)N¯ = 0. “ ¢ Q NN Q¢4 4
5r

1 r r
Hence
r r
Q(I) = â â Q( ) ¯( − I) N¯ N + â Q( )N â 4 ¯( − I) N¯
2Ù 5r 5r 2Ù 5r 5r

1 r r
Q(I) = â Q( ) N Êâ ¯( − I) + 4 ¯( − I) Ë N¯
2Ù 5r 5r

Q(I) = /Û ´5r Q( ) N „´5r … N¯


6 r r `â(Œ5=)

1 r r
Q(I) = â 5`â=
N¯ â Q( ) `âŒ
N .
2Ù 5r 5r

This relation is known as Fourier’s complex Integral.

Example 1. Express the following function

1 Úℎ 4 I ≤ 1=
Q(I) = U
0 Úℎ 4 I > 1

235
MSCPH501

as a Fourier integral. Hence evaluate


r
sin ¢ cos ¢I
â N¢.
g ¢

Solution: we know the Fourier Integral theorem, the Fourier Integral of a function Q(I) is given
by

1 r r
Q(I) = â â Q( ) ¯( − I) N¯ N .
Ù g 5r

Using ¯ = ¢ we have

1 r r
Q(I) = â â Q( ) ¢( − I) N¢ N
Ù g 5r

1 r 6
Q(I) = â â ¢( − I) N N¢. 4 Q( ) = 1
Ù g 56

Now integrating w.r.t. t we have

1 r sin ¢( − I) 6
Q(I) = â æ ç N¢
Ù g ¢ 56

1 r sin ¢(1 − I) + sin ¢(1 + I)


Q(I) = â Ê Ë N¢.
Ù g ¢

Now using sin p + 4à = 2 4


Ž i Ž5i
/ /
and solving it we will get

2 r sin ¢ cos ¢I
Q(I) = â N¢.
Ù g ¢

We can rewrite this


r
sin ¢ cos ¢I Ù
â N¢ = Q(I)
g ¢ 2

Ù Ù
r
sin ¢ cos ¢I ×1= , Q I<1
â 2
N¢ = ÆÙ 2 =
g ¢ × 0 = 0, Q I > 1.
2
ã
g
For x=1, which is a point of discontinuity of f(x), value of integral = =
& Û
/ c
Ans.

Self Assessment Question (SAQ) 1: Find the Fourier cosine integral of

236
MSCPH501

sin I, Q 0≤I≤Ù=
Q(I) =
0 Q I > Ù.

Self Assessment Question (SAQ) 2: Find the Fourier sine integral of

Q(I) = 5h=
.

´g N¯ = .
r â Ø" â= Û 5h=
h& â& /
Hence prove that

9.9. FOURIER TRANSFORMS:


From the Fourier complex integral we know that

1 r r
Q(I) = â 5`â=
N¯ â Q( ) `âŒ
N .
2Ù 5r 5r

We can rewrite the above expression as follows using ¯ =

1 r r
1 r
1 r
Q(I) = â 5`•=
N â Q( ) `•Œ
N = Ê â 5`•=
N ËÊ â Q( ) `•Œ
N Ë.
2Ù 5r 5r √2Ù 5r √2Ù 5r

´ Q( ) N = h( ) in above equation, we get


6 r `•Œ
√/Û 5r
Now using

1 r
Q(I) = â 5`•=
h( )N .
√2Ù 5r

Where h( ) is called the Fourier Transform of Q(I).

And Q(I) is called the Inverse Fourier transform of h( ).

Thus, we obtain the definition of Fourier transform is

Ä(¡) = Äjk(l)l = ´ k(Œ)•‹¡Œ ~Œ


D r
√EÒ 5r

k(l) = ´ •5‹¡l . Ä(¡)~¡.


D r
√EÒ 5r

237
MSCPH501

9.10. FOURIER SINE TRANSFORMS


We know that from Fourier sine integral

Q(I) = ´ sin I N ´g Q( ) sin N = % ´g sin I N Ê% ´g Q( ) sin N Ë.


/ r r / r / r
Û g Û Û

Now putting h( ) = %Û ´g Q( ) sin N .


/ r

We have

Q(I) = %Û ´g sin I N h( ).
/ r

In above equation h( ) is called Fourier Sine transform of Q(I)

E r
Ä(¡) = Ä¡ jk(l)l = ä â k(Œ) .ˆ- ¡Œ ~Œ.
Ò •

And, Q(I) given below is known as inverse Fourier Sine transform of h( )

E r
k(l) = ä â Ä(¡) .ˆ- ¡l ~¡.
Ò •

9.11 FOURIER COSINE TRANSFORM


From Fourier cosine integral we know that

2 r r
Q(I) = â ¯I N¢ â Q( ) ¯ N
Ù g g

Q(I) = %Û ´g cos I N Ê%Û ´g Q( ) cos N Ë.


/ r / r

Now putting h( ) = %Û ´g Q( ) cos N


/ r

Q(I) = %Û ´g cos I N h( ).
/ r

In above equation h( ) is called Fourier cosine transform of Q(I)

238
MSCPH501

Ä(¡) = Ä” jk(l)l = %Ò ´• k(Œ) ÌÍ. ¡Œ ~Œ.


E r

And, Q(I) given below is known as inverse Fourier cosine transform of h( )

k(l) = % ´• ÌÍ. ¡l Ä(¡)~¡.


E r
Ò

Example 2: Find the Fourier transform of 5˜= &


, where a>0.

Solution : The Fourier transform of f(x):

h•Q(I)– = ´5r Q(I) NI.


6 r `•=
√/Û

Hence

hå æ= ´5r NI = ´ NI
5˜= & 6 r 5˜= & `•= 6 r 5˜= & `•=
√/Û √/Û 5r

1 r •& •& 1 r `• & •&


§ 5
⇒ hå æ=
5¦=√˜5
5˜= &
â 5˜= & 5

`•=
c˜ NI = â /√˜ c˜ NI
√2Ù 5r √2Ù 5r

•&
5 r `• &
c˜ §
⇒ hå æ=
5¦=√˜5
5˜= &
â /√˜ NI.
√2Ù 5r

A¢ 4t I √ − = ¢ ⇒ NI =
`• •q
/√˜ √˜
in above expression we get,

•&
5 r r

⇒ hå 5˜= &
æ= â 5q&
N¢ Ê 4 â 5= &
NI = √Ù Ë
√2Ù 5r 5r

•& •&
5 5
c˜ c˜
⇒ hå 5˜= &
æ= √Ù = 4 .
√2Ù √2
Example 3: Find the Fourier transform of

2 Q |I| < =
Q(I) =
0 Q |I| >

Solution: We know that the Fourier transform of a function is given by

1 r
h•Q(I)– = â Q(I) `•=
NI.
√2Ù 5r

239
MSCPH501

Using the given value of f(x) we get,

1 ˜
2 ˜
h•Q(I)– = â 2 `•=
NI = â `•=
NI =
√2Ù 5˜ √2Ù 5˜

2 `•= ˜
2 4 „ `˜•
− 5`˜•

h•Q(I)– = Ê Ë = „ `˜•
− 5`˜•
…=
√2Ù 5˜ √2Ù √2Ù 2

4 2 sin
h•Q(I)– = sin = 2ä 4 .
√2Ù Ù

6
=
Example 4: Find Fourier Sine transform of .

We have to find the Fourier sine transform of Q(I) = .


6
=
Solution:

We know that from Fourier sine transform

2 r
h• jQ(I)l = ä â Q(I) sin I NI.
Ù g

Now using the value of Q(I) = =, we get,


6

h• jQ(I)l = %Û ´g sin I NI
/ r6
=

N
4 Ú ¢ 4t I = ⇒ NI = .

= %Û ´g N = %Û ž / Ÿ . ⇒ 4 ´g N =
/ r Ø" Œ / Û r Ø" Œ Û
Œ Œ /
We get

h• jQ(I)l = % / 4 .
Û
Hence

5 ˜=
Example 5: Find the Fourier Sine Transform of .
Solution: Here, Q(I) = 5 ˜=
.
The Fourier sine transform of Q(I):

h• jQ(I)l = %Û ´g Q(I) sin I NI.


/ r

On putting the value of Q(I) in (1), we get

240
MSCPH501

h• j 5 ˜= l
= %Û ´g sin I NI.
/ r 5 ˜=

On Integrating by parts, we get


r
2 5 ˜=
h• j 5 ˜= l
=ä j− 4 I− Il
Ù / + /
g
r ˜=
¢ 4t Êâ ˜=
sin “I NI = ( sin “I − “ “IË
g
/ + “/

= % º0 − (− )» = % ž Ÿ 4 .
/ 6 / •
Û ˜& •& Û ˜& •&

Example 6: Find the Fourier Cosine Transform of Q(I) = 5 5/=


+2 5š=
.

Solution: The Fourier Cosine Transform of Q(I) is given by

hÛ •Q(I)– = %Û ´g Q(I) cos I NI.


/ r

Putting the value of Q(I) , we get

2 r
hÛ •Q(I)– = ä â (5 5/=
+2 5š= )
cos I NI
Ù g

= 5 ´g cos I NI + 2 ´g cos I NI
r 5/= r 5š=

r ˜=
¢ 4t Êâ ˜=
“I NI = ( “I + “sin “I)Ë
g
/ + “/
r
= 5 º(5/)& (−2 cos I + sin I)» + 2 º(5š)& (−5 cos I +
d Ê&d d ʧd
•& g •&
r
sin I)»
g

= 5 º0 − (−2)» + 2 º0 − (−5)» = 5 ž•& cŸ + 2 ž•& Ÿ


6 6 / š
c •& /š •& /š

= 10 ž•& + •& Ÿ 4 .
6 6
c /š

Self Assessment Question (SAQ) 3: Find Fourier sine transform of Q(I) = 5=

Assessment Question (SAQ) 4: Find Fourier transform of Q(I)

241
MSCPH501

1 Q |I| < 1=
Q(I) = U
0 ℎ Ú
Assessment Question (SAQ) 5: Find Fourier cosine transform of Q(I)

I Q 0<I<1
Q(I) = Ô 2 − I Q 1 < I < 2=
0 Q I>2

9.12 PROPERTIES OF FOURIER TRANSFORMS

LINEAR PROPERTY: If h6 ( ) and h/ ( ) are Fourier transforms of Q6 (I)


and Q/ (I) respectively then
9.12.1

hj Q6 (I) + “ Q/ (I)l = h6 ( ) + “ h/ ( ). Úℎ 4N “ 4 4 .

Proof: we know from the definition of Fourier transform

1 r
h( ) = â Q(I) `•=
NI.
√2Ù 5r

We can write

1 r
h6 ( ) = â Q6 (I) `•=
NI.
√2Ù 5r

1
And
r
h/ ( ) = â Q/ (I) `•=
NI.
√2Ù 5r

Now

1 r
hj Q6 (I) + “ Q/ (I)l = â j Q6 (I) + “ Q/ (I)l `•=
NI
√2Ù 5r

1 r
1 r
= â Q6 (I) `•=
NI + “ â Q/ (I)l `•=
NI
√2Ù 5r √2Ù 5r

⇒ hj Q6 (I) + “ Q/ (I)l = h6 ( ) + “ h/ ( ). A N

9.12.2 CHANGE OF SCALE PROPERTY:

We know that Fourier transform equation is given by

242
MSCPH501

1 r
h( ) = â Q(I) `•=
NI.
√2Ù 5r

Then

1
h•Q( I)– = h ž Ÿ.

Proof: we know

1 r
h( ) = â Q(I) `•=
NI
√2Ù 5r

1 r
N
⇒ h•Q( I)– = â Q( I) `•=
NI. æ4 Ú O¢ I = ⇒ NI = ç
√2Ù 5r

We have

1 r • N 1 1 r •
h•Q( I)– = â Q( ) ` Œ
˜ = â Q( ) ˜ N
`ž ŸŒ
√2Ù 5r √2Ù 5r

1
⇒ h•Q( I)– = hž Ÿ. A N

9.12.3 SHIFTING PROPERTY:

The Fourier transform equation is given by

1 r
h( ) = â Q(I) `•=
NI.
√2Ù 5r

Then,

h•Q(I − )– = `•˜
h( ).

Proof: Given

1 r
h( ) = â Q(I) `•=
NI.
√2Ù 5r

1
Then,
r
h•Q(I − )– = â Q(I − ) `•=
NI
√2Ù 5r

A¢ (I − ) = ¢ ⇒ I = ¢ + 4N NI = N¢.

We have

243
MSCPH501

1 r
1 r
h•Q(I − )– = â Q(¢) `•(q ˜)
N¢ = `•˜
â Q(¢) `•q

√2Ù 5r √2Ù 5r

⇒ h•Q(I − )– = `•˜
h( ). A N

Following are few more properties of Fourier transform which can be proved as same manner as
above properties. Students are advice to proof these properties by themselves.

Assessment Question (SAQ) 6: Prove following properties of Fourier Transform

1. hå `˜=
Q(I)æ = h( + ).
2. h•Q(I) cos I– = jh( + ) + h( − )l.
6
/

3. h•I © Q(I)– = (− )© h( ).
•g
•• g
4. h•Q′(I)– = h( ).
5. h•Q (I)– = (− )© h( ).
©

6. hå´˜ Q (I) NIæ = .


= À(•)
`•

9.13 FOURIER TRANSFORM OF DERIVATIVES


As we know from the properties of Fourier Transform

h•Q © (I)– = (− )© h( ).

I. h ž•= & Ÿ = (−
œ& •
™ „Úℎ
)/ h•Q(I)– = − / Q. Q™ h ¥ 4 Q q Q Q…

II. %Q hÛ 4N h• 4 4N 4 h 4Q q Q(I) ℎ 4
2
hÛ •Q @ (I)– = −ä Q(0) + h• ( ).
Ù
Proof: From cosine Fourier transform we know that

2 r 2 r
hÛ jQ′(I)l = ä â Q′(I) cos I NI = ä â cos I N•Q(I)–.
Ù g Ù g

Now integrating by parts, we get

2 2 r
= ä jcos I Q(I)lr − ä Ê− â 4 I Q(I) NIË
Ù g
Ù g

244
MSCPH501

2 2 r
= ä j0 − Q(0)l + ä Êâ 4 I Q(I) NIË. • ¢q qt Q(I) → 0 I → ∞–
Ù Ù g

Hence,

2 2 r
hÛ •Q @ (I)– = −ä Q(0) + h• ( ) Úℎ ä Êâ 4 I Q(I) NIË = h• ( ).
Ù Ù g

Proved

Similarly we can prove the following relations

III. h• •Q @ (I)– = − hÛ ( ).

hÛ •Q @@ (I)– = −%Û Q @(g) − hÛ ( ).


/ /
IV.

h• •Q @@ (I)– = %Û Q(0) − h• ( ).
/ /
V.

9.14 FOURIER TRANSFORM OF PARTIAL DERIVATIVE OF A


FUNCTION
The Fourier transform of the partial derivatives is given by

› /¢
hÊ Ë=− /
h(¢).
›/I

Where h(¢) is the Fourier transform of ¢.

The Fourier sine transform of the partial derivatives is given by

› /¢
h• Ê Ë = (¢)=Åg − /
h• (¢).
›/I

Where h• (¢) is the Fourier sine transform of ¢

The Fourier cosine transform of the partial derivatives is given by

› /¢ ›¢
hÛ Ê / Ë = − æ ç − /
hÛ (¢).
› I ›I =Åg

Where hÛ (¢) is the Fourier cosine transform of ¢.

245
MSCPH501

Note: From the above formula, it is clear that if • |Œ l = • is given then we apply sine
|Œ l = • is given the, we apply Fourier cosine transform.
j•

Fourier transform and if

9.15 APPLICATION TO SIMPLE HEAT TRANSFER EQUATIONS

One of the important applications of Fourier transforms is to solve the simple heat
transfer equations. Example of this is given below.

Example 7: Solve the equation

=
œq œ& q
œŒ œ& =
.

Subject to the conditions

1 Q 0<I<1
(i) u = 0 when x = 0. t > 0 (ii) ¢ = Úℎ 4 = 0=
0 Q I≥1

(iii) ¢(I , ) “ ¢4N N.

Solution: In view of the initial conditions we know that if ¢ I = 0 is given then we apply
I = 0 is given the, we apply Fourier cosine transform.
œq
œŒ
sine Fourier transform and if

So here we apply Fourier sine transform in given equation

›¢ › /¢
= / .
› › I
We get

2 r ›¢ 2 r › /¢
ä â sin I NI = ä â / sin I NI
Ù g › Ù g › I

› r › ¢
r /
â ¢ sin I NI = â / sin I NI .
› g g › I

Now using the Fourier sine transform partial derivative given below

› /¢
h• Ê Ë = (¢)=Åg − /
h• (¢).
›/I

Where h• (¢) is the Fourier sine transform of ¢

We will get

246
MSCPH501

›h• (¢)
= (¢)=Åg − /
h• (¢) • t 4 ¢ = 0 Úℎ 4 I = 0–

›h• (¢) ›h• (¢)
=− /
h• (¢) ⇒ + /
h• (¢) = 0 ⇒ (à + / )h (¢)
= 0.
› › •

Now the auxiliary equation (A.E) is

q+ /
= 0 ⇒ q = − /.

Hence, the solution is given by

h• (¢) = 5•& Œ
= h• (¢, )
r
h• (¢, ) = â ¢(I, ) sin I NI.
g

Now using the given condition

1 Q 0<I<1
¢= Úℎ 4 = 0=
0 Q I≥1

We get,
6 6
− cos I 6 1 − cos
h• (¢, 0) = â ¢(I, 0) sin I NI = â 1. sin I NI = º » = .
g g g

But we have the solution

h• (¢, ) = 5•& Œ
⇒ h• (¢, 0) = .

Hence, from above equation we get

1 − cos
= .

Hence solution is

1 − cos
h• (¢, ) = 5•& Œ
.

And, finally the complete solution of given equation using inverse Fourier sine transform

2 r 1 − cos
¢= â 5•& Œ
N 4 .
Ù g

247
MSCPH501

Solution of partial differential Equation by Fourier Transform

= -/ , − ∞ < I < ∞, ≥0
œ& q œ& q
œŒ & œ= &
Example 8: Solve

With conditions u(x,0)=f(x),

(I, 0) = t(I) and assuming u, ¢ 4N →0 I → ±∞.


œq œq
œŒ œŒ

Solution: We have given

› /¢ › /¢
= - /
.
› / ›I /
Taking Fourier transform on both sides of the differential equations,

1 › /¢
r
1 › /¢ r
â `•=
NI = â - / `•=
NI.
√2Ù 5r › / √2Ù 5r ›I /

Now using the Fourier transform of derivative

› /Q
~h ¼ / ½ = (− )/ h•Q(I)– = − / h•Q(I)–¿.
NI

We have

› / h(¢)
= −- / / h(¢)
N /

› / h(¢)
+ - / / h(¢) = 0 ⇒ à/ h(¢) + - / / h(¢) = 0.
N /
Now Auxiliary equation corresponding to this equation is

q/ + - / /
=0⇒q=±- .

Hence, solution corresponding this equation is

h( , )= `h•Œ
+) 5`h•Œ
… … .1

Using given condition

¢(I, 0) = Q(I) 4N (I, 0) = t(I)


œq
œŒ

h( , 0) = h( ) and ( , 0) = ç( ) on taking transform.


•À(q)
•Œ

248
MSCPH501

Using these conditions in above equation, we get

h( , 0) = + ) = h( ) … … … … … … 2

( , 0) = - ( − )) = ç( ) … … … .3
•À(q)
•Œ

Solving (2) and (3), we get

= ºh( ) + »
6 è(•)
/ `h•

ºh( ) − ».
6 è(•)
/ `h•
B=

Using the value of A and B in (1), we get

h( , ) = ºh( ) + » + / ºh( ) − » .
6 è(•) `h•Œ 6 è(•) 5`h•Œ
/ `h• `h•
…..(4)

By inversion theorem, (4) reduces to

1 1 =5hŒ 1 1 = hŒ
u (x, t) = ÊQ(I − - ) − â t(,)N,Ë + ÊQ(I + - ) + â t(,)N,Ë.
2 - h 2 - h

Using the result

hY´h Q( )N Z = (5`•) .
= À(•)

Self Assessment Question (SAQ) 7: Solve

›¢ › /¢
= . Q ≤ I < ∞ 4N > 0
› › /I
The given conditions are

() ¢(I, 0) = 0, Q I≥0 ( ) (0, ) = − ( 4 4 )


œq
œŒ

( ) ¢(I, ) “ ¢4N N.

9.16 SUMMARY
The main aim of to study the Fourier transforms is the solution of partial differential
equations and systems of such equations. Firstly we have learned about brief review of Fourier
series, which very important to learn the Fourier transform. After that Fourier Integral along with
Fourier sine Integral and Fourier cosine Integral have been explained. The knowledge of Fourier
Integral is very important for Fourier transform. After this we have explained Fourier transform
and their important properties in detailed. We also have explained Fourier sine transform and

249
MSCPH501

Fourier cosine transform, which is equally important to Fourier transform. To understand this
unit more clearly various solved examples are includes almost in each section. For students
assessment self assessment question is also incorporated throughout the chapter.
9.17 GLOSSARY
Periodic Function: The function which repeat itself after a fix time.
Computation: Calculations such as addition, subtraction etc.

9.18 TERMINAL QUESTIONS

1. Find the Fourier Transform of Q(I) if


I, |I| ≤ =
Q(I) =
0, |I| >

− |I| Q |I| < =


2. Show that the Fourier Transform of
Q(I) =
0 Q |I| > > 0
Is %Û ž Ÿ.
/ 65XY ˜•
•&
r •`©Œ /
Hence show that ´g ž Ÿ N = .
Û
Œ /

√2Ù
3. Show that the Fourier Transform of

Q(I) = Ô 2 Q |I| ≤ =
0 Q |I| >
Is •˜ .
Ø" •˜

5 ˜=
4. Find the Fourier cosine Transform of .

I/, |I| < =


5. Find Fourier transform of
h(I) =
0, |I| >

1
6. Find Fourier Sine Transform of
Q(I) = .
+ /)I(I /
7. Find the Fourier Sine and Cosine Transform of 5h=
+“ 5é=
. -, ä > 0

.

8. Find f(x) if its Fourier Sine transform is 6 •&
9. Find f(x) if its Fourier Sine Transform is (2Ù )& .
Á

= œ& = Q I ≥ 0 4N ≥ 0 under the given condition ¢ = ¢g I=


œq œ& q
œŒ
0, 4N > 0 with initial condition ¢(I, 0) = 0, I ≥ 0.
10. Solve

250
MSCPH501

9.19 ANSWERS
Self Assessment Question (SAQ)

1. Q(I) = − Û ´g ž Ÿ cos ¢I N¢.


/ r 6 XY q=
q& 56

3. Q(I) = % ž6 Ÿ.
/ •
Û •&

4. Q(I) = % .
/ Ø" •
Û •

5. Q(I) = .
/ Ø" •(65XY •)
•&
&
7. Q(I) = . ´g cos IN .
/ r 65d êë ì
Û •&

Terminal Questions:

.
6 /`
√/Û •&
1.

4. hÛ •Q(I)– = % ž Ÿ.
/ ˜
Û ˜& •&

5. ž − & Ÿ sin + cos .


/˜& c c˜
• • •&
(1 −
Û 5˜=
/˜&
6. ).
+ •& , + •& .
˜• ›• ˜h ›é
•& h& é& •& h& é&
7.

.
/•`©& ˜=
Û&= &
8.
6
= √=
9. .
&
10. ¢(I, ) = ´g ¦ § 4 IN .
/qÝ r 65d êë ì
Û •

9.20 REFERENCES
George Arfken, H. A. Weber,: Mathematical Methods For Physicists.
H.K. Dass,: Mathematical Physics, S. Chand Publication.
Erwin Kreyszig,: Advanced Engineering Mathematics, Wiley Plus Publication.
B.S. Rajput,: Mathematical Physics, Pragati Prakashan.
Satya Prakash,: Mathematical Physics, Pragati Prakashan.

9.21 SUGGESTED READINGS

251
MSCPH501

George Arfken, H. A. Weber,: Mathematical Methods For Physicists.


H.K. Dass,: Mathematical Physics, S. Chand Publication.

252
MSCPH501

________________________________________________________
UNIT 10: LAPLACE TRANSFORM
Structure
10.1 Objectives
10.2 Introduction
10.3 Laplace Transform
10.4 Linearity of the Laplace Transform
10.5 Change of Scale Property
10.6 First Shifting Theorem
10.7 Second Shifting Theorem (Heaviside’s Shifting Theorem):
10.8 Laplace Transform of the Derivative of Q( )
10.9 Laplace Transform of the Derivative of Order N
10.10 Laplace Transform of the Integral of Q( )
10.11 Laplace Transform of Some Important Functions

Q( )
6
Œ
10.12 Laplace Transform of

10.13 Laplace Transform of . Q( )


10.14 Unit Step Function
10.15 Laplace Transform of Unit Step Function
10.16 Periodic Functions:
10.17 Some Important Formulae of Laplace Transform
10.18 Inverse Laplace Transform
10.19 Some Important Formulae of Inverse Laplace Transform
10.20 Multiplication by s
6

10.21 Division by s (Multiplication By )

10.22 First Shifting Property


10.23 Second Shifting Property
10.24 Inverse Laplace Transforms of Derivatives:
10.25 Inverse Laplace Transform of Integrals

253
MSCPH501

10.26 Inverse Laplace Transform by Partial Fraction Method


10.27. Solution of Differential Equations by Laplace Transforms
10.28. Summary
10.29. Glossary
10.30. Terminal Questions
10.31. Answers
10.32. References
10.33. Suggested Readings

254
MSCPH501

10.1 OBJECTIVES

After studying this chapter we will learn about how Laplace transforms is useful for
solving differential equations with boundary values without finding the general solution. With
the use of different properties of Laplace transform and Inverse Laplace transform one can solve
many important problem of physics with very simple way. Thus we will learn from this unit to
use the Laplace transform for solving the differential equations.

10.2 INTRODUCTION

The Laplace transform is named for the French mathematician Laplace, who studied this
transform in 1782. Laplace transforms is an integral transform. It helps in solving the differential
equations with boundary values without finding the general solution and values of the arbitrary
constants. The method of Laplace transforms is a system that relies on algebra (rather than
calculus-based methods) to solve linear differential equations. While it might seem to be a
somewhat cumbersome method at times, it is a very powerful tool that enables us to readily deal
with linear differential equations with discontinuous forcing functions.

Laplace transforms are invaluable for any engineer’s mathematical toolbox as they make solving
linear differential equations and related initial value problems, as well as systems of linear
differential equations, much easier. Applications abound: electrical networks, springs, mixing
problems, signal processing, and other areas of engineering and physics. The process of solving
differential equations using the Laplace transform method consists of three steps:

Step 1. The given differential equations is transformed into an algebraic equation, called the
subsidiary equation.

Step 2. The subsidiary equation is solved by purely algebraic manipulations.

Step 3. The solution in Step 2 is transformed back, resulting in the solution of the given problem.

10.3 LAPLACE TRANSFORM

Definition: The Laplace transform of a function f(t) is defined as follows

255
MSCPH501

r
F (s) = â e5 Q( )N .
g

For all positive values of t and integral should exits. The Laplace transform is denoted by

r
Îjk(Œ)l = Ä (¡) = â •5¡Œ k(Œ)~Œ.

The Laplace transform is an operation that transforms a function of t (i.e., a function of time
domain), defined on [0, ∞), to a function of s (i.e., of frequency domain). F(s) is the Laplace
transform, or simply transform, of f (t). Together the two functions f (t) and F(s) are called a
Laplace transform pair.

10.4 LINEARITY OF THE LAPLACE TRANSFORM


The Laplace transform is a linear operation; that is, for any functions Q( ) and t( )whose
transforms exist and any constants and “ the transform of Q( ) + “t( ) exists, and

Îj|k(Œ) + ˜¼(Œ)l = |Îjk(Œ)l + ˜j¼(Œ)l.

r
Proof:

ƒj Q( ) + “t( )l = â 5•Œ j
Q( ) + “t( )lN .
g

As we know that integration is a linear operation. So we can use the linearity property of
integration in above equation
r r
ƒj Q( ) + “t( )l = â 5•Œ
Q( )N + “ â 5•Œ
t( )N
g g

ƒj Q( ) + “t( )l = ƒj Q( )l + “ƒjt( )l.

Proved

10.5 CHANGE OF SCALE PROPERTY


If the Laplace transform of Q( ) is h( ) then
D ¡
Îjk(|Œ)l = Ä ž Ÿ.
| |
r
Proof: From the definition of Laplace transform
ƒjQ( )l = â 5•Œ
Q( )N
g
N
O¢ = ⟹N = 4N r =

256
MSCPH501

r •• N 1 r
⇒ ƒjQ( )l = â 5
˜ Q( ) = â 5Ì•
Q( )N ºÚℎ Ã= »
g g

1 1
= h(Ã) = hž Ÿ. A N

10.6 FIRST SHIFTING THEOREM:


If h( ) has the Laplace transform of Q( ) then
Îj•|Œ k(Œ)l = Ä(¡ − |).
Proof: Using the definition of Laplace transform
r
h( − ) = â 5(•5˜)Œ
Q( )N
g
r r
=â 5•Œ ˜Œ
Q( )N = â 5•Œ ˜Œ
Q( )N
g g
5•Œ • ˜Œ
= ´g Q( )–N = ƒj Q( )l.
r ˜Œ
Proved

Alternative Method:
r r
ƒj ˜Œ
Q( )l = â ˜Œ 5•Œ
Q( )N = â 5•Œ ˜Œ
Q( )N
g g
r r
=â 5(•5˜)Œ
Q( )N = â 5qŒ
Q( )N
g g

Using ( − ) = ¢

= h(¢) = h( − ). Proved

10.7 SECOND SHIFTING THEOREM (HEAVISIDE’S SHIFTING


THEOREM):

Q( − ), Q >
If ƒjQ( )l = h( ) and t( ) = .
0, Q 0< <

Then, ƒjt( )l = 5˜•


h( ).

Proof: As per the definition of Laplace transform


r
ƒjt( )l = â 5•Œ
t( )N
g

257
MSCPH501

˜ r
ƒjt( )l = â 5•Œ
t( )N + â 5•Œ
t( )N .
g ˜

Using the given condition t( ) = 0 Q 0< < 4N t( ) = Q( − ) Q >


r
ƒjt( )l = 0 + â 5•Œ
Q( − )N .
˜

S Ú ¢ 4t ( − ) = ⇒N =N 4N = ( + ) we get
r r
ƒjt( )l = â 5•(• ˜)
Q( )N = 5•˜
â 5••
Q( )N = 5•˜
h( ).
g g

Hence, ƒjt( )l = 5˜•


h( ) A N

10.8 LAPLACE TRANSFORM OF THE DERIVATIVE OF k(Œ)


If ƒjQ( )l = h( ) and Q @ ( ) is the derivative of Q @ ( ) then

Îjk@ (Œ)l = ¡Îjk(Œ)l − k(•).

Proof: As we know

ƒjQ @ ( )l = ´˜ Q N .
r 5•Œ @(Œ)

Solving above equation using integration by parts we get


r
ƒjQ ( )l = j
@ 5•Œ
Q( )lr
g − â (− 5•Œ )Q( )N .
g

As we know that
5r
= 0 4N g
=1⇒ 5•Œ
Q( ) = 0 Úℎ 4 = ∞ 4N 5•Œ
Q( ) = Q(0)Úℎ 4 = 0
⇒ ƒjQ @ ( )l = −Q(0) + ´g Q( )N = −Q(0) + ƒjQ( )
r 5•Œ

⇒ Îjk@ (Œ)l = ¡Îjk(Œ)l − k(•). „C»Ž•~

10.9 LAPLACE TRANSFORM OF THE DERIVATIVE OF ORDER n


ƒjQ © ( )l = ©
ƒjQ( )l − ©56
Q(0) − ©5/
Q′(0) − ©5_
Q′′(0) − ⋯ − Q ©56 (0).

ƒjQ @ ( )l = ƒjQ( )l − Q(0) ………………. 1


Proof: As we know that the Laplace transform of derivative is given by

258
MSCPH501

Using this equation we can find the Laplace transform of jQ @@ ( )l

ƒjQ @@ ( )l = ƒjQ′( )l − Q′(0).

Using equation 1 we get

ƒjQ @@ ( )l = • ƒjQ( )l − Q(0)– − Q′(0)

ƒjQ @@ ( )l = /
ƒjQ( )l − Q(0) − Q′(0) … … … … .2

Similarly we can find the value of ƒjQ @@ ′( )l by using equation 1 &2

ƒjQ @@@ ( )l = _
ƒjQ( )l − /
Q(0) − Q′(0) − Q @@ (0) … … … … .3

Similarly, using above method we get

ƒjQ © ( )l = ©
ƒjQ( )l − ©56
Q(0) − ©5/
Q′(0) − ©5_
Q′′(0) − ⋯ − Q ©56 (0).

Proved

10.10 LAPLACE TRANSFORM OF THE INTEGRAL OF k(Œ)


If ƒjQ( )l = h( ) and Q @ ( ) is the derivative of Q @ ( ) then
Œ
D
Î Êâ k(Œ)~ŒË = Ä(¡).
• ¡

Proof: Let t( ) = ´g Q( ) 4N t(0) = 0 then t@ ( ) = Q( ).


Œ

As we know that ƒjt′( )l = ƒjt( )l − t(0)

⇒ ƒjt′( )l = ƒjt( )l t(0) = 0

1
⇒ ƒjt( )l = ƒjt′( )l.

Using the value of t( ) = ´g Q( ) 4N t′( ) = Q( ) we will get


Œ

Œ
1
⇒ ƒ Êâ Q( )N Ë = ƒjQ( )l
g

Œ
D
⇒ Î Êâ k(Œ)~ŒË = Ä(¡). „C»Ž•~
• ¡

10.11 LAPLACE TRANSFORM OF SOME IMPORTANT FUNCTIONS

259
MSCPH501

Î(D) = .
D
Ù
1.

Proof: From the definition of Laplace transform, the Laplace transform of L(1) can be
written as
r
ƒ(1) = ´g 1. N =º » = − j0 − 1l = .
r 5•Œ d Êëì 6 6
5• g • •

j 5r
= 0 4N g
= 1l

Î(•|Œ ) = . Úℎ >
D
¡5|
2.

Proof: As per the definition of Laplace transform


r r
ƒ( ˜Œ
)=â ˜Œ
. 5•Œ
N =â (˜5•)Œ
N
g g

r 5(•5˜)Œ r
1
=â 5(•5˜)Œ
N = Ê Ë = . j 5r
= 0 4N g
= 1l
g −( − ) g −

Î(.ˆ- |Œ) = .
|
¡E |E
3.

Proof:

ƒ(sin ) = ƒ( ) = /` „ƒY Z − ƒY Z….


d íÜì 5 d ÊíÜì 6 `˜Œ 5`˜Œ
/`

1
Since sin , = „ `Ü
− 5`Ü

2

î 4t ƒ Or 4 Q q ƒ( ) = •5˜ Ú Ú rr t
˜Œ 6

1 1 1 1 ( + )−( − )
ƒ(sin )= æ − ç= Ê Ë
2 − + 2 ( − )( + )

1 2
= æ / ç= .
2 + / / + /

Î(ÌÍ. |Œ) =
¡
¡E |E
4. .

ƒ(cos ) = ƒ( ) = / „ƒY Z + ƒY Z….


d íÜì d ÊíÜì 6 `˜Œ 5`˜Œ
/
Proof:

1
Since cos , = „ `Ü
+ 5`Ü

2

260
MSCPH501

î 4t ƒ Or 4 Q q ƒ( )= Ú Ú rr t
˜Œ 6
•5˜

1 1 1 1 ( + )+( − )
ƒ(cos )= æ + ç= Ê Ë
2 − + 2 ( − )( + )

1 2
= æ ç= . j Ú 4 Ú /
= −1l
2 / + / / + /

Proved

Î(.ˆ-ï |Œ) =
|
¡E 5 | E
5. .

Proof:

ƒ(sinh ) = ƒ( ) = / jƒ( ˜Œ )
− ƒ( 5˜Œ )l.
d Üì 5 d ÊÜì 6
/`

1
Since sinh , = „ Ü
− 5Ü

2

î 4t ƒ Or 4 Q q ƒ( ) = •5˜ Ú Ú rr t
˜Œ 6

1 1 1 1 ( + )−( − )
ƒ(sin )= æ − ç= Ê Ë
2 − + 2 ( − )( + )

1 2
= æ ç= . A N
2 / − / / − /

Î(ÌÍ.ï |Œ) = .
¡
¡E 5 | E
6.

Proof:

ƒ(cosh ) = ƒ( ) = / jƒ( ˜Œ )
+ ƒ( 5˜Œ )l.
d Üì d ÊÜì 6
/

1
Since cos , = „ Ü
+ 5Ü

2

î 4t ƒ Or 4 Q q ƒ( ) = •5˜ Ú Ú rr t
˜Œ 6

1 1 1 1 ( + )+( − )
ƒ(cosh )= æ + ç= Ê Ë
2 − + 2 ( − )( + )

1 2
= æ ç= .
2 / − / / − /

261
MSCPH501

Proved

Î(Œ} ) = ñ’•C• } |}~ ¡ |C• Š»¡‹Œ‹Ž•


}!
¡}ðD
7.

Proof: Î(Œ} ) = ´g . N
r © 5•Œ

¢ N¢
4 Ú ¢ 4t =¢⇒ = ⇒N = .

We will get

©)
r
¢© N¢ 1 r
ƒ( =â ©
. 5q
= © 6
â 5q
¢© N¢
g g

r
Ú 4 Ú ℎ â 5q
¢© N¢ = Γ(4 + 1) = 4!.
g

4!
Hence we have
ƒ( © ) = © 6
. A N

Example 1: Find the Laplace transform of 4_ 2 .

Solution: we have given Q( ) = 4_ 2

And, we also know that sin 3, = 3 sin , − 4 4_ ,.

4_ 2 = j3 sin 2 − sin 6 l.
6
c
From above equation

ƒj 4_ 2 l = j3 L(sin 2 ) − L(sin 6 )l
6
c
Hence,

1 6 6
= æ − ç.
4 / +4 / + 36

Ú 4 Ú ℎ ƒ(sin )= / + /

6 /
+ 36 − / − 4 48
= Ê Ë= .
4 ( / + 4)( / + 36) ( / + 4)( / + 36)

Ans

Example 2: Find the Laplace transform of 42 43

Solution: we have given Q( ) = 42 43

262
MSCPH501

1
î 4t r 4 → 4 4) = jcos( − )) − cos( + ))l
2
1
≫ 42 43 = jcos − cos 5 l.
2

ƒ( 4 2 43 ) = jƒ(cos ) − L(cos 5 )l.


6
/
So

ƒ(cos )=

•& ˜&
Now using relation .

ƒ( 4 2 43 ) = º − »= .
6 • • 6/•
/ •& 6 •& /š (• & 6)(• & /š)
We have

Ans

ƒž Ÿ= . ç 4 ℎ ƒ ¼2% ½ =
6 6 Œ 6
Â
√ÛŒ √• Û • ó&
Example 3: Show that

Solution: Suppose h( ) = ¼2% ½ ℎ 4 h @ ( ) = 4N r Ú 4 ℎ h(0) = 0


Œ 6
Û √ÛŒ

Now we know that ƒjh @ ( )l = ƒjh( )l − h(0).

ƒž Ÿ = ƒ ¼2%Û½ − 0 = . −0
6 Œ 6
Â
√ÛŒ • ó&
Hence

1 1
⇒ ƒ¦ §= . o 4 A N
√Ù √

Example 4: Find the Laplace transform of + /


+ _
.

Solution: we have given Q( ) = + /


+ _

Now using the relation ƒ( © ) = •gðÁ


©!

We haveƒj Q( )l = ƒ( ) + ƒ( / ) + ƒ( _ ) = •& + •Â + •î 4 .
6 / a

Example 5: Find the Laplace transform of ℎ

Solution: we have given Q( ) = ℎ

We know that ƒ( ℎ )= .

•& 5˜&

ƒj © Q( )l = (−1)© ••g jh( )l.


•g
Now using the relation

263
MSCPH501

ƒ( ℎ )=− ž Ÿ=− =− = . 4 .
• • (•& 5˜& ).65•./• (•& 5˜& 5/•& (•& ˜& )
•• •& 5˜& (•& 5˜& )& (•& 5˜& )& (•& 5˜& )&
We will get

Self Assessment Question (SAQ) 1: Find the Laplace transform of 2 4 2 cos 4 .

.
Á
Self Assessment Question (SAQ) 2: Find the Laplace transform of &

1, 0≤ <1
Self Assessment Question (SAQ) 3: Find the Laplace transform of h( ) = ~ , 1 ≤ < 2=
/
, 2≤ <∞

Self Assessment Question (SAQ) 4: Find the Laplace transform of 1 + 42 .

Self Assessment Question (SAQ) 5: Find the Laplace transform of 4ℎ_ .

k(Œ)
D
Œ
10.12 LAPLACE TRANSFORM OF

If ƒjQ( )l = h( ) then If ƒ º Œ Q( )» = ´• Q( ) N .
6 r

Proof: As per the Laplace transform

ƒjQ( )l = h ( ) = ´g Q( )N .
r 5•Œ

Integrating with respect to s, we get


r r r
â h( )N = â Êâ 5•Œ
Q( )N Ë N
• • g

r r r 5•Œ r
= â Q( ) Êâ 5•Œ
N Ë N = â Q( ) Ê Ë N
g • g − •
r
Q( ) r
Q( )
= −â j 5•Œ lr
g N = −â j 5r
− 5•Œ lN
g g
r
Q( ) r
1 1
= −â j0 − 5•Œ lN
=â 5•Œ
æ Q( )ç N = ƒ æ Q( )ç
g g

D r
⇒ Î æ k(Œ)ç = â Ä(¡)~¡. „C»Ž•~
Œ ¡

10.13 LAPLACE TRANSFORM OF Œ. k(Œ)


~}
ÎjŒ} . k(Œ)l = (−D)} Ä(¡).
~¡}
Ø" /Œ
Œ
Example 6: Find the Laplace transform of .

264
MSCPH501

Solution: ƒ(sin 2 ) =
/
•& c

• r
ƒž Ÿ = ´• N = 2. ºtan56 » = ºtan56 ∞ − tan56 » = − tan56
Ø" /Œ r / 6 • Û •
Œ •& c / / • / / /

=cot 56 / .

Ans

Example 7: Find the Laplace transform of the function

Q( ) = 5Œ
sin 2 .

Solution: ƒjsin 2 l =
/
•& c

ƒj sin 2 l = (• = h( ) J
5Œ /
6)& c

ƒj sin 2 l = −h @ ( ) = •• º(• » = j(• = j(• .


5Œ • / /./(• 6) c(• 6)
6)& c 6)& cl& 6)& cl&
Ans.

10.14 UNIT STEP FUNCTION


The unit step function is defined as follows:
•, ñ’•} Œ < =
•(Œ − |) = ñ’•C• | ≥ •.
D, ñ’•} Œ ≥ |
10.15 LAPLACE TRANSFORM OF UNIT STEP FUNCTION
•5|¡
Îj•(Œ − |)l = .
¡
Proof: Using the definition of Laplace transform, we have
r
Îj•(Œ − |)l = â •5¡Œ •(Œ − |)~Œ.

Now using the condition of unit step function


˜ r
ƒj¢( − )l = â 5•Œ
¢( − )N + â 5•Œ
¢( − )N
g ˜

˜ r 5•Œ r
=â 5•Œ
0. N + â 5•Œ
1. N = 0 + Ê Ë
g ˜ − ˜
5˜•
ƒj¢( − )l = . A N

Example 8: Convert the following function in terms of unit step function and then find the
Laplace Transform

265
MSCPH501

6, Úℎ 4 < 2 =
Q( ) =
4, Úℎ 4 ≥ 2
Solution: Given that
6, Úℎ 4 < 2 =
Q( ) =
4, Úℎ 4 ≥ 2
This further can be written as
6 + 0, Úℎ 4 < 2 = 0, Úℎ 4 < 2 =
Q( ) = =6+
6 − 2, Úℎ 4 ≥ 2 −2, Úℎ 4 ≥ 2
0, Úℎ 4 < 2 =
= 6 + (−2) = 6 − 2¢( − 2)
1, Úℎ 4 ≥ 2
[Using the condition of unit step function]

6 5/•
ƒjQ( )l = 6ƒ(1) − 2ƒj¢( − 2)l = −2 . 4

10.16 PERIODIC FUNCTIONS:


If Q( ) be a periodic function with period ¥, ⇒ Q( + ¥) = Q( ) then

´• •5¡Œ k(Œ)~Œ
ô

Îjk(Œ)l = .
D − •5¡ô
Proof: As we know
r
ƒjQ( )l = â 5•Œ
Q( )N .
g

This can be written as in the following manner

ƒjQ( )l = ´g Q( )N = ´g Q( )N + ´ Q( )N + ´/ Q( )N + ⋯
r 5•Œ 5•Œ / 5•Œ _ 5•Œ

Now substituting = ¢ + ¥, = ¢ + 2¥, … 4N N = N¢ in second integral, third integral, and


so on respectively, we will get

ƒjQ( )l = ´g 5•Œ
Q( )N + ´g 5•(q )
Q(¢ + ¥)N¢ + ´g 5•(q / )
Q(¢ + 2¥)N¢ + ⋯

=â 5•Œ
Q( )N + 5•
â 5•q
Q(¢ + ¥)N¢ + 5/•
â 5•q
Q(¢ + 2¥)N¢ + ⋯
g g g

=â 5•Œ
Q( )N + 5•
â 5•q
Q(¢)N¢ + 5/•
â 5•q
Q(¢)N¢ + ⋯
g g g

As Q(¢) be a periodic function with period ¥, ⇒ Q(¢ + ¥) = Q(¢ + 2¥) = ⋯ = Q(¢).

266
MSCPH501

Now we can write

=â 5•Œ
Q( )N + 5•
â 5•Œ
Q( )N¢ + 5/•
â 5•Œ
Q(¢)N + ⋯
g g g

=â 5•Œ
Q( )N j1 + 5•
+ 5/•
+ ⋯ l.
g

S Ú ¢ 4t ℎ 4N 4 j1 + I + I / + I _ + … = 65= we have
6

´g 5•Œ
Q( )N
ƒjQ( )l = . A N
1− 5•

Example 8: Find the Laplace transform of the waveform

Q( ) = ž _ Ÿ , 0 ≤ ≤ 3.

ƒjQ( )l = 65d Êëõ ´g Q( )N


6 5•
Solution:
_
ƒ º _ » = 65d ÊÂë ´g ž_ Ÿ N = 65d ÊÂë _ º − (1) »
/Œ 6 _ 5•Œ / 6 / Œd Êëì d Êëì
5• •& g

º − + •& » = _ º + »
/ 6 _d ÊÂë d ÊÂë 6 / 6 _d ÊÂë 65d ÊÂë
=_ 65d ÊÂë 5• •& 65d ÊÂë 5• •&

=5•(65d ÊÂë ) + _•& .


/d ÊÂë /
Ans.

Self Assessment Question (SAQ) 6: Find the Laplace transform of .

4/ .
6
Œ
Self Assessment Question (SAQ) 7: Find the Laplace transform of

Self Assessment Question (SAQ) 8: Find the Laplace transform of

− 1, 1< <2=
Q( ) = U
0 r Ú ℎ
Self Assessment Question (SAQ) 9: Find the Laplace transform of the periodic function

Q( ) = Œ
Q 0 < < 2Ù.

10.17 SOME IMPORTANT FORMULAE OF LAPLACE TRANSFORM

267
MSCPH501

S.No. Q( ) h( )

˜Œ 1

1

sin / + /
2

cos / + /
3

sinh / − /
4

cosh / − /
5

© 4!
© 6
6

7 eö sin ( − “)/ + /

−“
eö cos
( − “)/ + /
8

sin ( / )/
2 +
/
9
/
− /
cos
( +
/ / )/
10

10.18 INVERSE LAPLACE TRANSFORM


If h( ) is the Laplace Transform of a function Q( ), then Q( ) is known as Inverse Laplace
Transform.

Q( ) = ƒ56 jh( )l.

The Inverse Laplace Transform is very useful to solving the differential equations without
finding the general solution and arbitrary constants.

10.18 SOME IMPORTANT FORMULAE OF INVERSE LAPLACE TRANSFORM

S.No. h( ) Q( ) = ƒ56 jh( )l


1 ˜Œ

1

sin
/ + /
2

cos
/ + /
3

268
MSCPH501

sinh
/ − /
4

cosh
/ − /
5
4! ©
© 6
6

7
( − “)/ + / eö sin
−“
eö cos
( − “)/ + /
8

( / )/ sin
+
/
2
9
/
− /
cos
( +
/ / )/
10
1 1
11

= ƒº ».
6 6
• Á/& √ÛŒ
Example 9: Prove that

= æ(4−1)!ç = æ ç.
1 4−1 4−1
Solution: we know that ƒ56 º 4 »
Γ4
1 1

= = Ã4 Γ 2 = √π
−1 −
1 1
Using above relation we can write ƒ º 1/2 »
56 2 2

Γ2
1 √π

1 1 1 1
⇒ ƒ56 Ê Ë=Ê Ë ⇒Ê Ë = ƒÊ Ë. A N
1/2
√πt 1/2
√πt

Example 10: Find the inverse Laplace Transform of the following:


6 6 •
•5_ •& 5/š •& 6a
(i) (ii) (iii)

.
6 6 •56
(iv)
•& —
(v) (•5/)& 6
(vi) (•56)& c

Solution.

ƒ56 •5_ = . j 4 ƒ56 •5˜ = l


6 _Œ 6 ˜Œ
(i)

ƒ56 •& 5/š = ƒ56 š U•& 5(š)& V = 4ℎ5 . j 4 ƒ56 = 4ℎ l


6 6 š 6
(ii)
š 2− 2

ƒ56 •& = ƒ56 •& = cos 4 . j 4 4 ƒ56 = l


• •
(iii)
6a (c)& 2+ 2

269
MSCPH501

1
ƒ56 = ƒ56 = sin 3 . j 4 4 ƒ56 = 4 l
6 6 6 6̃
(iv)
•& — •& (_)& _ 2+ 2

1
ƒ56 (•5/)& = sin . º 4 4 ƒ56 ( = 4 »
6 /Œ ›Œ
6 −“)2 + 2
(v)


ƒ56 (•56)& = ƒ56 (•56)& = cos 2 . æ 4 4 ƒ56 = “ ç
•56 •56 Œ ˜Œ
c (/)& ( − )2 +“2
(vi)

Find ƒ56 .
•& _• ¦
•Â
Example 11:

Solution: Here, we have

ƒ56 = ƒ56 º + + » = 1 + + j 4 ƒ56 = (©56)!l


•& _• ¦ 6 _ ¦ _Œ ¦ / 6 Œ gÊÁ
•Â • •& • 6! /! •g

=1+3 +4 /
. 4

.
6
•5š
Self Assessment Question (SAQ) 10: Find the Inverse Laplace transform of

.
/•5š
—•& 5/š
Self Assessment Question (SAQ) 11: Find the Inverse Laplace transform of

10.20 MULTIPLICATION BY S

Î5D j¡Ä(¡) l = ~Œ k(Œ) + k(•)Ç(Œ).


~

.
¡
¡E ™
Example 12: Find the Inverse Laplace Transform of

Solution: we know that ƒ56 = sin .


6
•& ˜&

ƒ56 •& = sin 2


6
c
Hence

ƒ56 •& = •Œ ( 4 2 ) + sin(0) †( ) = 2 cos 2 . 4 .


• •
c
And

D
¡
10.21 DIVISION BY s (MULTIPLICATION BY )

ƒ56 º » = ´g j ƒ56 |h ( )|l N = ´g Q( )N .


À (•) Œ Œ

Example 13: Find the Inverse Laplace Transform of

.
6 6 •& _
• (• ˜) •(•& 6) •(•& —)
(i) (ii) (iii)

Solution:

270
MSCPH501

4 ƒ56 ž• ˜Ÿ =
6 5˜Œ
(i)

ƒ56 º » = ´g ƒ56 ž ŸN
6 Œ 6
•(• ˜) • ˜

Œ
= ´g N =º » = + = j1 − 5˜Œ l.
4 .
Œ 5˜Œ d ÊÜì d ÊÜì 6̃ 6̃
5˜ g 5˜

Ú 4 Ú ℎ ƒ56 = sin .
6
(•& 6)
(ii)

ƒ56 • ž•& Ÿ = ´g ƒ56 ž•& 6Ÿ = ´g sin N =


6 6 Œ 6 Œ
6)

= j− cos lŒg = j− cos + 1l = j1 − cos l. 4 .

.
•&
•& ˜&
Self Assessment Question (SAQ) 12: Find the Inverse Laplace transform of

.
6
•(•& ˜&)
Self Assessment Question (SAQ) 13: Find the Inverse Laplace transform of

10.22 FIRST SHIFTING PROPERTY

If the inverse Laplace transform of h( ) Q( ) such that

ƒ56 jh( )l = Q( )

Then, ƒ56 h( + ) = 5˜Œ


ƒ56 jh( )l.

Example 14: Find the Inverse Laplace Transform of

.
6 • 6
(• c)î
(i) (ii) •& c• 6_
(iii) —•& a• 6

Solution:

(i) Ú 4 Ú ℎ ƒ56 º•î » =


6 ŒÂ
_!

Then, ƒ56 º(• »= ƒ º• î » ¢ 4t Q ℎQ 4t O O J


6 5cŒ 56 6
c )î

= 5cŒ Œ
=a . 4 .
 6 5cŒ _
_!

ƒ56 ž Ÿ = ƒ56 (•
• • /5/
•& c• 6_ /)& (_)&
Solution (ii)

271
MSCPH501

= ƒ56 − ƒ56 .
• / /
(• /)& (_)& (• /)& (_)&

Using First shifting property ⇒ ƒ56 h( + ) = 5˜Œ


ƒ56 jh( )l

ƒ − ƒ ž & & Ÿ
5/Œ 56 • 5/Œ 56 / _
(•)& (_)& _ • _
=

= cos 3 − sin 3 . 4 .
5/Œ / 5/Œ
_

ƒ56 —•& = ƒ56 (_•


6 6
a• 6 6 )&
Solution (iii)

= ƒ56 = ƒ & j î 4t h ℎ Q 4t O O Jl
6 6 6 5Œ⁄_ 56 6
Á &
— (• ) — •
Â

=— = . 4 .
6 5Œ⁄_ Œd Êì⁄Â

10.23 SECOND SHIFTING PROPERTY

ƒ56 j 5˜•
h ( ) l = Q( − )¢( − ).

Example 15: Obtain Inverse Laplace Transform of

() ( ) .
d Êãë d Êë
(• _ ) ( • 6 )Â

Solution:

(i) As we know that

ƒ56 =
6 5_Œ
• _

Now using second shifting theorem we can find the inverse Laplace transform of

ƒ56 (• = ¢( − Ù). 4 jƒ56 j h ( )l = Q( − )¢( − )l


d Êãë 5_( Œ5Û) 5˜•
_)

Ans.

Ú 4 Ú ℎ ƒ56 •Â = /! .
6 Œ&
(ii)

ƒ56 ( • . j¢ 4t Q ℎQ 4t O O Jl
6 5Œ Œ
&

6 )Â /!
Then

Hence
ƒ56 = ¢( − 1). j¢ 4t 4N ℎ Q 4t O O Jl
d Êë 5( Œ56) ( Œ56 )
&

(• 6 )Â /!

272
MSCPH501

Ans.

10.24 INVERSE LAPLACE TRANSFORMS OF DERIVATIVES:

ƒ56 º •• h ( )» = − ƒ56 jh( )l = − Q( )


⇒ ƒ56 jh( )l = − Œ ƒ56 º •• h ( )» .


6 •

Example 16: Find ƒ56 Ulog ž Ÿ V.


• 6
•56

ƒ56 Ulog ž•56Ÿ V = − Œ ƒ56 º•• log( •56) »


• 6 6 • • 6
Solution:

¢ 4t 4 r Or 4 Q q QN

= − ƒ56 º log( + 1) − log ( − 1)» = − ƒ56 º − »


6 • • 6 6 6
Œ •• •• Œ • 6 •56

=−Œj − Œl
= Œj − 5Œ l.
4 .
6 5Œ 6 Œ

10.25 INVERSE LAPLACE TRANSFORM OF INTEGRALS

ƒ56 „´• h( )N … = = Œ ƒ56 jh( )l


r •(Œ) 6
Œ

⇒ ƒ56 jh( )l = ƒ56 „´• h( )N ….


r

.
/•
(•& 6)&
Example17: Find the Inverse Laplace Transform of

Solution: we have to find ƒ56 ž(•& Ÿ.


/•
6)&

We will solve this using inverse Laplace transform of integrals

2 r
2 N
ƒ56 ¦ § = ƒ56
â
( + 1)
/ /
• (
/ + 1)/

1 r 1 1
ƒ 56
æ− / ç = ƒ56 æ−0 + ç = ƒ56 æ ç
+1 • / +1 / +1

= 4 . 4 .

10.26 INVERSE LAPLACE TRANSFORM BY PARTIAL FRACTION METHOD


6
Example 18: Find the Inverse Laplace Transform of •& 5š• a.

Solution: Let us convert the given function into partial fractions.


273
MSCPH501

ƒ56 º » = ƒ56 º − »
6 6 6
•& 5š• a •5_ •5/

= ƒ56 ž Ÿ − ƒ56 ž Ÿ= − . 4 .
6 6 _Œ /Œ
•5_ •5/

• 6
•& 5a• /š
Example 19: Find the Inverse Laplace Transform of .

Solution:

ƒ56 º » = ƒ56 º » = ƒ56 º »


6 6 •5_ c
•& 5a• /š (•5_)& (c)& (•5_)& (c)&

= ƒ56 º (•5_)& » + ƒ56 º (•5_)& »


•5_ c
(c)& (c)&

= _Œ
cos 4 + _Œ
sin 4 . jî 4t Q ℎQ 4t O O Jl 4 .

(• –)î
Self Assessment Question (SAQ) 14: Find the Inverse Laplace transform of

.
d Êë
(• /)Â
Self Assessment Question (SAQ) 15: Find the Inverse Laplace transform of

Self Assessment Question (SAQ) 16: Find the Inverse Laplace transform by partial fraction
.
6
•& 5–• 6/
method of

10.27 SOLUTION OF DIFFERENTIAL EQUATIONS BY LAPLACE


TRANSFORMS

Ordinary linear differential equations with constant coefficients can be easily solved by the
Laplace Transform method, without finding the general solution and the arbitrary constants. The
method wil be clear from the following examples:

Let us now discuss how the Laplace transform method solves ODEs and initial value problems.
We consider an initial value problem

J @@ + J @ + “J = ( ). J(0) = wg , 4N J @ (0) = w6

where a and b are constant. Here is the given input (driving force) applied to the mechanical or
electrical system and is the output (response to the input) to be obtained.

In Laplace’s method we do three steps:

Step 1. Setting up the subsidiary equation.

This is an algebraic equation for the transform b = ƒ(J) obtained by transforming the given
differential equation using the Laplace transform of derivatives,

274
MSCPH501

j / b − J(0) − J @ (0)l + j b − J(0)l + “b = +( ).

Where +( ) = ƒ( ).

Now collecting the Y-terms, we have the subsidiary equation as follows

( /
+ + “)b = ( + 1)J(0) + J @ (0) + +( ).

Step 2. Solution of the subsidiary equation by algebra.

We divide by and use the so-called transfer function

1
B( ) = .
( / + + “)

This gives the solution

b = ( + 1)J(0)B( ) + J @ (0)B( ) + +( )B( ).

Note that Q depends neither on r(t) nor on the initial conditions (but only on a and b).

Step 3. Inversion of Y to obtain ‡ = Î5D ¾.

Now take the inverse Laplace transform to get the solution of differential equations.

Example 20: Solve the following equation by Laplace transform

J @@ − J = ; J(0) = 1 4N J @ (0) = 1.

Solution: The given equation is

J @@ − J = ; J(0) = 1 4N J @ (0) = 1

Step 1: Taking the Laplace transform of the given equation, we get the subsidiary equation

ƒ(J @@ ) − ƒ(J) = ƒ( ).

Now using the condition of Laplace transform of derivatives with ƒ(J) = b

ƒjQ © ( )l = ©
ƒjQ( )l − ©56
Q(0) − ©5/
Q′(0) − ©5_
Q′′(0) − ⋯ − Q ©56 (0).

We get

b − J(0) − J @ (0) − b = •&


/ 6

1 1
( /
− 1)b = J(0) + J @ (0) + /
= +1+ /
. t 4J(0) = 1 4N J @ (0) = 1

275
MSCPH501

Step 2: Transfer function is given by

B( ) =
6
(•& 56)
and hence

B( ) ( + 1) 1
b = ( + 1)B( ) + = + .
/ ( / − 1) /( / − 1)

On simplifying

1 1 1
b= + − / .
( − 1) ( / − 1)

Step 3: Now taking the inverse Laplace transform to get the solution of differential equation

1 1 1
J( ) = ƒ56 b = ƒ56 + ƒ56 − /
( − 1) ( / − 1)

1 1 1
J( ) = ƒ56 + ƒ56 − ƒ56
( − 1) ( / − 1) /

J( ) = Œ
+ sinh − . 4 .

Self Assessment Question (SAQ) 17: Solve the differential equation using Laplace transform
method

N/J NJ
+ J = 0, Úℎ J = 1 4N = −1 I = 0.
NI / NI
Self Assessment Question (SAQ) 18: Solve the differential equation using Laplace transform
method

J @@ + 4J @ + 4J = 6 5Œ
, Úℎ J(0) = −2 4N J @ (0) = 8.

10.28 SUMMARY
The main purpose of Laplace transforms is the solution of differential equations and
systems of such equations, as well as corresponding initial value problems. Firstly we have
learned about Laplace transform and their various properties and theorems. Using these
properties we have find the Laplace transform of some very important function which were used
to solve many important problems. Later we have learned about Inverse Laplace transform and
their various properties. And finally we used these properties of Laplace transform and inverse
Laplace transform to solve the differential equation and many other important problems which is
in the form differential equations. To understand this unit more clearly various solved examples

276
MSCPH501

are includes almost in each section. For students assessment self assessment question is also
incorporated throughout the chapter.
10.29 GLOSSARY
Domain: one system, one type of region.
Subsidiary equation: contributory equation, secondary equation.
Arbitrary constants: random constant.
Cumbersome method: unmanageable method, bulky method.
10.30. TERMINAL QUESTIONS
Find the Laplace transform of the following:

sin cos .
e[ .
1.

t sinh a .
2.
3.
Prove that ƒj © Q( )l = (−1)© ••g jh( )l.
•g
4.
(1 − Œ ).
6
Œ
5.

Obtain the Inverse Laplace Transform of the following:

.
• ¦
(•& c• š )
6.
.

(• _ )& c
7.
.

(• – )î
8.
.
• /
•& 5/•5¦
9.
.

•& a• /š
10.

Solve the differential equation using Laplace transform method

J @@ + 9J = 6 cos 3 . Úℎ J(0) = 2 4N J @ (0) = 0


J @@ − 3J @ + 2J = 4 /= . Úℎ J(0) = −3 4N J @ (0) = 5
11.

J @@ + 2J @ + J = 5Œ , Úℎ J(0) = 1 4N J @ (0) = −2
12.
13.

10.31 ANSWERS
Self Assessment Question (SAQ)


_ 6
•& — •& 6
1. .
277
MSCPH501

2. % .
Û

+ + + + .
6 / 5/• d Êë _ 5/• / 5/•
• • •& •& •Â
3.
+
6 /
• •& c
4. .
.
a
(• & 56)(• & 5—)
5.
(• & 56)
(• & 6)&
6. .

r t .
6 •& c
c •&
7.

− .
d 5d
Êë Ê&ë d Ê&ë
•& •
8.
d &(ÁÊë)ã56
(65•)(65d Ê&ãë )
9. .
šŒ

ℎ − 4ℎ .
10. .
/ šŒ 6 šŒ
— _ _ _
12. − 4 + 1.
11.

65XY ˜Œ
˜&
13. .
5–Œ Œ (3 ).
−7
&

a
14.
5(Œ5/) (Œ5/)
¢( − 2).
&

/
16. cŒ − _Œ .
15.

17. J = cos I − sin I.


18. J = 6 5Œ − 8 5/Œ − 2 5/Œ
.
Terminal Questions:
6
•& c
1. .
6
(•5˜)&
2. .
.
/˜•
(•& 5˜& )&
3.

5. r t .
•56

5/Œ
(cos + 6 sin ).
(cos 2 − 1.5 4 ).
6.
5_Œ
7.
5–Œ Œ
(3 – 7t).
&

a
8.
5Œ (
9. ℎ3 + 4ℎ 3 ).

278
MSCPH501

ºcos 4 − sin 4 ».
5_Œ _
c
10.
11. J = 2 cos 3 − 43 .
12. J = −7 =
+4 /=
+ 4I /=
.
13. J = ž1 − + Ÿ
ŒÂ 5Œ
a
.

10.32. REFERENCES
George Arfken, H. A. Weber,: Mathematical Methods For Physicists
H.K. Dass,: Mathematical Physics, S. Chand Publication
Erwin Kreyszig,: Advanced Engineering Mathematics, Wiley Plus Publication
B.S. Rajput,: Mathematical Physics, Pragati Prakashan
Satya Prakash,: Mathematical Physics, Pragati Prakashan

10.33. SUGGESTED READINGS

George Arfken, H. A. Weber,: Mathematical Methods For Physicists


H.K. Dass,: Mathematical Physics, S. Chand Publication

279

You might also like